You are on page 1of 131

Acute Care Cardiology

Acute Care Cardiology


Jo E. Rodgers, Pharm.D., BCPS (AQ Cardiology)
University of North Carolina School of Pharmacy Chapel Hill, North Carolina

2008 American College of Clinical Pharmacy 1-245

Acute Care Cardiology


Learning Objectives: 1. Formulate treatment strategies for patients with acute decompensated heart failure (ADHF) and formulate an appropriate pharmacotherapeutical regimen for a given case scenario (e.g., warm and wet, cold and dry, other). Create an evidence-based medication regimen for a patient with acute coronary syndrome given a variety of clinical scenarios (e.g., invasive/ conservative strategy, upstream antiplatelet therapy). Describe an appropriate treatment strategy for ventricular arrhythmias using evidence-based medicine. Prepare a treatment strategy for a newly diagnosed patient with idiopathic pulmonary arterial hypertension (PAH). Develop an appropriate pharmacological and monitoring plan for antihypertensive drug therapy for managing hypertensive emergencies. (IV) vasoactive drug to treat this patient if the IV diuretic therapy fails? A. Dobutamine. B. Milrinone. C. Nesiritide. D. IV nitroglycerin. 3. H.E. is a 53-year-old woman admitted to the hospital after the worst headache she has ever experienced. Her medical history includes exertional asthma, poorly controlled hypertension, and hyperlipidemia. She is drug noncompliant and has not taken her BP drugs, including clonidine, in 4 days. Vital signs include BP 220/100 mm Hg and heart rate (HR) 65 beats/minute. She receives a diagnosis of a cerebrovascular accident and hypertensive emergency. Which one of the following choices is the best management option for this patients hypertensive emergency? A. Fenoldopam 0.1 mcg/kg/minute. B. Nicardipine 5 mg/hour. C. Labetalol 0.5 mg/minute. D. Enalaprilat 0.625 mg IV every 6 hours. W.M. is a 69-year-old man who presents to the hospital with a 3-mm ST-elevation myocardial infarction (MI) within 2 hours of chest pain onset. He is given clopidogrel 300 mg once daily and instructed to chew aspirin 81 mg 4 times/ day. Abciximab and unfractionated heparin are initiated as he is being wheeled to the cardiac catheterization laboratory for primary percutaneous coronary intervention (PCI). Four hours after returning to the intensive care unit, a complete blood cell count shows a platelet count of 15 103 cells/mm3. Which one of the following choices is the most likely culprit of this patients thrombocytopenia? A. Clopidogrel. B. Aspirin. C. Unfractionated heparin. D. Abciximab. The Sudden Cardiac Death in Heart Failure trial evaluated the efcacy of amiodarone or internal cardioverter-debrillator (ICD) versus placebo in preventing all-cause mortality in ischemic and non-ischemic NYHA class II and III patients with HF. There was a 7.2% absolute risk reduction and a 23% relative risk reduction in all-cause mortality at 60 months with an ICD versus placebo. What is the number of patients needed to treat with an ICD to prevent one death versus placebo? A. 1.3 patients. B. 4.3 patients.

2.

3.

4.

5.

4.

Self-Assessment Questions: Answers to these questions may be found at the end of this chapter. 1. A.A. is a 25-year-old woman with a new diagnosis of idiopathic PAH. Her home drugs include warfarin 5 mg/day, furosemide 60 mg 2 times/ day, and bosentan 62.5 mg 2 times/day. Which one of the following is the best contraceptive strategy for this patient? A. Estrogen-progesterone oral contraceptive. B. Injectable hormonal contraceptive. C. Any hormonal contraceptive and barrier methods. D. Barrier method only. F.F. is a 64-year-old man with New York Heart Association (NYHA) class III heart failure (HF) admitted for increased shortness of breath, dyspnea on exertion, and decreased exercise tolerance. He admits to dietary noncompliance and has indulged in bacon and eggs, popcorn, and french fries in the past 72 hours. His extremities appear well perfused, but he does have 3+ pitting edema in all four extremities. F.F.s blood pressure (BP) is 115/75 mm Hg. Which one of the following is the most appropriate intravenous 5.

2.

2008 American College of Clinical Pharmacy 1-246

Acute Care Cardiology


C. 13.8 patients. D. 43.4 patients. 6. A.D. is a 52-year-old woman with a history of witnessed cardiac arrest in a shopping mall; she was resuscitated with an automatic external debrillator device. On electrophysiology study, she is found to have inducible ventricular tachycardia. Which one of the following has signicantly decreased the incidence of sudden cardiac death over other therapies in a patient population such as this (secondary prevention)? A. Propafenone. B. Amiodarone. C. ICD. D. Metoprolol. S.V. is a 75-year-old woman with a history of NYHA class III HF (LV ejection fraction [LVEF] 25%) and multiple nonST-elevation MIs. She had an episode of sustained ventricular tachycardia during hospitalization for pneumonia. Her QTc interval was 380 milliseconds on the telemetry monitor. Given her comorbidities, what is the most appropriate treatment option for S.V.? A. Procainamide. B. Metoprolol. C. IV magnesium. D. Amiodarone. You are working on a review article on newer treatment strategies for antiplatelet and anticoagulant therapy during PCI. You want to ensure that you retrieve all relevant clinical trials and related articles on your subject. You have searched MEDLINE and found 42 articles, but a few that you have seen before were not pulled up in a search by Medical Subject Headings. Which one of the following comprehensive databases should be searched next to ensure that you did not miss any key articles? A. International Pharmaceutical Abstracts. B. Iowa Drug Information Service. C. Clin-Alert. D. Excerpta Medica. A physician on your team wants you to do the paperwork regarding an adverse drug reaction (ADR) that a patient on your team experienced caused by nesiritide. The patient had severe hypotension after the initial bolus dose of nesiritide for treatment of decompensated HF, even though his BP was safely in the normal range before therapy initiation. The hypotension led to decreased renal perfusion, resulting in oliguric acute renal failure and subsequent hemodialysis initiation. The patient had no known renal insufciency before developing this complication. Which one of the following statements is correct regarding Joint Commission on Accreditation of Healthcare Organizations requirements for institutional ADR reporting? A. A MedWatch form explaining the scenario in which the ADR occurred must be completed. B. I n s t i t u t io n s m u s t c r e a t e t h e i r ow n denition of ADR and make sure that practitioners are familiar with it. C. Hospital practitioners/staff must use the Naranjo Algorithm for assessing the severity of the ADR. D. Only severe or life-threatening ADRs need to be reported. 10. Your Pharmacy and Therapeutics Committee wants you to perform a pharmacoeconomic analysis on a new drug available to treat decompensated HF. This drug works through a specic mechanism of action. Unlike other available inotropics (dobutamine and milrinone), which can increase mortality over time, this drug appears to reduce mortality in the longterm setting. However, the price is 10 times higher per day than other available drugs. Your ndings will be presented at the next P and T committee meeting to make a formulary decision about which medications to stock in your hospital pharmacy. What type of pharmacoeconomic analysis should be used to determine whether this new drug would be a better formulary choice than currently available products? A. Cost-minimization analysis. B. Cost-effectiveness analysis. C. Cost-benet analysis. D. Cost-utility analysis.

7.

8.

9.

2008 American College of Clinical Pharmacy 1-247

Acute Care Cardiology I. ACUTE DECOMPENSATED HEART FAILURE ADHF may occur because of one of the following: 1. An acute worsening of chronic HF, 2. A new cardiac event (e.g., MI, atrial brillation), or 3. An acute massive MI whose initial presentation is severe HF. A. Hemodynamic Monitoring
Table 1. Hemodynamic Values in Patients with ADHF and Sepsisa Parameter Normal Value 80100 Typical ADHF Value 6080 Typical Sepsis Value 6080

Mean arterial pressure (MAP) (mm Hg) Heart rate 6080 7090 90100 (beats/minute) Cardiac output (CO) 47 24 58 (L/minute) Cardiac index (CI) 2.83.6 1.32 3.54 (L/minute/m2) Pulmonary capillary wedge pressure (PCWP) 812b 1830 58 (mm Hg) Systemic vascular resistance (SVR) (dynes/ 8001200 15003000 300800 second/cm5) Central venous pressure 26 615 26 (CVP) (mm Hg) a MAP = DBP + 1/3(SBP DBP) (SBP = systolic blood pressure; DBP = diastolic blood pressure), CI = CO/BSA (BSA = body surface area), SVR = (MAP CVP)(80)/CO. b 1518 mm Hg often desired/optimal in patients with HF to ensure optimal lling pressures.

1. Hemodynamic equations a. BP = cardiac output (CO) systemic vascular resistance b. CO = stroke volume (SV) HR 2. Parameters inuencing CO a. Heart rate i. Also known as chronotropy ii. Controlled by autonomic nervous system b. Stroke volume i. Amount of blood ejected by the heart during systole ii. Controlled by four factors: (a) Inotropyventricular contractility or squeezing force. (b) Afterloadresistance or pressure the left ventricular (LV) pumps against to eject blood into the aorta; estimated by the systemic vascular resistance. (c) Preloadamount of tension applied to the LV before contraction; equivalent to LF end diastolic volume; estimated by the pulmonary capillary wedge pressure. (d) Lusitropydiastolic relaxation of the ventricle.

2008 American College of Clinical Pharmacy 1-248

Acute Care Cardiology 3. Forrester hemodynamic classication

4 3
Cardiac 2.2 2 index (L/minute)

Category I: Normal Warm and dry

Category II: Pulmonary congestion Warm and wet

Category III: Hypoperfusion Cold and dry

Category IV: Pulmonary congestion and hypoperfusion Cold and wet

10

15

18
PCWP

20
mm Hg

25

Figure 1. Forrester Hemodynamic Classication Reprinted with permission from the American Medical Association. Nohria A, Lewis E, Warner Stevenson L. Medical management of advanced heart failure. JAMA 2002;287:62840.

B. Clinical Presentation of ADHF


Table 2. Signs and Symptoms of ADHF Congestion (Elevated Pulmonary Capillary Wedge Pressure) Dyspnea on exertion or at rest Orthopnea, paroxysmal nocturnal dyspnea Peripheral edema Rales Early satiety, nausea/vomiting Ascites Hepatomegaly, splenomegaly Jugular venous distention Hepatojugular reux Hypoperfusion (Reduced CO) Fatigue Altered mental status or sleepiness Cold extremities Worsening renal function Narrow pulse pressure Hypotension Hyponatremia

C. Guideline Recommendations for ADHF


Table 3. Level of Evidence and Class of Effect for ADHF Guidelines Level of Evidence Level A Randomized, controlled clinical trials Level B Case-control or cohort studies (post hoc, subgroup analysis, meta-analysis) Level C Expert opinion or observational studies Class of effect Recommended for routine clinical practice in all patients Should be considered for the majority of patients May be considered for individual patients Is not recommended for any patient

2008 American College of Clinical Pharmacy 1-249

Acute Care Cardiology D. Drug Therapy Recommendations


Table 4. Overview of ADHF Guideline Recommendations Diuretic Therapy 1. Recommended as an IV loop diuretic for patients admitted with uid overload (LOE = B). 2. When response to diuretics is minimal, the following options should be considered: a. Fluid and sodium restriction, b. Initiation of increased doses or continuous infusion of loop diuretic, c. Addition of a second type of diuretic (metolazone or chlorothiazide), or d. Ultraltration. (LOE = C). Inotropic Therapy 1. May be considered to relieve symptoms and improve end-organ function in patients with diminished peripheral perfusion or end-organ dysfunction (low output syndrome), particularly if: a. marginal systolic blood pressure (less than 90 mm Hg), b. symptomatic hypotension despite adequate lling pressure, or c. unresponsive to, or intolerant of, IV vasodilators (LOE = C) 2. May be considered in similar patients with evidence of uid overload if they respond poorly to IV diuretics or manifest diminished or worsening renal function (LOE = C). Vasodilator Therapy 1. May be considered in addition to IV loop diuretics to rapidly improve symptoms in patients without symptomatic hypotension (LOE = B). 2. May be considered in patients with persistent symptoms despite maximal loop diuretics and oral drug therapy (LOE = C). 3. When adjunctive therapy is required in addition to loop diuretics, IV vasodilators should be considered over inotropic drugs (LOE = B). Invasive Hemodynamic Monitoring 1. Routine use of hemodynamic monitoring with invasive IV lines (e.g., Swan-Ganz pulmonary artery catheters) is not recommended (LOE = A).

E. Diuretic Medications 1. Loop diuretics (ascending limb of loop of Henle) a. Most widely used and most potent, effective at low glomerular ltration rate (less than 30 mL/minute) b. Furosemide (Lasix) most commonly used 2. Thiazides (distal tubule) a. Relatively weak diuretics when used alone, not effective at low glomerular ltration rate b. Reserved for add-on therapy in patients refractory to loop diuretics 3. Therapy for patients with diuretic resistance a. Increase dose rather than frequency of loop diuretic. (Note: Ceiling effect at approximately 160200 mg IV furosemide.) b. Add a second diuretic with a different mechanism of action. i. Hydrochlorothiazide 12.525 mg PO daily, metolazone 2.55 mg PO daily ii. Chlorothiazide 250500 mg IV daily; consider if gastrointestinal edema (poor PO absorption) c. Continuous infusion of loop diuretic i. Furosemide 0.1 mg/kg/hour IV doubled every 48 hours to a maximum of 0.4 mg/kg/ hour 4. Adverse effects: electrolyte depletion (potassium and magnesium), worsening renal function

2008 American College of Clinical Pharmacy 1-250

Acute Care Cardiology

F. Inotropic Medications
Table 5. Inotropic Therapy for ADHF Dobutamine (Dobutrex) Mechanism of action Milrinone (Primacor)

PDE inhibitor: inhibits cAMP breakdown in 1-agonist: stimulates AC to convert ATP to cAMP to CO; slight peripheral vasodilation. heart to CO and in vascular smooth muscle to SVR. Clinical effects Positive inotropic, chronotropic, and lusitropic Positive inotropic and lusitropic effects, effects no chronotropic effects Indication ADHF short-term managementeffective in cold and wet exacerbations with hypoperfusion and congestion Dosing Start at 2.55 mcg/kg/minute; 50 mcg/kg IV bolus for 10 minutes, may titrate up to maximum 20 mcg/kg/minute then 0.3750.75 mcg/kg/minute Typical dose 5 mcg/kg/minute xed infusion No bolus, 0.10.5 mcg/kg/minute xed infusion Half-life 2 minutes 1 hour, prolonged to 23 hours if CrCl < 50 mL/ min Elimination Hepatically metabolized (inactive) and renally 90% renal eliminated Proarrhythmia, hypotension (avoid bolus), Adverse effects Proarrhythmia, tachycardia, hypokalemia, myocardial ischemia, and tachyphylaxis (> 72 tachycardia, < 1% thrombocytopenia, hours), possible increased mortality with long- possible increased mortality with long-term use term use Other comments Recommended if hypotensive. Recommended if receiving a -blocker. AC = adenylate cyclase; ADHF = acute decompensated heart failure; ATP = adenosine triphosphate; cAMP = cyclic adenosine monophosphate; CO = cardiac output; CrCl = creatinine clearance; PDE = phosphodiesterase; SVR = systemic vascular resistance.

2008 American College of Clinical Pharmacy 1-251

Acute Care Cardiology G. Vasodilator Therapy


Table 6. Vasodilator Therapy for ADHF Sodium Nitroprusside Nesiritide (Nipride) (Natrecor) Nitric oxideinduced Recombinant B-type natriuretic stimulation of GC to peptide binds to natriuretic peptide convert GTP to cGMP. receptor A to stimulate guanylate cyclase and production of cGMP. Natriuretic mechanism unknown. IV Nitroglycerin

Combines with sulfhydryl groups in vascular endothelium to create s-nitrosothiol compounds, which mimic nitric oxides stimulation of guanylate cyclase and production of cGMP. Clinical effects Balanced arterial and Hemodynamic effects: PCWP and Preferential venous vasodilator venous vasodilator SVR, minimal changes in HR or CI > arterial vasodilator, arterial Neurohormonal effects: NE, ET-1, vasodilation at high doses and aldosterone Natriuretic effects: urine output and sodium excretion Indication Warm and wet ADHF, Warm and wet ADHF, alternative Warm and wet ADHF, ACS, or hypertensive crises hypertensive crises to inotropes in cold and wet ADHF Dosing 0.30.5 mcg/kg/minute, 2 mcg/kg bolus, 0.01 mcg/kg/ 5 mcg/minute, by 5 mcg/minute by 0.5 mcg/kg/minute minute, by 0.005 mcg/kg/minute up to 200 mcg/minute up to 3 mcg/kg/minute up to 0.03 mcg/kg/minute Typical dose 0.51 mcg/kg/minute 0.01 mcg/kg/minute xed infusion 2575 mcg/minute, titrated to May omit bolus if low SBP response Half-life < 10 minutes 20 minutes 14 minutes Natriuretic peptide receptor C Inactive metabolites in urine Elimination Cyanide hepatically metabolized, (no renal/hepatic adjustment) thiocyanate renally excreted Adverse effects Hypotension, cyanide or Primarily hypotension (up to Hypotension, reex tachycardia, thiocyanate toxicity 1 hour), tachycardia (less than headache, tachyphylaxis inotropes) ACS = acute coronary syndrome; ADHF = acute decompensated heart failure; CI = cardiac index; cGMP = cyclic guanine monophosphate; ET = endothelin; GC = guanylate cyclase; guanosine triphosphate = GTP; HR = heart rate; NE = norepinephrine; PAC = pulmonary artery catheter; PCWP = pulmonary capillary wedge pressure; SBP = systolic blood pressure; SVR = systemic vascular resistance.

Mechanism of action

2008 American College of Clinical Pharmacy 1-252

Acute Care Cardiology

Patient Cases 1. D.D. is a 72-year-old man admitted to the ward team for HF decompensation. D.D. notes progressively increased dyspnea on exertion (now 10 feet, previously 30 feet) and orthopnea (now four pillows, previously two pillows), increasing bilateral lower extremity swelling (3+), a 13.6-kg weight gain in the past 3 weeks, and dietary noncompliance. He has a history of idiopathic dilated cardiomyopathy (LVEF 25%, NYHA class III), paroxysmal atrial brillation, and hyperlipidemia. Pertinent laboratory ndings: B-type natriuretic peptide 2300 pg/mL (050 pg/mL), potassium+ 4.9 mEq/L, blood urea nitrogen (BUN) 22 mg/dL, serum creatinine (SCr) 1 mg/dL, aspartate aminotransferase 40 IU/L, alanine aminotransferase 42 IU/L, international normalized ratio 1.3, partial thromboplastin time 42 seconds, BP 108/62 mm Hg, and HR 82 beats/minute. Home drugs include carvedilol 12.5 mg 2 times/day, lisinopril 40 mg/day, furosemide 80 mg 2 times/day, spironolactone 25 mg/day, and digoxin 0.125 mg/day. Which one of the following is the best option for treating his ADHF? A. Carvedilol 25 mg 2 times/day. B. Nesiritide 2 mcg/kg bolus, then 0.01 mcg/kg/minute. C. Furosemide 120 mg IV 2 times/day. D. Milrinone 0.5 mcg/kg/minute. 2. After being started on IV loop diuretics and metolazone 2.5 mg with only minimal urine output and rising creatinine (CrCl now 30 mL/minute), he is transferred to the coronary care unit for further management of diuretic-refractory decompensated HF. His carvedilol dose is now 6.25 mg 2 times/day, and lisinopril and spironolactone are being withheld. His BP is 110/75 mm Hg, and his HR is 75 beats/minute. How else should his decompensated HF be treated? A. Nesiritide 2 mcg/kg bolus, then 0.01 mcg/kg/minute. B. Sodium nitroprusside 0.3 mg/kg/minute. C. Dobutamine 5 mcg/kg/minute. D. Milrinone 0.5 mcg/kg/minute. D.D. initially responds with 2 L of urine output overnight and weight decreased by 1 kg the next day. However, by day 5, his urine output has diminished again, and his SCr has risen to 4.3 mg/dL. He was drowsy and confused this morning during rounds. His extremities are cool and cyanotic, BP is 102/58 mm Hg, and HR is 98 beats/ minute. It is believed that he is no longer responding to his current regimen. A Swan-Ganz catheter is placed to determine further management. Hemodynamic values are cardiac index 1.5 L/minute/m2, systemic vascular resistance 2650 dynes/cm 5, and pulmonary capillary wedge pressure 30 mm Hg. Which of the following is the most appropriate medication based on his current symptoms? A. Milrinone 0.2 mcg/kg/minute. B. Dobutamine 5 mcg/kg/minute. C. Nitroglycerin 20 mcg/minute. D. Phenylephrine 20 mcg/kg/minute.

3.

II. ARRHYTHMIAS A. Guideline Recommendations for Arrhythmias


Table 7. Level of Evidence and Classication for Arrhythmia Guidelines Level A Level B Level C Level of Evidence Multiple (35) clinical trials or registries or meta-analyses. Limited (23) clinical trials or registries; generally, a single randomized clinical trial or several non-randomized trials. Few (12) clinical trials/registries; may be only expert opinion, case series, or standard care.

2008 American College of Clinical Pharmacy 1-253

Acute Care Cardiology


Table 7. Level of Evidence and Classication for Dysrhythmia Guidelines (continued) Class I Class IIa Class IIb Class III Indeterminate Class of Effect Agreement that benet strongly outweighs the risk of the procedure or treatment, and it should be performed because of its efcacy. May be conicting evidence or diverging opinions, but the benet is greater than the risk; it is reasonable to consider the procedure or treatment because of its efcacy. May be conicting evidence or diverging opinions, but benet outweighs the risk; procedure or treatment may be considered, but additional data are needed. Agreement that the risk outweighs the benet; procedure or treatment is not effective and should not be performed and may be harmful. An area of ongoing research but no denitive recommendations for or against.

A. Terminology
Table 8. Common Arrhythmia Terms and Denitions Term QT interval QTc interval Supraventricular arrhythmias Ventricular arrhythmias Ventricular tachycardia Denition Time from beginning of ventricular depolarization to end of ventricular repolarization QT interval corrected for heart rate (HR) Typically narrow QRS complex arrhythmiasatrial brillation, atrial utter, multifocal atrial tachycardia (MAT), paroxysmal supraventricular tachycardia, including atrialventricular nodal reentrant tachycardia (AVNRT) and AV reentrant tachycardia (AVRT) Typically wide QRS complex arrhythmiasventricular tachycardia (VT) At least three consecutive premature ventricular contractions at a rate > 100 beats/minute - monomorphic ventricular tachycardia and polymorphic ventricular tachycardia (e.g., torsade de pointes) Spontaneously terminates in less than 30 seconds

Nonsustained ventricular tachycardia Sustained ventricular Lasts > 30 seconds or requires cardioversion because of hemodynamic instability tachycardia Torsade de pointes Induced primarily when QTc interval > 500 milliseconds Sudden cardiac death Rapid death caused by pulseless VT or ventricular brillation (VF), pulseless electrical activity, or asystole Proarrhythmic events New type of arrhythmia or worsening of existing arrhythmia after initiation of an antiarrhythmic medication (drug-induced), most commonly torsade de pointes Structural heart HF, status post-acute myocardial infarction, valvular heart disease, left valvular (LV) hypertrophy disease Note: Class IA and IC medications are contraindicated because of increased proarrhythmia risk. Debrillation Energy requirement (joules) necessary for debrillation of an arrhythmia into sinus rhythm threshold

2008 American College of Clinical Pharmacy 1-254

Acute Care Cardiology


Table 9. Antiarrhythmic Drug Effects Na+ channel blockers ECG Effect Class IA Quinidine, procainamide, disopyramide QRS, QT Class IB Lidocaine, mexiletine QRS, QT Class IC Propafenone, ecainide, moricizine QRS -blockers Metoprolol, esmolol, atenolol PR Class II K+ channel blockers Amiodarone, sotalol, dofetilide, ibutilide QT Class III Ca2+ channel blockers Diltiazem, verapamil PR Class IV AV = atrioventricular; ECG = electrocardiography; K+ = potassium plus. Table 10. Antiarrhythmic Drug Properties Drug Quinidine Mechanism of Action, Pharmacokinetics Adverse Effects, and Drug Interactions MOA: strong vagolytic and anticholinergic properties, Na+ channel blockade AEs: Nausea, vomiting, diarrhea 30%, use-dependent proarrhythmia rst 72 hours (torsade de pointes), cinchonism (CNS symptoms, tinnitus), and HF (causes exacerbation) DI: warfarin, digoxin MOA: Na+ channel blockade AEs: decrease dose in renal and liver dysfunction (active metabolite NAPA accumulates), lupus-like syndrome in 30% of patients if > 6 months treatment, hypotension (IV): 5%, and HF (causes exacerbation) Dosing by Indication Avoid use for atrial brillation cardioversion caused by GI AEs Ab maintenance: sulfate: 200400 mg every 6 hours Gluconate: 324 mg every 812 hours Primary Effect Slows depolarization Slows depolarization Slows depolarization Slows AV nodal conduction Slows repolarization Slows AV nodal conduction

Procainamide

Disopyramide

Lidocaine

Mexiletine

Ab conversion (IV): 1 g for 30 minutes, followed by 2 mg/ minute over one hour (efcacy 51% at 1 hour) Ab maintenance: Not recommended VT (preserved LVEF > 40%): 20 mg/minute loading infusion until 17 mg/kg, arrhythmia ceases, or QRS widens > 50% VT maintenance: 24 mg/minute MOA: potent Na+ and M2 blockade; strong negative Ab conversion: inotropic effect 200 mg PO q4h (maximum 800 mg) AEs: anticholinergic side effects (urinary retention, Maintenance: constipation, tachycardia, dry eyes), heart block 400600 mg/day in divided doses and HF (causes exacerbation) CI: glaucoma DI: 3A4 inhibitors MOA: inactive Na+ channel blocker VT/VF conversion: Dose adjust in HF, liver failure, elderly patients, pulseless VT/VF or stable VT with renal dysfunction LVEF > 40%: 11.5 mg/kg IVP 1; AEs: CNS symptoms, perioral numbness, seizures, repeat 0.50.75 mg/kg every 35 minutes confusion, blurry vision, tinnitus (maximum 3 mg/kg) CI: third-degree AV heart block LVEF < 40%: 0.50.75 mg/kg IVP; repeat 0.50.75 mg/kg every 35 minutes (maximum 3 mg/kg) VT maintenance: 12 mg/minute VT maintenance: 200300 mg every 8 hours MOA: inactive Na+ channel blocker CI: third-degree AV heart block

2008 American College of Clinical Pharmacy 1-255

Acute Care Cardiology


Drug Propafenone Mechanism of Action, Pharmacokinetics Adverse Effects, and Drug Interactions MOA: Na+ and Ca2+ channel blocker, -blocker AEs: metallic taste, dizziness CI: HF (causes exacerbation), liver disease DI: digoxin by 70%; warfarin by 50% MOA: strong Na+ channel blockade; vagolytic, anticholinergic, and negative inotropic effects AEs: dizziness, tremor CI: HF (causes exacerbation), CAD DI: digoxin by 25% MOA: sodium, K+, calcium channel blocker, -blocker AEs: pulmonary brosis 3%17%, hyperthyroidism 3%; hypothyroidism 30%50%; neurologic toxicity 20%40%; blue-gray skin 15%; torsade < 1%; AV block 14%; hypotension, phlebitis (IV) CI: iodine hypersensitivity, hyperthyroidism, heart block DI: warfarin, digoxin, HMG-CoA-reductase inhibitors, phenytoin 50% (among others) Inhibits cytochrome P450 (CYP) metabolism: CYP2C9, CYP2D6, and CYP3A4 Inhibits P-glycoprotein in GI tract to increase digoxin bioavailability Maximum simvastatin dosage 20 mg/day *Does not increase mortality in patients with HF. MOA: K+ channel blocker, also 1- and 2-receptor blocking properties AEs: HF exacerbation, bradycardia, wheezing; 3%8% torsade within 3 days of initiation; bronchospasm CI: baseline QTc > 440 milliseconds or CrCl < 40 mL/minute in atrial arrhythmias
*Double dose every 3 days; NTE QTc > 500 milliseconds; hospitalization mandatory for initiation

Dosing by Indication Ab conversion: 600 mg PO 1 (efcacy 45% at 3 hours) Ab maintenance: 150300 mg every 812 hours Ab conversion: 300 mg PO 1 (efcacy 50% at 3 hours) Ab maintenance: 50150 mg every 12 hours Ab conversion: IV: 57 mg/kg over 3060 minutes then 1.21.8 g/day continuous IV or in divided oral dose until 10 g PO/5gIV PO: 400 mg 3 times/day for 57 days (until 10 g) Ab maintenance: 200400 mg/day Pulseless VT/VF conversion: 300 mg IVP 1 in 20 mL D5W; repeat 150 mg IVP every 35 minutes (ARREST trial) Or 5 mg/kg IVP in 30 mL D5W; repeat 2.5 mg/kg (ALIVE trial) Stable VT: 150 mg IVP 1 over 10 minutes VT/VF maintenance: 1 mg/minute 6 hours, followed by 0.5 mg/minute (maximum 2.2 g/day) Not effective for conversion Ab maintenance: CrCl 80 mg 2 times/day > 60 mL/minute 80 mg/day 4060 mL/minute Contraindicated < 40 mL/minute VT maintenance: CrCl 80 mg 2 times/day > 60 mL/minute 80 mg/day 3060 mL/minute 80 mg every 3648 hours 1030 mL/minute 80 mg > every 48 hours < 10 mL/minute Ab conversion: CrCl 500 mcg PO 2 times/day > 60 mL/minute 250 mcg PO 2 times/day 4060 mL/minute 125 mcg PO 2 times/day 2040 mL/minute (efcacy 12% at 1 month) Ab maintenance: Titrate upward based on QTc NTE 500 milliseconds or > 15% in QTc after initial 5 doses (hospitalization mandatory!)

Flecainide

Amiodarone

Sotalol

Dofetilide

MOA: Pure K+ channel blocker only AEs: torsade (0.8%; 4% if no renal adjust), dizziness, diarrhea DI: 3A4 inhibitors or drugs secreted by kidney: cimetidine, ketoconazole, verapamil, trimethoprim, prochlorperazine, megestrol and hydrochlorothiazide CI: baseline QTc > 440 milliseconds or CrCl < 20 mL/minute
*Does not increase mortality in patients with HF.

2008 American College of Clinical Pharmacy 1-256

Acute Care Cardiology


Mechanism of Action, Pharmacokinetics Adverse Dosing by Indication Effects, and Drug Interactions Ibutilide MOA: K+ channel blocker; also Na+ and Ab conversion: 1 mg 1 IV ( 60 kg) or -blocking properties 0.01 mg/kg (< 60 kg), repeat in 10 minutes AEs: 8% torsade risk; requires ECG monitoring if ineffective during and 4 hours after cardioversion (efcacy 47% at 90 minutes) DI: 3A4 inhibitors or QT prolonging drugs CI: receiving concomitant antiarrhythmics or QTc Slightly more effective for converting atrial > 440 milliseconds before initiation utter than atrial brillation AE = adverse effect; Ab = atrial brillation; ALIVE = amiodarone versus lidocaine in prehospital refractory ventricular brillation evaluation; ARREST = amiodarone out of hospital resuscitation of refractory sustained ventricular tachycardia; CAD = coronary artery disease; CI = contraindications; CrCl = creatinine clearance; CNS = central nervous system; ECG = electrocardiogram; DI = drug interactions; GI = gastrointestinal; HF = heart failure; HMG-CoA = 3-hydroxy-3-methylglutaryl coenzyme A; IV = intravenous; IVP = intravenous push; K+ = potassium plus; MOA = mechanism of action; NTE = not to exceed; VF = ventricular brillation; VT = ventricular tachycardia. Drug

C. Acute Tachyarrhythmias 1. Tachycardia with a pulse a. Give oxygen; assess airway, breathing, and circulation and treat reversible causes. b. If unstable, administer immediate direct current cardioversion (DCC). c. If stable, determine whether QRS complex is narrow or wide. i. Narrow complex tachycardia (usually atrial arrhythmias). (a) Regular rhythm (1) Supraventricular tachycardia (SVT) or sinus tachycardia likely (a) Vagal maneuvers (class I, level of evidence [LOE B]) and/or adenosine 6 mg IVP, followed by 12 mg IVP (may repeat 1 time) (class I, LOE A) (i) Use adenosine cautiously in severe coronary artery disease (CAD) because it may produce atrial brillation and induce myocaridal ischemia.
Table 11. Management of Supraventricular Tachycardia Scenario Management If converts, likely AT, AVNRT, AVRT, or WPW If AVNRT Any of the following depending on clinical presentation: Catheter ablation -ORVerapamil, diltiazem, -blockers, sotalol, amiodarone -ORFlecainide, propafenonea -ORIf infrequent, no therapy or pill-in-the-pocket (LOE varies depending on clinical presentation) If WPW Same as above; however, AVOID verapamil, diltiazem, and digoxin If persistent with AV block, likely atrial utter or AT IV ibutilide,b procainamide, or ecainide plus AV nodal blocking agents or overdrive pacing/DCC a Relatively contraindicated for patients with coronary artery disease, LV dysfunction, or other signicant heart disease., bIbutilide is especially effective if atrial utter but should not be used if LVEF < 30% because of risk of polymorphic ventricular tachycardia., AT = atrial tachycardia; AV = atrioventricular; AVNRT = atrioventricular nodal reciprocating/reentrant tachycardia; AVRT = atrioventricular reciprocating/reentrant tachycardia; DCC = direct current cardioversion; SVT = supraventricular tachycardia; WPW = Wolff-Parkinson-White [syndrome].

2008 American College of Clinical Pharmacy 1-257

Acute Care Cardiology

(b) Irregular rhythm (1) Atrial brillation or atrial utter likely (a) Rate control versus rhythm control (i) Rate control recommended for patients with persistent or permanent atrial brillation (class I, LOE B). (ii) If hemodynamically unstable, DCC recommended (class I, LOE C).
Table 12. Management of Atrial Fibrillation Type Management Newly Discovered AF Paroxysmal No therapy needed unless symptoms, AC as needed Persistent If accepted as permanent, AC and RC as needed; otherwise, consider AAD with DCC Recurrent AF Paroxysmal If minimal or no symptoms, AC and RC as needed Persistent If disabling symptoms, use anticoagulation and rate control as needed with AAD (ablation if AAD fails) Permanent AF AC and RC as needed AAD = antiarrhythmic drug; AC = anticoagulation; AF = atrial brillation; DCC = direct current cardioversion; RC = rate control.

Table 13. Rate Control and Rhythm Control Rate Control -blockers or nondihydropyridine calcium channel blockers (diltiazem, verapamil) (class I, LOE B) If HF and no accessory pathway present Digoxina (class I, LOE B) or amiodarone (class IIa, LOE C) If accessory pathway present Amiodarone (class IIa, LOE C) Rhythm Control Unstable and known duration < 48 hours Before DCC, IV UFH immediately beforehand DCC (class I, LOE C) Unstable and duration unknown Before DCC, TEE to rule out thrombus + IV UFH before DCC if or > 48 hours possible DCC (class I, LOE C) Stable and duration unknown Before DCC, RC + warfarin (international normalized ratio 23) 34 or > 48 hours weeks before and 4 weeks after (class I, LOE C) If AF for up to 7 days, either elective DCC or chemical cardioversionb - Flecainide, dofetilide, propafenone, ibutilide (class I, LOE A) or amiodarone (class IIa, LOE A) - Digoxin and sotalol may be harmful when used for pharmacological cardioversion of AF and are not recommended (class II, LOE A). If AF more than 7 days, either elective DCC or chemical cardioversion - Dofetilide (class I, LOE A), amiodarone, ibutilide (class IIa, LOE A)b a If paroxysmal AF, avoid digoxin (class III, LOE B), bQuinidine, procainamide, disopyramide, and dofetilide should NOT be started out of hospital for conversion of AF to sinus rhythm (class III, LOE B). AF = atrial brillation; DCC = direct current cardioversion; IV = intravenous; LOE = level of evidence; RC = rate control; TEE = transesophageal echocardiograph; UFH = unfractionated heparin. General presentation ii. Wide complex tachycardia (usually ventricular arrhythmias)

(a) Ventricular tachycardia or unknown mechanism

2008 American College of Clinical Pharmacy 1-258

Acute Care Cardiology (1) IV procainamide, amiodarone (class I, LOE B), or lidocaine (class IIb, B) (2) Amiodarone preferred if LV dysfunction or signs of HF (class I, LOE B) (3) Prepare for synchronized DCC if drug therapy fails. (b) Denite supraventricular tachycardia (see narrow complex tachycardia) (1) If supraventricular tachycardia and bundle branch block (BBB), treat as supraventricular tachycardia (see above). (2) If pre-excited supraventricular tachycardia, IV, ibutilide or procainamide (class I, LOE B) or DCC (class I, LOE C) 2. Premature ventricular contractions a. Avoid ecainide, encainide, and moricizine (class IC) post MI due to increased mortality. . -blockers are useful for controlling symptomatic premature ventricular contractions. 3. Ventricular tachycardia a. Nonsustained ventricular tachycardia i. If asymptomatic, no therapy required. ii. If symptomatic, -blockers. (a) If unresponsive, use amiodarone or sotalol. b. Sustained ventricular tachycardia (QTc not prolonged)
Table 14. Management of Sustained Ventricular Tachycardia Management Stable Sustained Monomorphic VT General presentation Procainamide IV (class IIa, LOE = B) Associated with AMI Lidocaine IV (class IIb, LOE = C) Associated with CAD and idiopathic VT Amiodarone IV, -blockers, procainamide IV (class IIa, (Repetitive) LOE C) (Note: avoid procainamide if LV disfunction) Unstable Sustained Monomorphic VT General presentation DCC (class I, LOE = C) Refractory to DCC Amiodarone 150 mg IV for 10 minutes (class IIa, LOE = C) Recurrent with procainamide Amiodarone 150 mg IV for 10 minutes (class IIa, LOE = C) AMI = acute myocardial infarction; CAD = coronary artery disease; DCC = direct current cardioversion; IV = intravenous; LOE = level of evidence; VT = ventricular tachycardia. Presentation

c. Polymorphic ventricular tachycardia (prolonged QTc) i. Torsade de pointes (a) Induced primarily when QTc interval more than 500 milliseconds (b) Withdrawal of any offending medications that may induce torsade de pointes and correction of electrolyte abnormalities (low Mg2+ or potassium+) is recommended (class I, LOE = A). (1) Class I and class III antiarrhythmic drugs (2) Drug interactions via cytochrome P450 3A4 metabolized drugs: azole antifungals, erythromycin, clarithromycin, or QT prolonging drugs: pentamidine, haloperidol, ziprasidone, droperidol, sulfamethoxazole/ trimethoprim, promethazine.

2008 American College of Clinical Pharmacy 1-259

Acute Care Cardiology


Table 15. Management of Torsade de Pointes Presentation Management Unstable DCC with sedation (class I, LOE = B) Few episodes of TdP with prolonged QT IV Mg2+ 12 g IVP (maximum 16 g/24 hoursa) (class IIa, LOE = B) Heart block and symptomatic bradycardia Pacemaker placement (class I, LOE A) Recurrent -blockers (especially if ischemia suspected) (class I, LOE = B) If associated with AMI Lidocaine 11.5 mg/kg IVP (class IIb, LOE = C) a If normal renal function. AMI = acute myocardial infarction; DCC = direct current cardioversion; IVP = intravenous push; LOE = level of evidence; TdP = torsade de pointes.

d. Pulseless ventricular tachycardia or brillation algorithm i. Debrillate and perform cardiopulmonary resuscitation. ii. Epinephrine 1 mg IV/IO (intravenously/intraosseous) every 35 minutes during cardiopulmonary resuscitation (evidence = IIb) or vasopressin 40 U IV/IO 1 dose to replace rst or second epinephrine dose during cardiopulmonary resuscitation (evidence = indeterminant). iii. Amiodarone 300 mg IV/IO 1 (LOE = IIb), additional 150 mg IV/IO 1 can be administered. (a) Consider if pulseless ventricular tachycardia/ventricular brillation persists after two or three shocks, cardiopulmonary resuscitation, and vasopressor therapy. (b) Give during cardiopulmonary resuscitation to facilitate circulation of drug before or after shock. iv. Lidocaine 11.5 mg/kg intravenous push (IVP) second line to amiodarone (class indeterminant); may repeat with 0.50.75 mg/kg IV/IO every 510 minutes (maximum 3 mg/kg) and continue with 14 mg/minute infusion if return of spontaneous circulation with lidocaine boluses. 4. Pulseless electrical activity or asystole algorithm a. Cardiopulmonary resuscitation (no debrillation). b. Epinephrine 1 mg IV/IO every 35 minutes or vasopressin 40 U IV/IO (1 dose to replace the rst or second epinephrine doses) (class IIb). c. Atropine 1 mg every 35 minutes 3 (total 0.04 mg/kg) if HR < 60 beats/minute for asystole or slow pulseless electrical activity (class indeterminant). D. Primary Prevention of Sudden Cardiac Death 1. ICD therapy is recommended to reduce total mortality due to sudden cardiac death in patients with LV dysfunction caused by prior MI (more than 40 days post-MI), have LVEF 30%40% or less, are NYHA class II or III, are receiving optimal chronic medications, and have reasonable survival expectation (class I, LOE = A). E. Secondary Prevention of Sudden Cardiac Death 1. ICD implantation is a reasonable treatment of recurrent sustained ventricular tachycardia in post-MI patients with normal or near-normal LVEF who are on optimal medications and have a reasonable survival expectation of more than 1 year (class IIa, LOE = C). 2. Contraindications: evolving acute MI with ventricular tachycardia, acute ventricular tachycardia after coronary artery bypass graft, ventricular brillation caused by atrial brillation, terminal illness, psychiatric disorder, severe NYHA class IV (nontransplantable) HF

2008 American College of Clinical Pharmacy 1-260

Acute Care Cardiology


Table 16. Alteration of Debrillation Threshold Medications Comments Amiodarone, lidocaine, and mexiletine Reprogram ICD, increased energy (joules) required Sotalol May decrease energy needed for DCC -blockers, class IA DCC = direct current cardioversion; ICD = internal cardioverter-debrillator. Threshold Alteration Increase threshold Decrease threshold No change

Patient Cases 4. C.D. is a 68-year-old man admitted after an episode of syncope with a presyncopal syndrome of seeing black spots and dizziness before passing out. Telemetry monitor shows sustained ventricular tachycardia, however this patient still has a pulse and his BP is 100/64. His medical history includes HF NYHA class III, LVEF 30%, MI 2, hypertension 20 years, left-ventricular hypertrophy, diabetes mellitus, and diabetic nephropathy. His current drugs include lisinopril 5 mg/day, furosemide 20 mg 2 times/day, metoprolol 25 mg 2 times/day, digoxin 0.125 mg/day, glyburide 5 mg/day, and aspirin 325 mg/day. BP 120/75 mm Hg, HR 80 beats/minute, BUN 30 mg/dL, SCr 2.2 mg/dL, 70 kg. Which one of the following is the best therapy to initiate for conversion of his sustained ventricular tachycardia? A. Amiodarone 150 mg IV for 10 minutes, followed by 1 mg/minute 6 hours and then 0.5 mg/minute. B. Sotalol 80 mg 2 times/day, titrated to QTc about 450 milliseconds. C. Dofetilide 500 mcg 2 times/day, titrated to QTc about 450 milliseconds. D. Procainamide 20 mg/minute, maximum 17 mg/kg. 5. C.D. presents to the emergency department 3 months after amiodarone maintenance initiation (he refused ICD placement) after a syncopal episode in which he lost consciousness for 30 seconds, according to witnesses. He also complains of rapid HR episodes where he feels dizzy and lightheaded. He feels very warm all the time (he wears shorts even though it is winter), is unable to sleep, and has experienced a 3-kg weight loss. He received a diagnosis of hyperthyroidism due to amiodarone therapy. On telemetry, he shows runs of nonsustained ventricular tachycardia. With an amiodarone half-life of 50 days, how long do you expect the effects of amiodarone to be provoking hyperthyroidism in this patient? A. Never. B. 1 month. C. 6 months. D. 1 year.

F. Special Patient Populations 1. Heart failure a. Dofetilideneutral effect on mortality 2. Acute MI a. Encainide, ecainide, moricizineincreases mortality when used for premature ventricular contraction control post-MI b. Class IA medicationsincrease mortality in post-MI survivors c. Dofetilideneutral effect on mortality in LV dysfunction post-MI III. ACUTE CORONARY SYNDROMES A. See Levels of Evidence and Classes of Recommendations from the Arrhythmias section. B. Unstable Angina and NonST-Elevation MI

2008 American College of Clinical Pharmacy 1-261

Acute Care Cardiology


Table 17. Unstable Angina and NonST-Elevation MI Denitions Acute angina at rest, typically prolonged > 20 minutes, ST-segment depression, T-wave inversion, or no ECG changes may occur, but no biomarkers for cardiac necrosis present NSTEMI Same as above, only with positive cardiac enzyme biomarkers of necrosis (troponin I or T elevation, CKMB fraction > 5%10% of total CK) CKMB = creatine kinase myocardial band; ECG = electrocardiogram; NSTEMI = nonST-elevation MI; UA = unstable angina. UA

1. Unstable angina/nonST-elevation MI (UA/NSTEMI) goals a. Prevent total occlusion of the infarct-related artery. i. Glycoprotein IIb/IIIa inhibitors plus unfractionated heparin or enoxaparin (preferred). ii. PCI can be either or both: (a) PCI = either percutaneous transluminal coronary angiography, (i.e., balloon); (b) Stent implantation. (1) Thrombolytics have shown no benet in NSTEMI or unstable angina and bleeding. b. Control chest pain and associated symptoms. 2. Initial Management a. MONA

Table 18. MONA Algorithm Morphine 15 mg IV is reasonable for patients whose symptoms are not relieved despite NTG or if they recur (class IIa, LOE N/A). O = Oxygen Oxygen if O2 saturation < 90% or high-risk features for hypoxemia (class I, LOE B). N = Nitroglycerin Nitroglycerin spray or SL tablet 0.4 mg 3 doses to relieve acute chest pain (if pain unrelieved after 1 dose, call 911) (class I, LOE C). Nitroglycerin IV 5 mcg/minute, titrate to chest pain relief or 200 mcg/minute if pain is unrelieved by morphine and sublingual NTG (class I, LOE C). Dose may be limited by hypotension. - Used during rst 48 hours for treatment of persistent chest pain, HF, and HTN A = Aspirin Aspirin chew and swallow nonenteric-coated 162325 mg 1 dose (class I, LOE A). HF = heart failure; HTN = hypertension; IV = intravenous; LOE = level of evidence; N/A = not available; NTG = nitroglycerin; SL = sublingual. M = Morphine

b. Diagnosis i. History of chest pain syndrome, ECG changes, serum biomarkers. 3. Early Hospital Anti-ischemic and Analgesic Therapy a. IV nitroglycerin i. Indicated in rst 48 hours for persistent ischemia, HF, or hypertension (class I, LOE C) ii. Use should not preclude other mortality-reducing therapies (-blocker or angiotensinconverting enzyme inhibitor [ACEI]) due to no mortality benet. iii. Dose: 10 mcg/minute, titrate to pain relief or maximum 200 mcg/minute. iv. Adverse effects: headache, hypotension, and tolerance. v. Contraindications: sildenal or vardenal use within 24 hours or tadalal use within

2008 American College of Clinical Pharmacy 1-262

Acute Care Cardiology 48 hours; systolic BP less than 100 mm Hg or more than 30 mm Hg below baseline, HR less than 50 beats/minute, HR less than 100 beats/minute in the absence of symptomatic HF or right ventricular infarction b. -Blockers i. Indicated within rst 24 hours if no contraindications (class I-PO/class IIaIV, LOE B) ii. First-line therapy for reducing chest pain, infarction size, and LV wall stress. iii. Oral (PO) route preferred. iv. Metoprolol 5 mg IV every 5 minutes 3 doses, then 2550 mg PO 2 times/day uptitrated as tolerated. v. Contraindication: signs of HF or low-output state, other risk factors for cardiogenic shock, or other relative contraindications (PR > 0.24 seconds or third-degree heart block, active asthma, or reactive airway disease) c. Angiotensin-Converting Enzyme Inhibitors i. Indicated PO within rst 24 hours if pulmonary congestion or LVEF 40% or less in absence of hypotension (class I, LOE A). ii. Contraindication (IV therapy): increased risk of hypotension with exception of patients with refractory hypertension. iii. Angiotensin receptor blocker indicated if contraindication to ACEI. d. Nondihydropyridine calcium channel blockersverapamil, diltiazem i. Recommended if continuing or frequently recurring ischemia and contraindication to -blocker therapy (LOE I, class B) or recurrent ischemia after -blockers and nitrates fully used (LOE IIa, class C). (a) No real benet or detriment on mortality; primarily symptom relief effects. ii. Contraindication: clinically signicant LV dysfunction, immediate release dihydropyridine calcium channel blockers should not be administered in the absence of a -blocker. 4. Early Hospital Antiplatelet Therapy a. Aspirin i. Indicated immediately after hospital presentation and continue indenitely (class I, LOE A). b. Clopidogrel i. Indicated if aspirin allergy or major gastrointestinal intolerance (class I, LOE A). ii. Loading dose (LD) 300 mg PO followed by daily oral maintenance dose 75 mg. iii. Higher oral LD of 600 or 900 mg more rapidly inhibits platelet aggregation with higher absolute level of inhibition, but additive clinical efcacy and safety has not been established.

2008 American College of Clinical Pharmacy 1-263

Acute Care Cardiology


Table 19. Antiplatelet and Anticoagulant Therapy Initial Conservative Strategy Antiplatelet Therapy ASA + CLO or IV GP IIb/IIIa inhibitor (upstream) ASA + CLO 1 month (class I, LOE A) (class I, LOE A) -OR- ASA + CLO plus IV GP IIb/IIIa inhibitor -OR- ASA + CLO ideally up to 1 year (upstream) (class I, LOE B) (class IIa, LOE B)a -OR- ASA + CLO + eptibatide or tiroban -OR- ASA + CLO > 300 mg given > 6 hours before cath (class IIb, LOE B) if bivalirudin used as anticoagulant (i.e., omit IV GP IIb/ IIIa inhibitor) (class IIa, LOE B) Anticoagulant Therapy Enoxaparin or UFH (class I, LOE A)d Enoxaparin or UFH (class I, LOE A)b,c -OR- Bivalirudin or fondaparinux (class I, LOE B) -OR- Fondaparinux (class I, LOE B) -OR- other LMWH such as dalteparin (limited evidence) a Factors favoring administration of both clopidorel and GP IIb/IIIa inhibitor: delay to angiography, high risk features, early recurrent ischemic discomfort. b LMWH is reasonable alternative to UFH in patients with UA/NSTEMI (class IIa, LOE B) or STEMI (class IIb, LOE B) in patient undergoing PCI. c If HIT, bivalrudin or argatroban preferred (Class I, LOE B). d Enoxaparin or fondaparinux are preferable to UFH, unless CABG is planned within 24 hours (Class IIa, LOE B). ASA = aspirin; cath = catheterization; CLO = clopidogrel; GP = glycoprotein; IV = intravenous; LOE = level of evidence. Table 20. Managing Anti-platelet and Anticoagulant Therapies Based on Various Scenarios If, after stress testing, - ASA (Class I, LOE A) patient classied as - CLO for 1 month (class I, LOE A), ideally up to 1 year (class I, LOE B) low risk - D/C IV GP IIb/IIIa inhibitor (class I, LOE A) - UFH for 48 hours -OR- enoxaparin or fondaparinux for hospital duration, up to 8 days (class I, LOE B) CABG post-catheter - ASA (Class I, LOE A) - D/C CLO 57 days before elective CABG (class I, LOE B) - D/C IV GP IIb/IIIa inhibitor 4 hours before CABG (class I, LOE C) - Continue UFH (class I, LOE B) -OR- discontinue enoxaparin 1224 hours, fondaparinux 24 hours, or bivalirudin 3 hours before CABG and initiate UFH (class I, LOE B) PCI post-cathetera - ASA (Class I, LOE A) - CLO LD if not started before diagnostic angiography (class I, LOE A) - IV GP IIb/IIIa inhibitor if not started before diagnostic angiography (class I, LOE A) - Discontinue anticoagulant after PCI if uncomplicated case (class I, LOE B) If no signicant CAD on cath Medical therapy - Antiplatelet and anticoagulant therapy at clinician discretion (class I, LOE C) postcatheter If CAD on cath - ASA (Class I, LOE A) - CLO LD if not started before diagnostic angiography (class I, LOE A) - Discontinue IV GP IIb/IIIa inhibitor (class I, LOE B) - If given before angiography, continue UFH for 48 hours or until discharge -ORenoxaparin or fondaparinux for hospital duration, for up to 8 days (class I, LOE B) - If given before angiography, either discontinue bivalirudin or continue at 0.25 mg/kg/ hour for up to 72 hours at clinicians discretion (class I, LOE B) Initial Invasive Strategy

2008 American College of Clinical Pharmacy 1-264

Acute Care Cardiology


May omit IV GP IIb/IIIa inhibitor upstream if bivalirudin used as anticoagulant and > 300 mg CLO > 6 hours before catheterization (class IIa, LOE B) or if troponin () and without high-risk features. ASA = aspirin; CABG = coronary artery bypass grafting; cath = catheterization; CLO = clopidogrel; D/C = discontinue; GP = glycoprotein; LD = loading dose; LOE = level of evidence; PCI = percutaneous coronary intervention; UFH = unfractionated heparin.
a

Table 21. Aspirin and Clopidogrel Aspirina,b Dosing Initial therapy - ASA 162325 mg nonenteric PO or chewed 1 (class I, LOE C) Pre-PCI - If taking long-term ASA, ASA 75-325 mg before PCI (class I, LOE A) - If not taking long-term ASA, ASA 300-325 mg at least 2 hours before PCI, preferably 24 hours (class I, LOE C) No stent - ASA 75162 mg/day indenitely (class I, LOE A) Post-stent - ASA 162325 mg/day for: at least 1 month (baremetal stent), at least 3 months (sirolimus-eluting stent), at least 6 months (paclitaxel-eluting stent) (class I, LOE B); then, 75162 mg/day indenitely (class I, LOE A) Contraindications Anaphylaxis, bronchospasm after therapy, serious Withhold for up to 57 days before bleeding elective CABG a If ASA contraindicated, CLO 75 mg/day long-term or 300 mg LD at least 6 hours prior to PCI +/- Glycoprotein IIb/ IIIa inhibitor (class I, LOE A). b If bleeding risk, reduce ASA dosage post-stent PCI 75162 mg/day (class IIa, LOE C). c Higher oral LD of 600 or 900 mg more rapidly inhibits platelet aggregation with higher absolute level of inhibition, but additive clinical efcacy and safety have not been established. ASA = aspirin; CABG = coronary artery bypass graft; CLO = clopidogrel; LOE = level of evidence; PO = orally. Table 22. Glycoprotein IIb/IIIa Inhibitors FDAapproved Indications PCI only UA/NSTEMI Dose PCI Dose 0.25 mg/kg IV bolus, then 0.125 mcg/kg/ minute (maximum 10 mcg/kg) for 12 hours 180 mcg/kg IV bolus 2 (10 minutes apart), then 2 mcg/kg/minute for 1824 hours postPCI Same as UA/NSTEMI dose, continue for 1824 hours post-PCI Renal Adjustment Not necessary; reticuloendothelial system clearance If CrCl < 50 mL/minute, reduce infusion 50%; IP dialysis, contraindicated Clopidogrelc (Plavix) Initial therapy - CLO 300 mg LD 1 (class I, LOE A) Pre-PCI - CLO 300 mg LD at least 6 hours before PCI (class I, LOE A) No stent - CLO 75 mg/day for at least 1 month (class I, LOE A) and ideally for up to 1 year (class I, LOE B) Post-stent - If bare-metal stent, CLO 75 mg/day for at least 1 month and ideally for up to 1 year (class I, LOE B) - If drug-eluting stent, CLO 75 mg/day for at least 1 year (class I, LOE B)

Abciximab Same as PCI dose if (ReoPro) cardiac catheterization GP receptor planned soon irreversible inhibitor Eptibatide UA/NSTEMI, 180 mcg/kg IV bolus, (Integrelin) elective PCI followed by 2 mcg/kg/ GP receptor minute competitive antagonist

Tiroban UA/NSTEMI 0.4 mcg/kg/minute If CrCl < 30 mL/minute, (Aggrastat) only for 30 minutes (LD reduce infusion 50% GP receptor infusion), followed by competitive antagonist 0.1 mcg/kg/minute Abciximab is preferred upstream agent if no appreciable delay to angiography and PCI is likely to be performed; otherwise, eptibatide or tiroban preferred.

2008 American College of Clinical Pharmacy 1-265

Acute Care Cardiology


CrCl = creatinine clearance; GP = glycoprotein; LD = loading dose; NSTEMI = nonST-elevation myocardial infarction; PCI = percutaneous coronary intervention; SCr = serum creatinine; UA = unstable angina. Table 23. Anticoagulants Unfractionated Heparin Classication Dosing 60 U/kg IV bolus, 12 U/kg/hour infusion Fondaparinux (Arixtra) Factor Xa inhibitor 1 mg/kg 120 IU/kg 2.5 mg subcutaneously bcutaneously subcutaneously twice daily twice daily once daily 58 days Enoxaparin (Lovenox) LMWH Dalteparin (Fragmin) LMWH Bivalirudin (Angiomax) DTI

Weight > 175 Signicant Contraindicated Adjust infusion if CrCl < 30 mL/ dose in severe renal renal or kg or SCr > minute dysfunction hepatic 2.5 not well (bolus dose same) dysfunction studied CrCl = creatinine clearance; DTI = direct thrombin inhibitor; FDA = U.S. Food and Drug Administration; HIT = heparin-induced thrombocytopenia; LMWH = low-molecular-weight heparin; PCI = percutaneous coronary intervention; NSTEMI = nonST-elevation MI; SCr = serum creatinine; UA = unstable angina.

Maximum 4000 U bolus and 1000 U/ hour infusion; titrate to goal activated partial thromboplastin time 1.52.5 control Contraindications Previous HIT, serious active bleeding

Comments

UA/NSTEMI: 0.1 mg/kg IV bolus, then 0.25 mg/kg/hour PCI: 0.50.75 mg/kg IV bolus, then 1.75 mg/kg/hour for 4 hours post-PCI 1 mg/kg Maximum Not FDA Useful in setting of subcutaneously dose 10,000 approved for HIT HIT IU twice once daily daily if CrCl < 30 mL/minute

5. Additional Long-term Medical Therapy a. -Blockers i. Indicated for all patients unless contraindicated (class I, LOE B). ii. Initiate within a few days of even, if not acutely, and continue indenitely. iii. If moderate or severe LV failure, initiate with gradual titration. b. ACE Inhibitors i. Indicated for all patients with HF, LVEF less than 40%, hypertension, or diabetes mellitus (class I, LOE A). ii. Also reasonable if no LV dysfunction, hypertension, or diabetes mellitus (class I, LOE A). iii. Angiotensin receptor blocker if ACEI intolerant and signs of HF and LVEF less than 40% (class I, LOE A). iv. May consider combination ACEI and angiotensin receptor blocker if persistent symptomatic HF and LVEF less than 40% despite standard therapy (class IIb, LOE B). c. Aldosterone Receptor Blockers i. Indicated if LVEF less than 40% and symptomatic HF or diabetes mellitus and receiving therapeutic doses of ACEI. (a) Contraindicated if CrCl less than 30 mL/minute or potassium more than 5 mEq/L

2008 American College of Clinical Pharmacy 1-266

Acute Care Cardiology


Patient Cases 6. J.D. is a 66-year-old, 70-kg woman with history of MI, hypertension, hyperlipidemia, and diabetes who presents with sudden-onset diaphoresis, nausea, vomiting, and dyspnea, followed by a bandlike upper chest pain (8/10) radiating to her left arm. She had felt well until 1 month ago, when she noticed her typical angina was occurring with less exertion. The ECG showed ST-depressions in leads II , III, and AVF, and hyperdynamic T waves. Cardiac enzymes are positive, and she receives a diagnosis of NSTEMI. Home medications: aspirin 81 mg/day, simvastatin 40 mg every night, metoprolol 50 mg 2 times/day, and metformin 1 g 2 times/day. Which one of the following is the best antiplatelet/anticoagulant strategy for this patient? A. Eptibatide 180 mcg/kg bolus 1, then 2 mcg/kg/minute plus unfractionated heparin titrated to 5070 seconds, aspirin 325 mg, and clopidogrel 300 mg 1 and then 75 mg/day plus cardiac catheterization for possible PCI. B. Aspirin 325 mg and enoxaparin 80 mg subcutaneously 2 times/day plus cardiac catheterization for possible PCI. C. Medical management with abciximab 0.25 mg/kg bolus, then 0.125 mg/kg/minute for 12 hours plus enoxaparin 80 mg subcutaneously 2 times/day, aspirin 325 mg/day, and clopidogrel 300 mg 1, then 75 mg/day. D. Medical management with aspirin 325 mg and clopidogrel 300 mg 1; then 75 mg/day plus unfractionated heparin 70 U/kg bolus, followed by 15 U/kg/hour. 7. J.D. received a percutaneous transluminal coronary angioplasty and paclitaxel-eluting stent in her right coronary artery. What is the optimal length of time clopidogrel therapy be continued? A. 1 month. B. 6 months. C. 12 months. D. Lifelong. Which one of the following is the optimal lifelong aspirin dose once dual therapy with clopidogrel after PCI with stent implantation is completed? A. 25 mg. B. 81 mg. C. 325 mg. D. 650 mg.

8.

d. Lipid Management i. Statins indicated with low-density lipoprotein goal less than 100 mg/dL (class I, LOE A) with a goal less than 70 mg/dL reasonable (class IIa, LOE A). ii. See Chronic Cardiology section for additional detail. C. ST-elevation MI 1. Denition a. Complete occlusion of epicardial coronary artery (red blood cells plus platelets, brin). b. Clinical chest pain syndrome lasting more than 20 minutes with good angina history, plus ST-elevation of more than 1 mm above baseline on ECG, plus positive cardiac enzyme biomarkers of necrosis (troponin I or T, creatine kinase, myocardial bound fraction more than 5%10% of total creatine kinase). 2. STEMI Goals a. Restore patency of the infarct-related artery and minimize infarct size. i. Thrombolytic medications or Door-to-Needle within 30 minutes. ii. PCI or Door-to-Balloon within 90 minutes. (a) Either percutaneous transluminal coronary angioplasty i.e., balloon, (b) Stent implantation, or both.

2008 American College of Clinical Pharmacy 1-267

Acute Care Cardiology b. Prevent complications such as arrhythmias or death. c. Control chest pain and associated symptoms. 3. Acute Symptom Relief a. MONA plus -blocker.

Table 24. MONA Plus -Blocker for AMI Morphine 15 mg IV is reasonable for patients whose symptoms are not relieved despite NTG or if they recur (class I, LOE C) O = Oxygen Oxygen 4 L/minute by nasal cannula if O2 saturation < 90% N = Nitroglycerin Nitroglycerin spray or SL tablet 0.4 mg 3 doses to relieve acute chest pain (if pain unrelieved after one dose, call 911) (class I, LOE C) Nitroglycerin IV 5 mcg/minute, titrate to chest pain relief or 200 mcg/minute if pain unrelieved by morphine and sublingual NTG (class I, LOE C) - No mortality benets but high placebo crossover rate. - Used in rst 48 hours for treatment of persistent chest pain, HF, and hypertension. A = Aspirin Aspirin chew and swallow nonenteric-coated 162325 mg 1 dose (class I, LOE A/C) -blocker Oral or IV -blocker (class I, LOE Aoral, class IIa, LOE BIV) - Mortality benet in early phases of acute MI (metoprolol 5.7% vs. placebo 8.9%). AMI = acute myocardial infarction; HF = heart failure; IV = intravenous; LOE = level of evidence; MI = myocardial infarction; NTG = nitroglycerin. M = Morphine

4. Reperfusion a. If presenting to a facility without the capability for expert, prompt intervention with primary PCI within 90 minutes of rst medical contact, patient should undergo brinolysis unless contraindicated (class I, LOE A). b. Facilitated PCI might be performed as a reperfusion strategy in high-risk patients when PCI is not immediately available and bleeding risk is low (class IIb, LOE B). i. Facilitated PCI refers to a strategy of planned immediate PCI after an initial pharmacological regimen such as full-dose brinolysis, half-dose brinolysis, a glycoprotein IIb/IIIa inhibitor, or a combination of reduced-dose brinolytic therapy and a platelet glycoprotein IIb/IIIa inhibitor. c. Rescue PCI or PCI following failed thrombolysis is indicated in select patients if the following develops: shock, severe HF and/or pulmonary edema, hemodynamic or electrical instability, evidence of persistent ischemia.
Table 25. Thrombolytic Therapy Alteplase (TPA) Reteplase (r-PA) Tenecteplase (TNKase) Streptokinase (Streptase) Dosing 15 mg IVP and then 0.75 mg/kg for 30 minutes (maximum 50 mg), followed by 0.5 mg/kg (maximum 35 mg) for 60 minutes 10 U IVP, repeat 10 U IVP in 30 minutes < 60 kg to 30 mg, 6069 kg to 35 mg, 7079 kg to 40 mg, 8089 kg to 45 mg, > 90 kg to 50 mg *about 0.5 mg/kg 1.5 MU IV for 60 minutes

Thrombolytic therapy is preferred within six hours of the onset of chest pain. UFH should be given to patients undergoing reperfusion with thrombolytic therapy. (Class I, B/C)

2008 American College of Clinical Pharmacy 1-268

Acute Care Cardiology


Table 26. Contraindications to Thrombolytic Therapy Relative Contraindications Absolute Contraindications ANY prior hemorrhagic stroke BP > 180/110 mm Hg on presentation Ischemic stroke within 3 months (except in past 3 History of TIA or CVA > 3 months prior hours) History of chronic poorly controlled HTN Intracranial neoplasm or arteriovenous INR 23 on warfarin malformation Recent trauma, major surgery, CPR, internal bleeding Active internal bleeding in 24 weeks Streptokinase exposure > 5 days prior or prior allergic Aortic dissection Signicant facial trauma or closed-head trauma in reaction (if given streptokinase again) past 3 months Active peptic ulcer Age > 75 years Pregnancy Known intracranial pathology (dementia) CPR = cardiopulmonary resuscitation; CVA = cerebrovascular accident; HTN = hypertension; INR = international normalized ratio; TIA = transient ischemic attack.

Patient Cases 9. R.V. is a 52-year-old man with a history of hypertension and hypertriglyceridemia who presents to a major university teaching hospital with a cardiac catheterization laboratory. He has 3 hours of crushing 10/10 substernal chest pain radiating to both arms that began while he sat eating his lunch, accompanied by nausea, diaphoresis, and shortness of breath. He has never experienced chest pain of this character or intensity before. He usually can walk several miles without difculty and is a 1.5 pack/day smoker. Home medications: lisinopril 2.5 mg/day, gembrozil 600 mg 2 times/day. Current vital signs: HR 68 beats/minute and BP 178/94 mm Hg; weight 100 kg. An ECG shows 3-mm ST-elevation in leads V2V4, I, and AVL. Serum chemistry values are within normal limits. First set of cardiac enzymes showed positive myoglobin levels, creatine kinase 175 U/L, myokinase 17.4 U/L, and troponin T 0.8 mcg/L (less than 0.1 mcg/L). Which one of the following should be done to treat this patients STEMI? A. Cardiac catheterization with primary PCI (stent) of occluded artery together with abciximab, clopidogrel, aspirin, and unfractionated heparin. B. Reteplase 10 U bolus twice, 30 minutes apart, plus unfractionated heparin 60 U/kg bolus and 12 U/kg/ hour infusion. C. Abciximab 0.25 mg/kg IVP and 0.125 mg/kg/minute for 12 hours plus enoxaparin 100 mg subcutaneously 2 times/day plus tenecteplase 25 mg IVP 1 time. D. Tiroban 0.04 mcg/kg/minute 30 minutes, followed by 0.01 mcg/kg/minute plus unfractionated heparin 60 U/kg bolus and 12 U/kg/hour infusion. 10. W.F. is a 70-year-old male smoker with a history of hypertension, benign prostatic hypertrophy, and lower back pain. Three weeks ago, he started experiencing substernal chest pain with exertion (together with dyspnea), which radiated to both arms and was associated with nausea and diaphoresis. Episodes have increased in frequency to 45 times/day and are relieved with rest. He has never had an ECG. Today, he awoke with 7/10 chest pain and went to the emergency department of a rural community hospital 2 hours later. He was acutely dyspneic and had continuing pain. Home medications: aspirin 81 mg/day for 2 months, doxazosin 2 mg/day, ibuprofen 800 mg 3 times/day. Vital signs include HR 42 beats/minute (bradycardic with complete heart block); BP 104/48 mm Hg; weight 61 kg. Laboratory results include BUN 45 mg/dL, SCr 1.7 mg/dL, creatine kinase 277 U/L, creatine kinase, myocardial bound 35.2 U/L, troponin T 1.5 mcg/L (less than 0.1 mcg/L). ECG showed a 3-mm ST-elevation. Aspirin, clopidogrel, and sublingual nitroglycerin were given in the emergency department. Which one of the following best describes how he should be managed? A. Alteplase plus enoxaparin. B. Unfractionated heparin. C. Tenecteplase plus unfractionated heparin. D. Diagnostic cardiac catheterization for possible primary PCI.

2008 American College of Clinical Pharmacy 1-269

Acute Care Cardiology 5. Mortality-reducing Medications a. Aspirin i. See dosing under UA/NSTEMI (class I, LOE A). b. -Blockers ii. Shown to reduce mortality in both early and late phases of STEMI. iii. See dosing under UA/NSTEMI. iv. Oral -blocker therapy should be administered promptly to those patients without a contraindication, irrespective of concomitant brinolytic therapy or performance of primary PCI (class I, LOE A). v. IV -blockers are reasonable for patients without contraindications, especially if a tachyarrhythmia or hypertension is present (class IIa, LOE B). vi. -blockers should be used cautiously or not at all in patients with symptoms of heart failure. c. Thrombolytics i. Mechanism of action: Plasminogen is a proenzyme and is converted to the active enzyme plasmin by plasminogen activators (endogenous or exogenous alteplase [TPA] derivatives); plasmin digests brin to soluble degradation products. ii. Administer within 12 hours of symptom onset (preferably within 6 hours, and ideally within 30 minutes of arrival to hospital). iii. Primary PCI preferred over thrombolytics in hospitals with cardiac catheterization laboratories. iv. Alteplase, reteplase, and tenecteplase require concomitant unfractionated heparin administration of 60 U/kg bolus (maximum 4000 U) and 12 U/kg/hour (maximum 1000 U/hour), adjusted for activated partial thromboplastin time of about 5070 seconds (class I, LOE C). (a) Low-molecular-weight heparins can be used in combination with thrombolytics if the patient is younger than 75 years without signicant renal dysfunction; SCr less than 2.5 mg/dL in men and less than 2 mg/dL in women (class IIb, LOE B). (1) Enoxaparin 30-mg IV bolus, followed by 1 mg/kg subcutaneously every 12 hours until discharge. (b) In patients with known heparin-induced thrombocytopenia, it is reasonable to consider bivalirudin a useful alternative to heparin to be used in conjunction with streptokinase (class IIa, LOE B). v. No added mortality benet at 30 days or 1 year when thrombolytics administered in half-dose, together with full-dose unfractionated heparin or enoxaparin and full-dose glycoprotein IIb/IIIa inhibitor. (a) Increased risk of major bleeding events by 3 times (i.e., intracranial hemorrhage), particularly in patients older than 75 years. (b) Can be used to prevent reinfarction if anterior MI in patients younger than 75 years with no risk of bleeding. d. ACE Inhibitors i. Give oral ACEIs in low doses to all patients during rst 24 hours of anterior STEMI, HF signs (pulmonary congestion), or LVEF less than 40%, provided no hypotension exists (systolic BP less than 100 mm Hg) or other contraindication, to reduce mortality and remodeling (class I, LOE A). ii. Consider oral ACEIs even if the above indications are not present (class IIa, LOE B)benet less (ve lives saved per 1000 treated) than if LV dysfunction is present.

2008 American College of Clinical Pharmacy 1-270

Acute Care Cardiology iii. Angiotensin receptor blocker if ACEI intolerant (class I, LOE C). iv. Administer after -blockers and sublingual nitroglycerin to avoid excess hypotension, but in rst 24 hours. v. Avoid IV ACEIs as it may cause hypotension (class III, LOE B). e. Statin Therapy i. Lipid panel should be drawn within the rst 24 hours of admission to prevent checking during acute-phase reaction (low-density lipoprotein falsely low for up to 2 months post-MI). ii. All patients should be discharged on statin therapy unless a contraindication exists. f. Insulin i. During the rst 2448 hours after STEMI, control hyperglycemia with insulin infusion (class I, LOE B). (c) After 48 hours, manage patients with diabetes with individualized regimens. D. Post-MI Medical Care and Secondary Prevention 1. Antiplatelet medications a. Aspirin 75162 mg/day (class I, LOE A). b. Clopidogrel 75 mg/day if PCI or medical management (see dosing under UA/ NSTEMI). 2. BP control a. See Joint National Committee VII recommendations in outpatient section for goals. b. -Blocker therapy (class I/class IIa if low risk, LOE A) i. Begin within a few days of the event, if not initiated acutely. ii. If moderate or severe LV failure, use gradual titration scheme. c. ACEI therapy (class I, LOE A) or angiotensin receptor blocker if ACEI intolerant (class I, LOE B) i. Should be initiated at discharge if not already prescribed. d. Aldosterone antagonist (class I, LOE A) i. Recommended after STEMI in patients with normal renal function (SCr 2.5 mg/dL or lower in men and 2 mg/dL or lower in women) if: (a) Already receiving therapeutic dose of ACEI. (b) LVEF 40% or less. (c) Symptomatic HF or diabetes. 3. Lipid-lowering therapy: statins a. Goal low-density lipoprotein less than 100 mg/dL (class I, LOE A), consider low-density lipoprotein less than 70 mg/dL in high-risk patients. b. Goal nonhigh-density lipoprotein-cholesterol* less than 130 mg/dL (class I, LOE B). i. After low-density lipoprotein-cholesterollowering therapy, consider adding brate or niacin (class IIa, LOE B). 4. Diabetes management a. Goal glycosylated hemoglobin less than 7%. 5. Smoking cessation in all patients 6. Weight management a. Goal body mass index 18.524.9 kg/m2; begin diet program if above goal weight. 7. Exercise a. Minimum goal 34 times/week.

2008 American College of Clinical Pharmacy 1-271

Acute Care Cardiology IV. PULMONARY ARTERIAL HYPERTENSION A. Denition and Treatment Goals 1. Pulmonary arterial hypertension a. Idiopathic PAH i. Change in nomenclature during 2003 World Conference on Pulmonary Hypertension; used to be known as primary pulmonary hypertension. ii. Familial PAH. b. Pulmonary artery hypertension i. Secondary causes (a) Scleroderma (most common), chronic thromboembolic disease, human immunodeciency virus disease, liver disease, connective tissue diseases, medications, toxins, and others. 2. Symptoms i. Dyspnea with exertion (60% of patients), fatigue, chest pain, syncope, and weakness (40%). (a) Caused by impaired oxygen delivery to tissues and diminished CO. ii. Orthopnea, peripheral edema, liver congestion, abdominal bloating, and other signs of right ventricular hypertrophy and failure occur when disease progresses to the heart. 3. Diagnosis
Table 27. Diagnostic Findings of PAH MPAP > 25 mm Hg with a PCWP < 15 mm Hg on PA catheterization with failed vasodilator challenge Electrocardiography Signs of RV hypertrophy, right-axis deviation, and anterior ST- and T-wave abnormalities consistent with RV strain pattern Echocardiography Estimated RV systolic pressure elevation, enlarged RV, RV dysfunction Chest radiography Enlarged pulmonary arteries and diminished peripheral pulmonary vascular markings, RV enlargement Physical examination Cool and/or cyanotic extremities, jugular venous distension, pulsatile hepatomegaly, peripheral edema, ascites MPAP = mean pulmonary artery pressure; PA = pulmonary artery; PCWP = pulmonary capillary wedge pressure; RV = right ventricle. Hemodynamic alterations

4. World Health Organization functional assessment classication


Table 28. World Health Organization Classication for PAH Class Class I Class II Class III Class IV Denition No symptoms (dyspnea, fatigue, syncope, chest pain) with normal daily activities Symptoms with strenuous normal daily activities that slightly limit functional status and activity level Symptoms of dyspnea, fatigue, syncope, and chest pain with normal daily activities that severely limit functional status and activity level Symptoms at rest; cannot conduct normal daily activities without symptoms

2008 American College of Clinical Pharmacy 1-272

Acute Care Cardiology 5. Guideline Recommendations for PAH


Table 29. Levels of Evidence and Grades of Recommendations for PAH Guidelines Levels of Evidence Good Fair Low Expert opinion Grades of Recommendation Grade A Grade B Grade C Grade D Grade I Grade E/A Grade E/B Grade E/C Well-designed randomized controlled clinical trials or meta-analyses Other controlled trials or randomized controlled trials with minor aws Non-randomized trials, case-control studies, or observational studies Consensus opinion of a panel of experts in the topic eld; no clinical trials meet the criteria for inclusion Strong recommendation Moderate recommendation Weak recommendation Negative recommendation No recommendation possible (inconclusive) Strong recommendation based on expert opinion Moderate recommendation based on expert opinion Weak recommendation based on expert opinion

6. Treatment goals a. Relieve acute dyspnea symptoms. b. Improve exercise capacity/quality of life and prevent death. i. Acute vasodilator response testing (evidence: fair, grade A for idiopathic PAH; evidence: expert opinion, grade E/C for other PAH causes). (a) Use IV epoprostenol, inhaled nitric oxide, or IV adenosine. (b) Positive response: reduction in mean pulmonary artery pressure by 1040 mm Hg. (1) Positive response predicts mortality reduction with long-term calcium channel blocker or vasodilator use. A. Treatment of PAH 1. Supportive care a. Supplemental oxygen to maintain O2 saturation more than 90% at all times (evidence: expert opinion, grade E/A). b. Loop diuretics if symptoms of peripheral edema or ascites. c. Warfarin anticoagulation: goal international normalized ratio 1.52.5 in patients with idiopathic PAH (evidence: fair, grade B for idiopathic PAH; evidence: expert opinion, grade E/C for other PAH). i. Prevents catheter thrombosis in patients on epoprostenol, clot formation caused by venous stasis, or right ventricular failure. d. Immunizations for inuenza and pneumococcus. 2. Calcium channel blockers a. First-line drug for all patients with PAH with positive acute vasodilator response (evidence: low, grade B for idiopathic PAH; evidence: expert opinion, grade E/B for other PAH causes). i. If cannot use or calcium channel blocker therapy fails, consider use of other vasodilatory medications as second-line alternative. b. Should not be used empirically without acute vasodilatory response testing or without a positive response to testing (evidence: expert opinion, grade E/A). i. Diltiazem, amlodipine, and nifedipine are most commonly used. 2008 American College of Clinical Pharmacy 1-273

Acute Care Cardiology (a) Choose on the basis of HR at baseline. (1) If relatively bradycardic, choose amlodipine or nifedipine. (2) If relatively tachycardic, choose diltiazem. c. Functional class II patients who are not candidates or for whom calcium channel blocker treatment failed may benet from other therapies, but no specic drugs are recommended because of a lack of evidence (evidence: expert opinion, grade: E/B). Bosentan a. Mechanism of action: endothelin-1 types A (ET-A) and B (ET-B) receptor antagonist i. ET-A produces vasoconstriction and increases vascular smooth muscle. ii. ET-B clears endothelin-1 from vasculature in kidneys and lungs; produces vasodilation. b. Treatment effect: improves cardiac index, reduces mean pulmonary artery pressure and pulmonary capillary wedge pressure as well as mean right atrial pressure, and improves functional class. c. Place in therapy: i. First-line therapy for patients with PAH in functional class III who cannot take calcium channel blockers (evidence: good; grade A). ii. Alternative option in patients in functional class IV who cannot take calcium channel blockers or IV epoprostenol (evidence: fair, grade B). Epoprostenol a. Mechanism of action: prostacyclin analogue i. Potent vasodilator of pulmonary and systemic vessels. b. Treatment effect: improved survival by 35 years, increased 6-minute walk time and distance, increased quality of life and cardiac index, and symptomatic improvement. c. Place in therapy: i. First-line therapy for patients with PAH in functional class IV who experience failure with or cannot take calcium channel blockers (evidence: good, grade A). ii. Second-line therapy for patients with PAH in functional class III who experience failure with or cannot take calcium channel blockers or rst-line vasodilators (bosentan) (evidence: good, grade A). d. Contraindications: inability to keep drug refrigerated before and during infusion; requires ice packs to keep stable; unable to reconstitute drug in sterile environment. e. Other considerations: costs $100,000/year or more; if infusion is abruptly discontinued, because of the short half-life (t1/2 less than 6 minutes); acute symptomatic decompensation and possibly death may occur if medical attention is not sought immediately. Subcutaneous treprostinil a. Mechanism of action: prostacyclin analogue: potent vasodilator of pulmonary and systemic vessels. b. Treatment effect: dose-related symptomatic improvement and effect on exercise tolerance; more effective in functional class III and IV patients but is an alternative therapy (minimal/no effect in functional class II patients or patients with congenital heart disease) (evidence: fair, grade B). c. Other considerations: If subcutaneous infusion is abruptly discontinued because of the longer half-life (t1/2 = 3 hours), patients have more time to seek medical attention than with epoprostenol. i. Premixed, prelled syringe is easier to administer than epoprostenol. Inhaled iloprost a. Mechanism of action: potent pulmonary vasodilator. 2008 American College of Clinical Pharmacy 1-274

3.

4.

5.

6.

Acute Care Cardiology b. Treatment effect: symptomatic improvement of functional class, quality of life for functional classes III or IV but is an alternative therapy. i. Functional class III patients (evidence: fair, grade B). ii. Functional class IV patients (evidence: low, grade C). c. Other considerations: must be used with Prodose AAD nebulization system. 7. Sildenal a. Mechanism of action: sildenal selectively inhibits phosphodiesterase-5 in the lungs to increase and prolong pulmonary vasodilation in response to nitric oxide. b. Treatment effect: reduces pulmonary artery wedge pressure, increases cardiac index, decreases pulmonary vascular resistance, and improves NYHA functional class, symptoms, and distance walked on 6-minute walk test. i. Alternative option in any patient with PAH who is not a candidate for any other drug therapy (evidence: low, grade C) B. Treatment Options
Table 30. Overview of PAH Treatment Options Drug Dose Indications Epoprostenol 140 ng/kg/minute First line: chronic (Flolan) therapy in PAH functional class IV; alternative in functional class III Considerations Continuous IV infusion by a pump Unstable at room temperature and acidic pH Drug requires reconstitution Medical emergency if infusion interrupted (spare drug cassette and infusion pump should be available) Alternative in Severe erythema and Local treatments (hot Treprostinil 1.2540 ng/ functional class III induration (83%) and and cold packs or topical (Remodulin) kg/minute subcutaneous patients whose calcium injection site pain (85%) analgesics) can be used infusion channel blocker limit use; also headache, to minimize infusion site discomfort treatment failed and nausea, diarrhea, rash Moving infusion site every ET-1 antagonists 3 days minimizes irritation or class IV failing epoprostenol Mild, transient cough, Requires 69 inhalations Inhaled 2.5 1, followed Alternative in functional class III ushing, headache, daily (15 minutes each with iloprost by 5 mcg/ syncope jet nebulizer) (Ventavis) inhalation through in patients whose calcium channel Inhalation form has fewer a nebulizer 69 times/day while blocker treatment systemic adverse reactions awake failed and endothelin-1 than other prostacyclin antagonists or class IV analogues failing epoprostenol Use no more often than every 2 hours Adverse Effects Jaw pain, nausea, vomiting, ushing, headache, muscle aches and pain, catheterrelated thrombosis, IV line infections; rebound worsening of symptoms if abruptly discontinued

2008 American College of Clinical Pharmacy 1-275

Acute Care Cardiology


Drug Bosentan (Tracleer) Dose Indications 62.5125 mg PO 2 First-line therapy times/day for functional class III in patients whose calcium channel blocker treatment failed; alternative in class IV patients whose epoprostenol therapy failed Adverse Effects Hypotension, doserelated increased liver enzymes (up to 14%), possible male infertility, syncope, ushing Considerations Severe drug interactions with glyburide (increased LFTs) and cyclosporine (decreased efcacy of both cyclosporine and bosentan) Monitor LFTs monthly Efcacy decreased with because of risk of CYP2C8/9 and 3A4 hepatotoxicity inducers and toxicity Anemia risk requires Hgb/ increased with 3A4 and Hct monitoring periodically CYP2C8/9 inhibitors Potential teratogen; if child-bearing age, use two contraceptive methods (reduced efcacy of hormonal contraceptives); monthly pregnancy test required May augment effects of Headache, epistaxis, other vasodilators when facial ushing, bluish used in combination or blurry vision, light sensitivity, dyspepsia, (especially prostacyclin) insomnia Contraindicated in patients receiving nitrates

Indicated in WHO functional class I patients to improve exercise ability or for PAH patients for whom all other therapies have failed CYP = cytochrome P450; ET-1 antagonists = endothelin-1 antagonists; Hct = hematocrit; Hgb = hemoglobin; IV = intravenous; LFTs = liver function tests; PAH = pulmonary arterial hypertension; PO = orally; WHO = World Health Organization. Patient Case 11. R.W. is a 38-year-old obese woman who presents with increasing symptoms of fatigue and shortness of breath. She could walk only 1020 feet at baseline and is now short of breath at rest. Her arterial blood gas is pH 7.31/pCO2 65/pO2 53/85% O2 saturation. She has three-pillow orthopnea and 3+ pitting edema in her lower extremities. Medical history is signicant only for atrial brillation. On computed tomographic angiography, the pulmonary artery trunk was signicantly enlarged and had a mean pressure of 56 mm Hg. ECHO: right atrial and ventricular hypertrophy. Chest radiograph showed prominent interstitial markings. Pertinent laboratory ndings: BUN 21 mg/dL, SCr 1.2 mg/dL, aspartate aminotransferase 145 IU/L, alanine aminotransferase 90 IU/L, international normalized ratio 2.1, partial thromboplastin time 52 seconds. Vital signs: BP 108/62 mm Hg, HR 62 beats/minute. Home medications: warfarin 2.5 mg/day, ipratropium 2 puffs every 6 hours, salmeterol 2 puffs 2 times/day, diltiazem controlled delivery 480 mg/day. Her diagnosis is idiopathic PAH. Given the options below, which one is the best evidence-based management strategy? A. Increase diltiazem to 600 mg/day. B. Start sildenal 20 mg 3 times/day. C. Start epoprostenol 2 ng/kg/minute. D. Start bosentan 62.5 mg 2 times/day.

Sildenal (Revatio)

20 mg PO 3 times/day xed dose

2008 American College of Clinical Pharmacy 1-276

Acute Care Cardiology

V. HYPERTENSIVE CRISES (URGENCY AND EMERGENCY) A. Denitions 1. Hypertensive urgency a. Acutely elevated BP, particularly diastolic pressure more than 120130 mm Hg without evidence of target organ damage. 2. Hypertensive emergency a. Hypertension with evidence of target organ damage. i. Target organ damage: brain, heart, kidneys, and eyes. (a) Hypertensive encephalopathy, intracranial hemorrhage, or other acute neurologic decit, UA or acute MI, acute HF, pulmonary edema (shortness of breath), aortic dissection, retinopathy or papilledema, decreased urine output or acute renal failure, eclampsia. B. Goals 1. Urgency a. Lower mean arterial pressure to goal or near goal within 24 hours; oral medications can be used. 2. Emergency a. Lower mean arterial pressure by 25% or diastolic pressure to 100110 mm Hg within 3060 minutes. C. Treatment Options
Table 31. Recommended Antihypertensive Agents for Hypertensive Crises Preferred Antihypertensive Agents Nicardipine, fenoldopam, or NTP + NTG and a loop diuretic Esmolol, metoprolol, labetalol, or verapamil + low-dose NTG and a loop diuretic Acute myocardial ischemia Labetalol or esmolol + NTG Hypertensive encephalopathy Nicardipine, labetalol, or fenoldopam Acute aortic dissection Labetalol, nicardipine + esmolol, NTP + esmolol or IV metoprolol Pre-eclampsia, eclampsia Labetalol or nicardipine Acute renal failure/microangiopathic anemia Nicardipine or fenoldopam Sympathetic crisis/cocaine overdose Veramapmil, diltiazem, or nicardipine + a benzodiazepine Acute postoperative hypertension Esmolol, nicardipine, or labetalol Acute ischemic stroke/intracerebral bleed Nicardipine, labetalol, or fenoldopam IV = intravenous; NTG = nitroglycerin; NTP = nitroprusside. Reprinted with permission from Marik PE, Varon J. Hypertensive crises: challenges and management. Chest 2007; 131:194962. Conditions Acute pulmonary edema/systolic dysfunction Acute pulmonary edema/diastolic dysfunction

2008 American College of Clinical Pharmacy 1-277

Acute Care Cardiology

Table 32. Dosage and Adverse Effects of Commonly Used Parenteral Antihypertensive Drugs Drug Sodium nitroprusside (Nipride) Esmolol (Brevibloc) Dosage Adverse Effects 0.250.5 mcg/kg/minute, Cyanide/thiocyanate toxicity, nausea maximum 3 mcg/kg/minute CI: renal, hepatic failure 500 mcg/kg LD for 1 minute, Bronchospasm, especially in patients with asthma, followed by 2550 mcg/kg/ HF exacerbation, bradycardia/heart block minute; maximum of 300 mcg/kg/minute Labetalol 2080 mg IV every 15 minutes Bronchospasm, especially in patients with asthma, HF exacerbation, bradycardia/heart block (Normodyne, Trandate) OR 0.52 mg/minute with maximum 24-hour dose of 300 mg Nicardipine 515 mg/hour, Reex tachycardia, nausea, vomiting (Cardene) maximum 15 mg/hour Nitroglycerin 510 mcg/minute, Headache, nausea, tachyphylaxis maximum 100 mcg/minute Hydralazine 510 mg IV every Reex tachycardia, headache, variable duration of (Apresoline) 46 hours action (not to exceed 20 mg/dose) Enalaprilat 0.6251.25 mg IV every 46 Renal insufciency/failure, variable response based on (Vasotec) hours, renin state maximum 5 mg every 6 hours Fenoldopam 1 mcg/kg/minute, Hypotension; cerebral ischemia, headache (Corlopam) maximum 1.6 mcg/kg/minute CI = contraindications; HF = heart failure; IV = intravenously; LD = loading dose.

Patient Case 12. A.W. is a 68-year-old man with a history of end-stage renal disease on hemodialysis, hypertension, coronary artery disease status post-MI, moderately depressed LVEF, and gastroesophageal reex disease who presents with acute-onset shortness of breath and chest pain. After his recent dialysis, he had a large barbecue meal with salt and smoked some marijuana laced with cocaine. He was medication noncompliant for 2 days and noticed he had gained 2 kg during 24 hours. His baseline orthopnea worsened to sleeping while sitting up in a chair for the two nights before admission. He developed acute-onset chest tightness with diaphoresis and nausea, pain 7/10. He went to the emergency department, where a BP of 250/120 mm Hg was noted. He had crackles halfway up his lungs on examination, and a chest radiograph showed bilateral uffy inltrates with prominent vessel cephalization. An electrocardiogram showed sinus tachycardia, HR 122 beats/minute, STdepressions in leads II and III, and AVF. He was admitted for hypertensive emergency. Laboratory results: BUN 48 mg/dL, SCr 11.4 mg/dL, B-type natriuretic peptide 2350 pg/mL, troponin T 1.5 mcg/L (less than 0.1 mcg/L), creatinine kinase 227 U/L, and myokinase 22 U/L. Which one of the following is the best medication to manage A.W.s hypertensive emergency? A. IV nitroglycerin 5 mcg/minute, titrated to 25% reduction in mean arterial pressure. B. Labetalol 2 mcg/minute, titrated to 50% reduction in mean arterial pressure. C. Sodium nitroprusside 0.25 mcg/kg/minute, titrated to 25% reduction in mean arterial pressure. D. Clonidine 0.1 mg PO every 2 hours as needed, 50% reduction in mean arterial pressure.

2008 American College of Clinical Pharmacy 1-278

Acute Care Cardiology REFERENCES


Decompensated HF and Cardiogenic Shock 1. Adams KF, Lindenfeld J, Arnold JMO, et al. Executive summary: HFSA 2006 comprehensive heart failure practice guideline. J Card Fail 2006;12:1038. 2. Nohria A, Lewis E, Warner Stevenson LW. Medical management of advanced heart failure. JAMA 2002; 87:62840. Acute Dysrhythmias 1. Fuster V, Ryden LE, Cannom DS, et al. ACC/ AHA/ESC 2006 guidelines for the management of patients with atrial brillationexecutive summary: a report of the American College of Cardiology/American Heart Association Task Force on Practice Guidelines and the European Society of Cardiology Committee for Practice Guidelines (Writing Committee to Revise the 2001 Guidelines for the Management of Patients with Atrial Fibrillation). Circulation 2006;114:70052. 2. Zipes DP, Camm AJ, Borggrefe M, et al. ACC/ AHA/ESC 2006 guidelines for the management of patients with ventricular arrhythmias and the prevention of sudden cardiac deathexecutive summary: a report of the American College of Cardiology/American Heart Association Task Force and the European Society of Cardiology Committee for Practice Guidelines (Writing Committee to Develop Guidelines for Management of Patients with Ventricular Arrhythmias and the Prevention of Sudden Cardiac Death). Circulation 2006;114:1088132. 3. Blomstrm-Lundqvist C, Scheinman MM, Aliot EM, et al. ACC/AHA/ESC 2003 Guidelines for the management of patients with supraventricular arrhythmiasexecutive summary. A Report of the American College of Cardiology/ American Heart Association Task Force on Practice Guidelines and the European Society of Cardiology Committee for Practice Guidelines (Writing Committee to Develop Guidelines for the Management of Patients with Supraventricular Arrhythmias). JACC 2003;42:1493531. 4. Gregoratos G, Abrams J, Epstein AE, et al. ACC/AHA/NASPE 2002 guideline update for implantation of cardiac pacemakers and antiarrhythmia devices: a report of the American College of Cardiology/American Heart Association Task Force on Practice Guidelines (ACC/AHA/NASPE Committee on Pacemaker Implantation). 2002. 5. American Heart Association. 2005 guidelines for cardiopulmonary resuscitation and emergency cardiovascular care. Circulation 2005;112:1211.

Acute Coronary Syndromes 1. Antman EM, Anbe DT, Armstrong PW, et al. ACC/AHA guidelines for the management of patients with ST-elevation myocardial infarction: a report of the American College of Cardiology/ American Heart Association Task Force on Practice Guidelines (Committee to Revise the 1999 Guidelines for the Management of Patients with Acute Myocardial Infarction). Circulation 2004;110:588636. 3. Eagle KA, Guyton RA, Davidoff R, et al. ACC/ AHA 2004 guideline update for coronary artery bypass graft surgery: summary article: a report of the American College of Cardiology/American Heart Association Task Force on Clinical Practice Guidelines (Committee to Update the 1999 Guidelines on Coronary Artery Bypass Graft Surgery). Circulation 2004;110:116876. 4. Anderson JL, Adams CD, Antman EM, et al. ACC/AHA 2007 guideline update for the management of patients with unstable angina and nonST-segment elevation myocardial infarction: summary article: a report of the American College of Cardiology/American Heart Association Task Force on Practice Guidelines (Committee on the Management of Patients with Unstable Angina). Circulation 2007;50:e1157. 6. Smith SC Jr, Feldman TE, Hirshfeld JW Jr, et al. ACC/AHA/SCAI 2005 guideline update for percutaneous coronary intervention: a report of the American College of Cardiology/American Heart Association Task Force on Practice Guidelines (ACC/AHA/SCAI Writing Committee to Update the 2001 Guidelines for Percutaneous Coronary Intervention). J Am Coll Cardiol 2006;47:e1121. Pulmonary Arterial Hypertension 1. Badesch DB, Abman SH, Ahearn S, et al. Medical therapy for pulmonary arterial hypertension: ACCP evidence-based clinical practice guidelines. Chest 2004;126:35S62S. 2. Badesch DB, Abman SH, Simonneau G, et al. Medical therapy for pulmonary arterial hypertension: updated ACCP evidencebased clinical practice guidelines. Chest 2007;131:191728.

2008 American College of Clinical Pharmacy 1-279

Acute Care Cardiology


3. 4. McLaughlin VV, McGoon MD. Pulmonary arterial hypertension. Circulation 2006;114:141731. McGoon M, Gutterman D, Steen V, et al. Screening, early detection, and diagnosis of pulmonary arterial hypertension: ACCP evidence-based clinical practice guidelines. Chest 2004;126:14S34S.

Hypertensive Emergency 1. Haas AR, Marik PE. Current diagnosis and management of hypertensive emergency. Semin Dial 2006;19:50212. 2. Marik PE, Varon J. Hypertensive crises: challenges and management. Chest 2007;131:194962.

2008 American College of Clinical Pharmacy 1-280

Acute Care Cardiology ANSWERS AND EXPLANATIONS TO PATIENT CASES


1. Answer: C This patient has ADHF and is receiving a -blocker. Although long-term -blockers can improve HF symptoms and reduce mortality, in the short term, -blockers can worsen symptoms. It is recommended to maintain the maintenance -blocker therapy at the same or slightly reduced dose compared with outpatient therapy in patients with ADHF, and increasing the dose may acutely worsen symptoms and CO. In patients who are admitted with volume overload without signicant signs of decreased CO, it is reasonable to rst try IV loop diuretics. As gastrointestinal edema increases, oral loop diuretics (notably furosemide) become less effective because of decreased absorption. Nesiritide is a vasodilatory medication that can be initiated if IV loop diuretic therapy fails, but because of its adverse effects and signicant cost, it is not recommended before a trial of IV diuretics. Milrinone is an inotropic medication. Due to their adverse effects Inotropes are recommended in cold/wet exacerbations after vasodilatory medications have failed. 2. Answer: A Intravenous vasoldilators are reasonable options if IV diuretics fail and the patient needs further diuresis and afterload reduction. Although nesiritide is linked with the potential for worsening SCr in a meta-analysis, its use is not contraindicated in patients with pre-existing renal insufciency. This patient is demonstrating signs of a warm and wet exacerbation and has experienced failure with IV loop diuretics and the addition of the thiazide diuretic metolazone, so the addition of a vasodilatory medication such as nesiritide is the most appropriate option in this patient at this time. Sodium nitroprusside can lead to thiocyanate toxicity in patients with severe renal insufciency, so it is contraindicated in this case. Dobutamine is typically used in states of low CO decompensation and is counteracted by concomitant -blocker therapy, making it a poor choice in patients receiving -blockers. Although milrinone is a more acceptable inotropic medication in a patient receiving -blockers, the dosing strategy in this case is not appropriate for the degree of renal insufciency this patient exhibits. Because of its renal clearance, milrinone must be dose adjusted if creatinine clearance is less than 50 mL/ minute. 3. Answer: A Signs of a decreased CO state in HF, such as increased SCr, decreased mental status, and cool extremities, indicate that a cold and wet state exists and that adjunctive therapy is needed. Positive inotropic agents, such as milrinone, will increase cardiac output to maintain perfusion to vital organs. Milrinone will also vasodilate the peripheral vessels to unload the heart ( systemic vascular resistance). The milrinone dose has been adjusted to accommodate the patients degree of renal insufciency. Again, although dobutamine would be a potential choice in this patient, it is not recommended in patients receiving -blockers. Lowdose nitroglycerin will cause venous dilation; however, this patient would benet from arterial dilation as well. Only higher doses of nitroglycerin will cause arterial dilation. Phenylephrine has no positive effects, so it will not augment contractility. In addition, it will cause vasoconstriction through -stimulation, which will further increase systemic vascular resistance and likely worsen CO. Vasoconstrictors are reserved for patients in cardiogenic shock. Although this patient does exhibit signs of signicant hypoperfusion, the BP remains preserved. 4. Answer: A Treatment options for sustained ventricular tachycardia are dependent on concomitant disease states. In a patient with left-ventricular dysfunction, class I agents such as procainamide are contraindicated. In a patient with creatinine clearance less than 60 mL/minute, sotalol requires signicant dosage reduction to avoid excess torsade de pointes. Sotalol is not an effective cardioversion medication and is more useful for preventing future episodes of arrhythmias (maintaining sinus rhythm) once sinus rhythm is achieved. Dofetilide is indicated only for atrial brillation, not for ventricular arrhythmias, and similarly, cardioversion rates with dofetilide are low. Amiodarone is rst-line therapy for sustained ventricular tachycardia in patients with severe renal insufciency, HF, and structural heart disease. 5. Answer: C With the prolonged half-life of amiodarone and extensive fat tissue volume of distribution, it would be expected for hyperthyroid adverse effects to last for at least 35 half-lives of the drug, which is anywhere from 5 to 8 months. Although therapeutic levels may fall off substantially by then, 1 month is too soon to expect the effects to subside. Even though some iodine and amiodarone molecules will remain absorbed in fat stores likely for years, if not for life, therapeutic levels should not exist longer than what is predicted by half-life. 6. Answer: A In this patient, the presence of ST-depressions on ECG, positive biomarkers for myocardial necrosis, at least three risk factors for coronary artery disease,

2008 American College of Clinical Pharmacy 1-281

Acute Care Cardiology


and a prior history of coronary artery disease suggests high risk. (prior MI) In high-risk patients, cardiac catheterization is used to determine whether occluded or partially occluded epicardial arteries exist, which can be intervened on, and to make an intervention (stent or percutaneous transluminal coronary angiography). Glycoprotein IIb/IIIa inhibitors in combination with heparin (either unfractionated heparin or enoxaparin), aspirin, and clopidogrel have the best outcomes in the early invasive strategy. Low-molecular-weight heparins are now preferred over unfractionated heparin for managing nonST-elevation acute coronary syndromes, but medical management alone is not optimal in such a high-risk patient. Abciximab was benecial only in clinical trials of primary PCI or PCI during the abciximab infusion (early invasive strategy). Abciximab was not superior to placebo when used in a conservative medical management strategy without PCI. Unfractionated heparin alone with clopidogrel has been studied in medically managed patients not pursuing catheterization (CURE trial) but was studied in a low-risk patient population. Metformin should be withheld for 24 hours prior to PCI (esprcially in those with renal dysfunction) to prevent lactic acidosis. 7. Answer: C Clopidogrel has been studied most commonly for a 30-day post-stenting procedure to prevent acute reocclusion of coronary vessels. Because the stent is not endothelialized for a longer time after drugeluting stent placement compared with traditional stents, a clopidogrel duration of at least 3 months is desired to prevent acute stent thrombosis (sirolimuseluting stent). However, clopidogrel is recommended for use in combination with aspirin for at least 6 months after paclitaxel drug-eluting stent placement to prevent risk of acute stent thrombosis, and current guidelines recommend therapy for at least one year. Trials using 912 months of clopidogrel primarily used a medical management strategy (conservative) with unfractionated heparin, clopidogrel, and aspirin and enrolled mostly unstable angina patients, or they were elective PCI trials with low glycoprotein IIb/IIIa inhibitor use. Lifelong therapy is not warranted by any published trial to date. 8. Answer: B Doses of aspirin lower than 75 mg/day (e.g., 25 mg) have not been proven to be as efcacious as higher doses of aspirin after combination therapy with clopidogrel after a PCI procedure. Aspirin 325 mg should be given to all patients after a PCI procedure with stent implantation throughout the recommended duration of clopidogrel therapy (1 month for bare metal stents, 3 months for sirolimus-eluting stents, and 6 months for paclitaxel-eluting stents). Once the recommended duration of dual therapy is completed, patients should receive a reduced dose of 75162 mg/day to prevent gastrointestinal and bleeding complications, and 81 mg is within this range of doses (class I, LOE B). There is no evidence that a higher dose of aspirin (650 mg) has any benet over lower doses of aspirin, and it has a higher risk of adverse effects, so it is not recommended. 9. Answer: A Although this patient presented within 3 hours of chest pain onset and is a thrombolytic candidate (within less than 6 hours of onset is preferred), up to 95% of patients can achieve normal, brisk TIMI-3 ow rates with primary PCI, versus only 50%60% of patients achieving normal TIMI-3 coronary ow with thrombolytic therapy. Because he is in a hospital that can perform a primary PCI with stent implantation, this is the therapy of choice. Although glycoprotein IIb/IIIa inhibitors have been studied in combination with thrombolytics and anticoagulants for STEMI, the slight increase in TIMI-3 blood ow rates was accompanied by a signicantly increased risk of bleeding; thus, they are not recommended. Tiroban and unfractionated heparin alone are not indicated for treatment of ST-elevation MI (recommended for medical management of nonST-elevation acute coronary syndrome only). 10. Answer: C Unlike the patient in case 9, this patient presented with an ST-elevation MI to a rural community hospital. He presents within the window for thrombolytic therapy consideration (less than 6 hours after chest pain onset). He is experiencing complete heart block and bradycardia, which could indicate an occlusion above the area perfusing his SA and/or AV nodes. Because he is still having ischemic chest pain and STsegment elevation, he should benet from reperfusion therapy. Enoxaparin is a treatment option for medical management, but the patient is at higher risk of bleeding from impaired enoxaparin clearance and will require dosage adjustment. Simply managing this patient conservatively with unfractionated heparin alone in the setting of ongoing chest pain, shortness of breath, and pulmonary edema is not an optimal choice. Diagnostic catheterization and possible PCI to determine whether an artery can be reperfused is not desirable because of the patients elevated SCr (creatinine clearance 35 mL/minute). Because of the shorter half-life and ease of administration of tenecteplase, it is preferable to alteplase. Clearance of unfractionated heparin is not as altered as enoxaparin and would be a more appropriate therapy than enoxaparin in combination with a thrombolytic.

2008 American College of Clinical Pharmacy 1-282

Acute Care Cardiology


11. Answer: C This patient is already receiving calcium channel blocker therapy to control her HR caused by atrial brillation. She is on a signicant dose of diltiazem, and her HR likely would not tolerate further increases in therapy. Sildenal is indicated for functional class I patients or patients who have failed all other therapies. Although bosentan is an attractive oral option to manage her PAH, her liver enzymes are elevated more than 3 times the upper limit of normal. In this setting, administering bosentan is not recommended. If liver transaminases are elevated transiently because of hepatic congestion, bosentan may be reconsidered later. Because this patient is currently in functional class IV with symptoms at rest, epoprostenol is indicated for a survival benet. 12. Answer: A Hypertensive emergency should be immediately treated by a 25% reduction in mean arterial pressure, followed by slow reduction to goal for 57 days. This patients comorbidities guide which therapy is optimal. His dialysis and SCr of 11.4 mg/dL are a contraindication to sodium nitroprusside because of possible cyanide toxicity. Labetalol (-blockers in general) therapy is controversial for patients who have taken cocaine, but its nonselective nature makes it an option; however, a reduction of 50% initially is too rapid a decrease in BP for safety. Clonidine is not an appropriate medication for hypertensive emergency, because its unpredictable oral nature is difcult to titrate and can lead to precipitous drops in BP beyond the goal 25% reduction and possible stroke or worsening MI. Nitroglycerin is an optimal choice, considering the patients lack of contraindications to this therapy and his evolving MI.

2008 American College of Clinical Pharmacy 1-283

Acute Care Cardiology ANSWERS AND EXPLANATIONS TO SELF-ASSESSMENT QUESTIONS


1. Answer: C Bosentan is an inducer of cytochrome P450 3A4 and 2C9 isoenzymes. Bosentan has been shown to decrease the plasma concentrations of all hormonal contraceptive medications, including both estrogenand progesterone-containing formulations, because of its effects on cytochrome P450 metabolism. No hormonal contraceptive, including oral, injectable, topical (patch), and implantable formulations, should be used as the only means of contraception because these may not be effective in preventing pregnancy in patients on bosentan. Use of a double-barrier method with a condom and diaphragm plus spermicide is indicated in patients receiving bosentan and hormonal contraceptives. Because bosentan is also a known teratogen, a barrier method alone may not be a sufcient form of contraception. 2. Answer: C Patients with dietary and/or drug noncompliance most commonly present with warm and wet exacerbation of their HF. Although they are well perfused and CO has not signicantly changed (i.e., disease has not progressed), their habits have caused them to retain excess uid. Dobutamine and milrinone primarily act to increase CO, which is not a signicant problem in warm and wet exacerbations. In addition, the adverse effects of these agents (increased mortality and proarrhythmias) limit their use. Nesiritide is a balanced arterial and venous dilator that decreases afterload and preload, respectively, and is the best choice for this patient without an invasive hemodynamic monitoring catheter in place. Although IV nitroglycerin is effective in warm and wet exacerbations, its use requires dosage titration by a Swan-Ganz catheter or central venous catheter, which is not in place in this patient at this time. 3. Answer: B This patient is exhibiting target organ damage from poorly controlled hypertension in the form of a cerebrovascular accident. Nicardipine is an appropriate choice in this patient, because its calcium channel blocking effects will reduce BP and potentially decrease vasospasm in the cerebral arteries, which may lead to further ischemia or seizure activity. Although fenoldopam is indicated for treating hypertensive emergency, its use is cautioned in patients with stroke symptoms, because its dopamine agonist activity can cause cerebral vasodilation and potentially reduced blood ow to the ischemic areas of the brain. Although labetalol is an effective option for treating H.E.s hypertensive emergency, she has a history of asthma and a low HR, making this drug a less-than-ideal option for treating her symptoms. The antihypertensive effects of enalaprilat are dependent on a given patients renin activity, which is not known in this patient. Therefore, the BP-reducing effects may be more difcult to control than a drug with a more consistent effect in individuals. In addition, the bolus nature of the drug is not ideal for tightly controlling BP with a 25% reduction in mean arterial pressure. Continuous infusion drugs are preferable for easier titration to effect in hypertensive emergency. 4. Answer: D Although several drug classes for treating acute MI are linked with thrombocytopenia, the timeframe in which the thrombocytopenia occurs is key in distinguishing which agent is causative. Although unfractionated heparin can produce a rapid drop in platelets on reexposure, the nadir in platelet count is typically around 50 103/mm3. Adenosine diphosphate inhibitors, particularly ticlopidine, are linked with rare, isolated thrombocytopenia. Clopidogrel is known to rarely cause thrombotic thrombocytopenic purpura, which causes thrombocytopenia and a constellation of other symptoms (mental status changes, acute renal failure, etc.) usually lasting for a longer treatment duration of weeks to months after therapy initiation. Abciximab can cause acute profound thrombocytopenia in approximately 1.5% of patients treated. The timeframe is nearly immediate (usually within 224 hours of administration initiation) and causes a nadir of about 20 103/mm3 platelets. 5. Answer: C The number needed to treat can be calculated by 1/ absolute risk reduction. Because the absolute risk reduction in mortality at 60 months was 7.2% with ICD versus placebo, 1/0.072 would be used to calculate the number of patients needed to treat to prevent one death during this time. Approximately 13.8 patients would need to be treated with ICD to prevent one death in 60 months versus placebo. Other calculations in this fashion, including relative risk reduction, do not provides useful information for interpreting the trial results. 6. Answer: C The Cardiac Arrest Study Hamburg trial compared ICD implantation with antiarrhythmic therapy in cardiac arrest survivors for secondary prevention of sudden cardiac death. The propafenone study arm was discontinued early because it had a signicantly (61%) higher mortality rate than ICDs. Although this trial had a small sample size that prevented a statistically signicant difference in total mortality from being

2008 American College of Clinical Pharmacy 1-284

Acute Care Cardiology


shown in ICD-treated patients versus patients treated with either amiodarone or metoprolol, the incidence of sudden death was signicantly reduced in patients with an ICD implanted (33% versus 13%, p=0.005). The AVID trial also evaluated ICD implantation versus antiarrhythmic drug therapy (primarily amiodarone) in survivors of sudden cardiac death. Patients implanted with ICDs had a signicantly greater rate of survival than those treated with drug therapy (89% versus 82%, p<0.02). 7. Answer: D S.V. has a depressed LVEF less than 40%, so her drug therapy options are limited to prevent development of worsening HF, which could occur if she were administered procainamide for treatment of her arrhythmia. Procainamide is indicated only in secondary prevention of sustained ventricular tachycardia (SVT) in patients with a normal LVEF more than 40%. Metoprolol is indicated only for treatment of patients with symptomatic nonsustained ventricular tachycardia and SVT associated with CAD. S.V. had a symptomatic episode of SVT. Her QTc interval is not prolonged at 380 milliseconds, so she does not require IV magnesium therapy. She qualies for treatment with either amiodarone or lidocaine. Amiodarone is rst line for patients without contraindications because of its efcacy. 8. Answer: D International Pharmaceutical Abstracts is a database of primarily pharmaceutical abstracts in more than 750 journals, including foreign pharmacy journals and state pharmacy journals, in addition to key U.S. medical and pharmacy journals. Many of the citations are not included on MEDLINE, so a broader search can be performed; however, subject descriptors are not consistently dened in a uniform way, and multiword terms are often cited backward. Iowa Drug Information Service is a database that offers full-text articles from 1966 in approximately 200 medical and pharmacy journals (primarily U.S.-based journals). It is updated monthly, so newly available articles may take longer to be accessed from this service. ClinAlert is a database of more than 100 medical and pharmacy journals focused on adverse events, drug interactions, and medical-legal issues. It is used primarily to look up adverse events (especially recent reports) associated with medications. EMBASE is a comprehensive database of more than 4000 journals from 74 countries dating from 1974 to the present. Recently published articles appear in the system within 10 days of article publication, and this database often contains data not found in a typical MEDLINE search. 9. Answer: B MedWatch is a postU.S. Food and Drug Administration approval program established by the U.S. Food and Drug Administration for health care professionals to report adverse events to the U.S. Food and Drug Administration that occur after a drug is approved. Although it is commonly used only for reporting serious reactions to the U.S. Food and Drug Administration, it can be used to report any adverse event. Information recorded on these forms is reported to the manufacturer and is used to determine whether black box warnings are necessary or whether new adverse effects are seen with a drug. The Joint Commission on Accreditation of Healthcare Organizations requires that all institutions have a denition of an ADR for the institution that all health care professionals can understand and remember. In addition, the Joint Commission on Accreditation of Healthcare Organizations requires that each dose of drug administered be monitored for adverse effects, that each institution have a system for reporting ADRs in place, and that the institution ensure that the reporting mechanism is identifying all key ADRs. 10. Answer: B Because the Pharmacy and Therapeutics committee wants to discover whether the new drug is worth the extra cost for the added mortality benets it can provide for patients with decompensated HF over currently available therapies, a cost-effectiveness analysis is the best pharmacoeconomic analysis to perform. Cost-minimization analysis is used to determine whether a therapeutically equivalent drug within a class that provides the same therapeutic outcome as others available can be used for less cost. Cost-utility analysis is used to determine whether a drug can improve the quality of a patients life more than other available therapies. Cost-benet analysis is used to evaluate new programs or services to determine whether they provide enough benet to be worth the cost to run the program.

2008 American College of Clinical Pharmacy 1-285

Acute Care Cardiology

NOTES

2008 American College of Clinical Pharmacy 1-286

Critical Care

Critical Care
Tudy Hodgman, Pharm.D., BCPS, FCCM Midwestern University Northwest Community Hospital Arlington Heights, Illinois Gretchen M. Brophy, Pharm.D., FCCP BCPS , Virginia Commonwealth University Richmond, Virginia

2008 American College of Clinical Pharmacy 1-287

Critical Care Learning Objectives:


1. Identify and distinguish between the four primary acid-base disturbances and the expected compensatory responses when provided clinical presentation and laboratory data, including arterial blood gases. Select appropriate management (drug and nondrug) for the four primary acid-base disturbances. List the indications for mechanical ventilation. Describe the indications for sedation, neuromuscular-blocking drugs, and antidelirium drugs in mechanically ventilated patients. Select appropriate agents for the sedation, neuromuscular blockade, and control of delirium in mechanically ventilated patients. Distinguish between the different types of shock. Select appropriate pharmacotherapeutic management for severe sepsis and shock. Describe the pharmacotherapy of cardiac arrest. List the risk factors and select appropriate pharmacotherapy for the prevention of stressrelated mucosal damage. 3. A. B. C. D. 2. Sodium chloride infusion. Hydrochloric acid infusion. HCO3- 100 mEq for 30 minutes. Increased tidal volume on the mechanical ventilator.

2.

3. 4.

5.

6. 7. 8. 9.

A 21-year-old man post-gunshot wound to the abdomen is receiving mechanical ventilation and is thrashing around in bed and pulling at his breathing tube. He is considered very agitated. He is receiving morphine 4 mg/hour intravenously for control of pain, which is currently 3/10. Vital signs include blood pressure (BP) 110/70 mm Hg and HR 110 beats/ minute. What is the most appropriate intervention to control this patients agitation? A. Initiate propofol 50 mcg/kg/minute intravenously. B. Give lorazepam 3 mg intravenous load followed by lorazepam 3 mg/hour intravenously. C. Give haloperidol 10 mg intravenously 1 time. D. Give lorazepam 3 mg intravenous load followed by lorazepam 3 mg/hour intravenously, and give haloperidol 10 mg intravenously 1 time. A patient is admitted to the ICU for a traumatic brain injury and multiple abdominal injuries. He is started on a high dose of propofol, morphine, and vecuronium for sedation, analgesia, and paralysis to help control his intracranial pressure. On day 3 of hospitalization, the patient develops pancreatitis and requires total parenteral nutrition. He is started on a 2-in-1 total parenteral nutrition and given lipids 20% 250 mL/day. His electrolytes on day 5 are within normal limits, with the exception of an elevated magnesium concentration. His train-of-four is 0/4. His current medications include piperacillin/tazobactam 4.5 g intravenously every 8 hours and tobramycin 350 mg/day intravenously for a possible pancreatic abscess. Which one of the following pharmacotherapeutic issues is a concern in this patient? A. Vecuronium and propofol are having an additive effect on sedation. B. High magnesium concentrations and tobramycin can inhibit the effects of vecuronium. C. Propofol needs to be considered in his daily nutritional assessment. D. Morphine is antagonized by propofol and may require a dosage increase.

Self-Assessment Questions: Answers to these questions may be found at the end of this chapter.
1. A 58-year-old woman remains intubated in the intensive care unit (ICU) after a recent abdominal surgery. In the operating room, she received more than 10 L of uid and blood products but has been aggressively diuresed since that time. In the past 3 days, she has generated 7.5 L of urine output, and her blood urea nitrogen (BUN) and serum creatinine (SCr) have steadily increased to 40 and 1.5 mg/dL, respectively. Her urine chloride concentration was 9 mEq/L (24 hours after her last dose of furosemide). This morning, her arterial blood gas reveals pH 7.50, partial pressure of carbon dioxide in arterial blood (PaCO2) 46 mm Hg, and bicarbonate (HCO3-) 34 mEq/L. Which one of the following actions is best to improve her acid-base status?

2008 American College of Clinical Pharmacy 1-288

Critical Care
4. A 43-year-old man in the medical ICU has been receiving mechanical ventilation for 3 days for management of a severe asthma exacerbation. He is sedated and comfortable and responds to commands. The patient reports no pain, and no wheezes are heard on examination. During patient care rounds in the morning, the medical team decides to start weaning the patient from the ventilator. The current sedation regimen is lorazepam 3 mg/hour intravenously continuously and morphine 2 mg/hour intravenously continuously. Which one of the following interventions will most likely hasten the weaning process and his discharge from the ICU? A. Continue the current sedation regimen. B. Continue lorazepam, discontinue morphine, and initiate fentanyl at 50 mcg/hour intravenously continuously. C. Stop all sedation and wait for the patient to fully awaken. D. Continue lorazepam and discontinue morphine. A 62-year-old woman is admitted to your ICU for respiratory dysfunction requiring mechanical ventilation. Her medical history is nonsignicant, and she is currently on no medications at home. Her chest radiograph shows bilateral lower lobe inltrates, her white blood cell (WBC) count is 21,000, temperature is 39.6C, BP 82/45 mm Hg (normal for her is 115/70 mm Hg), and HR is 110 beats/minute. She is diagnosed with communityacquired pneumonia and is empirically started on ceftriaxone 1 g intravenously daily and levooxacin 500 mg/day intravenously. Thirty-six hours later, the patients temperature is 38.6C, WBC count is 16,000, and PaO2/fractional concentration of oxygen in inspired gas (FiO2) ratio is 200. She remains hypotensive even after uid resuscitation, and her SCr has increased from 0.5 on admission to 3.2 mg/dL. She is started on a dopamine drip for BP support. Her APACHE II score is 26. Which one of the following drug therapies should you recommend for this patient at this time? A. Continue current antibiotics and monitor the patient for clinical improvement. B. Initiate a 96-hour infusion of drotrecogin- and titrate to a partial prothrombin time of 6080; continue current antibiotics. C. Initiate a 96-hour infusion of drotrecogin- and monitor for bleeding; reevaluate current antibiotics based on culture and sensitivities. D. Continue current antibiotic therapy and evaluate for drotrecogin- in 24 hours. 6. A 92-year-old woman is admitted to the ICU with urosepsis and septic shock. She is living in a nursing home and has a medical history signicant for myocardial infarction, hypertension, and congestive heart failure. Her BP is 72/44 mm Hg, HR 120 beats/minute, O2 saturation of 99%, and laboratory tests normal, except for a BUN of 74 mg/dL and a Cr of 2.7 mg/dL. Empiric antibiotics were started. Which one of the following therapies should be initiated next? A. Dobutamine. B. Epinephrine. C. Normal saline. D. Vasopressin.

5.

7. A trauma patient is admitted to the ICU and found to be in hypovolemic shock after massive blood loss. She is given multiple blood transfusions and 6 L of intravenous uids. Her medical history is signicant for a myocardial infarction 2 years ago. The patients HR is 105 beats/minute, and BP is 89/45 mm Hg; the team starts dopamine at 5 mcg/kg/minute. The BP does not increase, but the HR increases to 138 beats/minute. Which one of the following is the best vasopressor recommendation for this patient? A. Increase the dose of dopamine to 8 mcg/kg/ minute. B. Add a vasopressin continuous infusion at 0.06 units/minute. C. Initiate phenylephrine continuous infusion at 100 mcg/minute. D. Initiate epinephrine 1 mg intravenous push every 35 minutes. 8. A 46-year-old man had a witnessed cardiac arrest in an airport terminal. When emergency medical services arrived and debrillator pads were applied, pulseless ventricular tachycardia was observed. He was debrillated with 200, 300, and 360 J without return of spontaneous circulation. Epinephrine 1 mg intravenous push was given, and chest compressions and articial respirations were initiated. Within 1 minute, the patient was reassessed. The cardiac monitor still shows ventricular tachycardia, and he remains pulseless; therefore, another shock of 360 J is given. Which one of the following drugs should be administered within the next minute of cardiopulmonary resuscitation? A. Atropine 1 mg intravenously. B. Epinephrine 3 mg intravenously. C. Vasopressin 10 units intravenously. D. Amiodarone 300 mg intravenously.

2008 American College of Clinical Pharmacy 1-289

Critical Care
9. A study comparing seven different drug regimens for the treatment of actively bleeding peptic ulcer disease is being conducted in your ICU (n=75 patients per group). The primary outcome is the time (measured in minutes) to cessation of clinically evident bleeding. Which one of the following statistical tests is most appropriate for the comparison of the means between treatment groups? A. Paired t-test. B. t-test for independent samples. C. Analysis of covariance. D. Analysis of variance.

10. C.J. is a 22-year-old man admitted to the trauma ICU after a motor vehicle accident. He has multiple rib fractures, a ruptured spleen, and a small brain contusion. He is taken to the operating room for a splenectomy, and the trauma team places a postpyloric nasojejunal (NJ) feeding tube before returning him to the ICU. The patient is unresponsive and mechanically ventilated. Which one of the following is the best recommendation for stress ulcer prophylaxis (SUP)? A. Pantoprazole intravenously. B. Famotidine suspension by NJ tube. C. Sucralfate slurry by NJ tube. D. The patient needs no SUP.

2008 American College of Clinical Pharmacy 1-290

Critical Care I. ACID-BASE DISORDERS A. Primary Acid-Base Disturbances 1. Respiratory acidosis a. Etiologies i. Pulmonary/cardiovascular (massive pulmonary embolus)/cardiopulmonary arrest ii. Central nervous system depression (anesthesia, trauma, stroke, medications) iii. Impaired chest bellows (severe pulmonary edema, severe pneumonia, acute respiratory distress syndrome, end-stage chronic obstructive pulmonary disease) iv. Spinal cord or peripheral nerve injury b. Compensation: renal HCO3- increases by 1 mEq/L above normal (24 mEq/L) for every 10mm Hg increase in partial pressure of carbon dioxide (pCO2) above normal (40 mm Hg) ACUTELY or 4 mEq/L for CHRONIC c. Treatment i. Oxygen ii. Treat causes 2. Respiratory alkalosis a. Etiologies i. Central nervous systemmediated respiratory stimulation (anxiety, pain, injury, central nervous system tumor, stroke, head trauma) ii. Hypoxia-induced respiratory stimulation (high altitude, hypotension, pulseless electrical activity, congestive heart failure, pneumonia, medications [salicylates, nicotine, thyroid hormones, catecholamines, theophylline]) b. Compensation: renal HCO3- decreases by 23 mEq/L below normal (24 mEq/L) for every 10mm Hg decrease in pCO2 below normal (40 mm Hg) ACUTELY or 5 mEq/L for CHRONIC c. Treatment i. Hypoventilation ii. Sedation 3. Metabolic acidosis a. Anion gap = [Na+] ([Cl-] + [HCO3-]) Normal anion gap = (140) (104 + 24) = 12 2 mEq/L = 1014 mEq/L b. Etiologies i. Elevated anion gap: Methanol Uremia Diabetic ketoacidosis Poisoning/propylene glycol ingestion Intoxication/infection Lactic acidosis/lactate Ethylene glycol Salicylates/sepsis ii. Normal anion gap (nongap-hyperchloremic)-bicarbonate loss is accompanied by increased renal reabsorption of Cl-): diarrhea, lower GI losses, ileostomy, acid ingestion, carbonic anhydrase inhibitors, renal tubular acidosis iii. Compensation: respiratory pCO2 decreased by 11.5 mm Hg below normal (40 mm Hg) for each 1-mEq/L decrease in HCO3- below normal (24 mEq/L) d. Treatment 2008 American College of Clinical Pharmacy 1-291

Critical Care i. pH 7.207.35: correct causes ii. pH less than 7.20: consider direct administration of base (a) HCO3- Calculate bicarbonate decit (in mEq) = (body wt)(0.4 L/kg)(24 serum HCO3-) Give 50 mEq intravenous bolus and then infusion (150 mEq of NaHCO in 1 L of dextrose 5% in water [D5W]). May replace up to half of bicarbonate decit during 4 hours. (b) Tromethamine (c) Bicarbonate equivalents: for mild and/or chronic disease (1) Citrate (oral) (2) Acetate (parenteral) (3) Lactate (parenteral) 4. Metabolic alkalosis a. Etiologies i. Net loss of hydrogen ion ii. Net gain of bicarbonate (a) Saline-resistant (urine Cl- more than 20 mmol/L) Hyperaldosteronism Mineralocorticoid excess (Cushing or Bartters syndrome) Severe K+ depletion (b) Saline-responsive urine (Cl- less than 10 mmol/L); chloride loss is the culprit (loss of Cl-rich, bicarbonate-poor uid)GI disorders, diuretics, cystic brosis, chronic respiratory acidosis b. Compensation: respiratory: pCO2 increases by 0.60.7 mm Hg above normal (40 mm Hg) for every 1mEq/L increase in HCO3- above normal (24 mEq/L) c. Treatment i. Saline-resistant treatment: correct the causes (renal or adrenal disease) ii. Saline-responsive treatment pH less than 7.55: intravenous normal saline pH more than 7.55: HCl intravenously Calculate chloride decit (in milliequivalents) = (body wt)(0.2 L/kg)(103 serum Cl-) Administer half of chloride decit over at least 12 hours through a central line (0.1 N HCl contains 100 mEq of chloride) (a) Alternatives: Ammonium chloride, Acetazolamide (Diamox) B. Detection of Acid-Base Disturbances 1. Serum bicarbonate: if no arterial blood gas is available (as is usually is the case), then an isolated decrease (serum bicarbonate less than 20 mEq/L) or increase (serum bicarbonate more than 28 mEq/L) indicates a metabolic acidosis and alkalosis, respectively. 2. Arterial blood gas interpretation is reported as pH/pCO2/pO2/HCO3-/oxygen saturation (SaO2) Normal values pH = 7.40 (range = 7.357.45) pCO2 = 40 mm Hg (range = 3545 mm Hg) pO2 = 80100 mm Hg HCO3- = 24 mEq/L (range = 2226 mEq/L) SaO2 = 95%100% (percent hemoglobin fully saturated with O2)

2008 American College of Clinical Pharmacy 1-292

Critical Care Step 1: Diagnose the acid-base disturbance a. Look at pH. Less than 7.35: acidosis More than 7.45: alkalosis b. Look at pCO2. An increased pCO2 with a low pH is a respiratory acidosis. A decreased pCO2 with a high pH is a respiratory alkalosis. c. Look at bicarbonate to verify your suspected diagnosis. An increased bicarbonate with a high pH is a metabolic alkalosis. A decreased bicarbonate with a low pH is a metabolic acidosis. Step 2: Check for compensation a. Compensatory value is near normal or normal: little or no compensation (i.e., pCO2 should increase or decrease in response to a metabolic disturbance). b. If the compensatory value has signicantly changed in the appropriate direction, then it is a compensated disturbance Step 3: Isolate the cause(s) of the acid-base disturbance (e.g., anion gap). Useful hints to remember for compensation: a. Acute respiratory acidosis: an acute 10-mm Hg increase in pCO2 will be buffered by an increase in HCO3- of about 1 mEq/L. b. Chronic respiratory acidosis: a chronic 10-mm Hg increase in pCO2 will be buffered by an increase in HCO3- of about 4 mEq/L. c. Acute respiratory alkalosis: an acute 10-mm Hg decrease in pCO2 will be buffered by a decrease in HCO3 of about 23 mEq/L. d. Chronic respiratory alkalosis: a chronic 10-mm Hg decrease in pCO2 will be buffered by a decrease in HCO3- of about 5 mEq/L. e. Metabolic acidosis: use Winters formula to predict full compensation: expected pCO2 = 1.5 (HCO3-) + 8 2. f. Metabolic alkalosis: each 10-mEq/L increase in HCO3- will be buffered by an increase in pCO2 of about 67 mm Hg.

2008 American College of Clinical Pharmacy 1-293

Critical Care Patient Cases


1.

A 62-year-old woman has been hospitalized in the ICU for several weeks. Her hospital course was complicated by aspiration pneumonia and sepsis, requiring prolonged courses of antibiotics. During the past few days, she has begun spiking fevers again, and her stool output has increased dramatically. Her most recent stool samples have tested positive for Clostridium difcile toxin, and her laboratory tests show serum Na 138 mEq/L; K 3.5 mEq/L; Cl 115 mEq/L; albumin 4.4 g/dL; pH 7.32; PaCO2 30 mm Hg; and HCO3- 15 mEq/L. Which one of the following acid-base disturbances is consistent with this patients arterial blood gas? A. Anion gap metabolic acidosis. B. Normal anion gap metabolic acidosis. C. Saline-responsive metabolic alkalosis. D. Acute respiratory acidosis.

2.

An 18-year-old man with no medical history is brought to the emergency department in a semicomatose state. His parents report that he was complaining of a vague abdominal pain earlier in the morning, and then began vomiting and urinating frequently in the hours before admission. His toxicology screen proved negative for drugs of abuse, but his urine tested positive for ketones. Laboratory tests show serum Na 142 mEq/L; K 4.5 mEq/L; Cl 100 mEq/L; glucose 795 mg/dL; lactate 1.0 mmol/L; pH 7.26; PaCO2 23 mm Hg; and HCO3- 10 mEq/L. Which one of the following acid-base disturbances is consistent with this patients arterial blood gas? A. Anion gap metabolic acidosis. B. Normal anion gap metabolic acidosis. C. Saline-responsive metabolic alkalosis. D. Acute respiratory acidosis. A 27-year-old man with no medical history is admitted to the hospital after being found down at a party, where he reportedly ingested a fth of whiskey during a 20-minute period. On arrival to the emergency department, he was neurologically unresponsive and had the following arterial blood gas values: pH 7.23, PaCO2 58 mm Hg, PaO2 111 mm Hg, HCO3- 24 mEq/L, and SaO2 100% on 2 L/minute of oxygen by nasal cannula. Which one of the following actions is best? A. Albuterol 4 puffs every 20 minutes for 4 hours; then, 24 puffs every 24 hours as needed. B. 100% oxygen by face mask. C. HCO3- 100 mEq over 30 minutes. D. Intubate and transfer to the ICU.

3.

2008 American College of Clinical Pharmacy 1-294

Critical Care II. RESPIRATORY FAILURE AND MECHANICAL VENTILATION


Table 1. Indications for Intubation/Mechanical Ventilation Indication Hypoventilation (hypercapnic respiratory failure) Examples Drug overdose Neuromuscular disease Cardiopulmonary resuscitation Central nervous system injury or disease Diagnostic Test Ventilation Indicated > 5055 mm Hg PaCO2

Hypoxemia Pulmonary injury or disease (hypoxic respiratory Pneumonia failure) Pulmonary edema Adult respiratory distress syndrome Airway protection Loss of airway patency (mechanical obstruction, tracheal/chest wall injury)

PaO2 SaO2

< 60 mm Hg < 88%92%

CXR, RR, airow assessment

Lack of airway, RR persistently < 6 Loss of consciousness, negative gag reex

Loss of gag/cough reex with large volume aspiration Gag/cough reex risk (central nervous system injury, central nervous system depression, CVA, seizures, cardiac arrest, etc.) CVA = cerebrovascular accident; CXR = chest x-ray; RR = respiratory rate.

A. Acute Respiratory Distress Syndrome 1. Denition: clinical syndrome including respiratory distress, severe hypoxemia, diffuse chest radiography inltrates, decreased lung compliance, and an association with an underlying medical or surgical condition 2. Etiologies: sepsis, trauma, pneumonia, pancreatitis, burns, noxious inhalation, emboli (amniotic, fat, air), massive blood transfusions, eclampsia, radiation, poisonings, others 3. Pathophysiology a. Inammatory phase exudative phase i. Mediators of injury ii. Apoptosis iii. Endothelial injury iv. Epithelial injury v. Cytokine cascade vi. Pulmonary vascular dysregulation b. Fibroproliferative phase chronic phase i. Repair phase of acute respiratory distress syndrome ii. Fibrosis iii. Angiogenesis c. Recovery phase 4. Diagnostic criteria (American-European Consensus Conference on Acute Respiratory Distress Syndrome, 1994) a. PaO2/FiO2 ratio less than 200 (acute lung injury requires PaO2/FiO2 ratio 200300) b. Chest radiograph with bilateral pulmonary inltrates consistent with pulmonary edema

2008 American College of Clinical Pharmacy 1-295

Critical Care c. No clinical or pulmonary artery catheterization evidence of heart failure (pulmonary artery occlusion pressure less than 18 mm Hg) 5. Clinical features a. Rapid (within 1248 hours of insult) development of dyspnea at rest and hypoxemia (PaO2 less than 60 mm Hg, SaO2 less than 90%) b. Decreased lung compliance c. Tachypnea (respiratory rate more than 20) d. Small VT (less than 5 mL/kg) 6. Nonpharmacologic management a. Mechanical ventilation (may require deeper sedation) i. Pressure-cycled (pressure-targeted) ventilation 1. Limit VT to 46 mL/kg. 2. Limit plateau pressure to less than 30 cm H2O. ii. Permissive hypercapnia: allow pCO2 to rise to meet the above objectives; keep pH more than 7.25. iii. Titrate positive end-respiratory pressure to keep PaO2/FiO2 ratio more than 200 without cardiac compromise iv. Titrate FiO2 down as quickly as possible (less than 55%) b. Fluid restriction 7. Pharmacologic management a. Fluid management b. Lung-specic pharmacotherapy i. Correction of physiologic abnormalities (a) Inhaled nitric oxide (b) Surfactant (c) Peruorocarbons ii. Suppression of lung inammation (a) Corticosteroids: after 7 days but before 28 days of acute respiratory distress syndrome; methylprednisolone 1 mg/kg intravenously every 12 hours (or 2 mg/kg/day) for 14 days, 1 mg/kg/day for 7 days, 0.5 mg/kg/day for 7 days, 0.25 mg/kg/day for 2 days, 0.125 mg/kg/day for 2 days, and then stop Other anti-inammatory agents were not effective.

Patient Cases 4. A 55-year-old woman with a history of severe chronic obstructive pulmonary disease is admitted after several days of worsening shortness of breath. Recently, she was discharged from the hospital after a similar episode and was doing ne until 3 days before admission, when she developed a productive cough. This cough required an increase in her home O2 and more frequent use of her metered-dose inhalers. On admission to the medical ICU, she was anxious and markedly distressed with rapid, shallow breaths. She was hypertensive (160/80 mm Hg), tachycardic (140 beats/minute), and tachypneic (28). Her arterial blood gas showed pH 7.30, PaCO2 59 mm Hg, PaO2 50 mm Hg, HCO3- 28 mEq/L, and SaO2 83% on 6 L/minute of oxygen by face mask, and she was immediately intubated. Which one of the following acid-base disturbances is consistent with this patients presentation and laboratory data? A. Metabolic acidosis. B. Metabolic alkalosis. C. Respiratory acidosis. D. Respiratory alkalosis.

2008 American College of Clinical Pharmacy 1-296

Critical Care
5. Her initial ventilator settings included assist control mode at a rate of 20 breaths/minute (although she is still breathing 36 times/minute) with a tidal volume of 700 mL and an FiO2 of 100%. A subsequent arterial blood gas showed pH 7.54, PaCO2 34 mm Hg, PaO2 49 mm Hg, HCO3- 28 mEq/L, and SaO2 82%. Which one of the following acid-base disturbances is consistent with this arterial blood gas? A. Mixed metabolic acidosis with a respiratory alkalosis. B. Mixed metabolic acidosis with a respiratory acidosis. C. Mixed metabolic alkalosis with a respiratory acidosis. D. Mixed metabolic and respiratory alkalosis.

III. SEDATION IN MECHANICALLY VENTILATED PATIENTS


Patient Cases 6. Overnight, the patient in case 4 continued to breathe rapidly, appeared very agitated, and eventually began pulling at her endotracheal tube. Her last arterial blood gas showed PaO2 55 mm Hg and SaO2 89%, despite 100% FiO2. The medical team decides to sedate her to help coordinate her breathing with the ventilator and reduce oxygen consumption. Which one of the following statements about sedatives is true? A. Lorazepam has a quicker onset of action than midazolam. B. Fentanyl causes more hypotension and histamine release than morphine. C. Haloperidol has been associated with ventricular arrhythmias. D. Propofol may accumulate in renal dysfunction. 7. An ICU patient is agitated and having difculty breathing. The physician decides to intubate the patient and wants you to recommend drug therapy for sedation. Which one of the following is the best recommendation for this patient? A. Start propofol at 5 mcg/kg/minute and titrate until the patient is sedated; then, intubate the patient. B. Start lorazepam intravenous continuous infusion before intubating the patient. C. Give vecuronium intravenous bolus and start a continuous infusion; then, intubate the patient. D. Give midazolam 2.5 mg intravenously now and rebolus until the patient is sedated; then, intubate the patient.

A. Goals of ICU Sedation (set a goal, evaluate response, and communicate) 1. Improve oxygenation/ventilator interaction. 2. Decrease anxiety/stress response. a. Initiate after achieving adequate analgesia, and treat reversible causes. b. Assess pain response to therapy regularly with a scale/tool appropriate for patient (e.g., visual analog scale, critical care pain observation tool). 3. Avoid self-injury. 4. Allow completion of invasive patient care. 5. Promote normal sleep cycles. B. Complications of ICU Sedation 1. Prolonged ICU stay 2. Prolonged mechanical ventilation 3. Physiologic dependence (withdrawal reactions) 4. Respiratory depression 5. Delirium

2008 American College of Clinical Pharmacy 1-297

Critical Care C. Sedative Agents 1. Benzodiazepines


Trade name Pharmacokinetics Onset (minutes) Duration of effect (hours) Prolonged in renal failure Prolonged in hepatic failure Elimination half-life (hours) Active metabolites Adverse effects Hypotension Thrombophlebitis Diazepam Valium 24 24 Yes Yes 2448 Yes Yes Yes Lorazepam Ativan 2040 46 No Yes 1020 No No Maybe Midazolam Versed 24 12 Yes Yes 14 Yes No No

2. Opioid analgesics
Trade name Pharmacokinetics Onset (minutes) Duration of effect (hours) Prolonged in renal failure Prolonged in hepatic failure Elimination half-life (hours) Active metabolites Adverse effects Hypotension Flushing Bronchospasm Constipation Morphine Various 24 24 Yes Yes 13 Yes Yes Yes Yes Yes Fentanyl Sublimaze 12 12 No Yes 25 No No No No Yes Hydromorphone Dilaudid 24 26 Yes Yes 23 No Yes Yes No Yes

3. Miscellaneous sedative agents a. Propofol (Diprivan) i. Rapid onset (12 minutes) and short duration (35 minutes) ii. Starting dosage is 5 mcg/kg/minute, titrating by 5 mcg/kg/minute every 5 minutes to achieve desired effect; no loading dose should be given because of the increased risk of hypotension. iii. Monitor BP and triglycerides; adjust lipid content in total parenteral nutrition (if applicable). iv. Propofol can induce apnea but is being used for conscious sedation; use caution in nonintubated patients. v. Propofol infusion syndrome (rare and often fatal): rapid-onset metabolic acidosis with cardiac failure, rhabdomyolysis, and renal failure b. Dexmedetomidine (Precedex) i. A selective 2-agonist with analgesic and sedative properties ii. Short distribution (5 minutes) and elimination half-lives (25 hours) iii. A loading dose of 1 mcg/kg given for 20 minutes is suggested for operating room patients; however, loading doses are NOT recommended for ICU patients (bradycardia risk). iv. The maintenance dose is 0.20.7 mcg/kg/hour. The dose should be decreased in patients with hepatic insufciency; maximum duration of 24 hours per U.S. Food and Drug Administration (FDA) labeling 2008 American College of Clinical Pharmacy 1-298

Critical Care D. Managing ICU Sedation 1. Nonpharmacologic therapy: frequent reorientation, maintain comfort and analgesia, optimize environment (e.g., lighting, noise), use relaxation techniques, back massage, music therapy 2. Correct any correctable causes of agitation (endotracheal tube placement, vent settings, pain, etc.) before pharmacologic intervention. 3. Address pain management before initiating a sedative agent. 4. Achieve desired level of sedation with boluses BEFORE initiating a maintenance infusion or bolus dose regimen (Bolus Before). 5. Regain desired level of sedation with boluses BEFORE increasing the maintenance infusion or bolus dose regimen (Bolus Before). 6. Evaluate adequacy of sedation using an objective sedation scale, and titrate infusion to minimum effective dose; set goal at least daily. Goal should be comfortable, arousable sedation. No evidence for one tool over another. a. Ramsey scale: intended for use in the operating room (general anesthesia) b. Sedation-agitation scale, Richmond Agitation-Sedation Scale, Motor Activity Assessment Scale: all validated in the ICU c. Bispectral analysis: correlate clinical signs with electroencephalogram, potential role for monitoring sedation in a patient who is paralyzed E. Clinical Considerations for Highest Quality Care: 1. Optimize combination therapy a. Additive sedation effects b. Minimize adverse events from high-dose, single-agent therapy. 2. Daily sedation interruption: reassess needs daily. What is your goal today? 3. Daily sedation tapering 4. Use of a sedation protocol/guideline with a sedation-assessment scale is strongly recommended. 5. When used for more than a few days, especially at high doses, taper over time to avoid withdrawal (10%20% decrease per day). 6. Remember adjunctive therapies: Lacri-Lube for eyes as needed, deep vein thrombosis prophylaxis, and physical therapy.

IV. ICU DELIRIUM A. Clinical Features 1. Denition: acute-onset mental status, with uctuating course, and the ability to focus, sustain, or shift attention is impaired 2. Common themes a. Sensory alterations b. Reversed sleep-wake pattern c. Alternating lethargy/agitation (hypoactive/hyperactive) d. Fluctuations in orientation and memory 3. Risk factors a. Age: older than 60 years b. Substance abuse history (including tobacco) c. Psychiatric history (especially dementia)/central nervous system event d. Infection 2008 American College of Clinical Pharmacy 1-299

Critical Care e. Untreated pain f. Electrolyte/metabolic derangements g. Use of benzodiazepines before ICU admission 4. Tool for assessment a. Confusion assessment method for the ICU: validated in the ICU, requires patient participation b. Intensive Care Delirium Screening Checklist B. Nonpharmacologic Interventions 1. Baseline mental status 2. Obtain psychiatric history if possible. 3. Maintain patient communication. 4. Maximize uninterrupted sleep (limit noise/talking). 5. Maximize natural lighting. 6. Remove unnecessary equipment from room. 7. Encourage patient autonomy. C. Pharmacologic Interventions 1. Butyrophenone neuroleptics a. Haloperidol (Haldol): drug of choice for ICU delirium i. Neuroleptization dosing: (a) Mild delirium: 2 mg intravenously (b) Moderate delirium: 5 mg intravenously (c) Severe delirium: 10 mg intravenously ii. Rapid dose escalation: (a) Double the previous dose every 20 minutes. If no response, increase to maximum 80-mg bolus. (b) After the rst two doses, start giving 1 mg of lorazepam with each haloperidol dose. (c) When patient is calm, add total milligrams administered and give intravenously for the next 35 days, divided into doses every 6 hours. iii. Taper over 57 days. iv. Adverse effects (a) Cardiovascular: hypotension, torsades de pointes (b) Extrapyramidal effects: after prolonged duration (more than 7 days) (c) Lowering of seizure threshold b. Droperidol (Inapsine): not effective for delirium 2. Atypical antipsychotics (possible alternative therapy for patients intolerant to haloperidol) a. Olanzapine (Zyprexa) 2.55 mg orally/nasogastric tube/orogastric tube/intramuscularly once daily (520mg total daily dose) b. Risperidone (Risperdal) 210mg oral total daily dose c. Quetiapine (Seroquel) 75750mg oral total daily dose d. Ziprasidone (Geodon) 40160mg orally or 10- to 40-mg intramuscular total daily dose

2008 American College of Clinical Pharmacy 1-300

Critical Care
Patient Case 8. A 42-year-old woman with a signicant history of alcohol and tobacco abuse and acute respiratory distress syndrome is transferred to the medical ICU from an outside hospital. She presented to the outside hospital after 1 week of productive cough, fevers, chills, and increasing shortness of breath. On admission to the medical ICU, she is hypotensive (80/60 mm Hg), tachycardic (130 beats/minute), and febrile (39.0). Her arterial blood gas shows pH 7.1, PaCO2 56 mm Hg, PaO2 49 mm Hg, HCO3- 16 mEq/L, and SaO2 76% on 100% FiO2. The only other signicant laboratory results were an SCr of 1.5 mg/dL and WBC 16,000. She appears to be adequately sedated with 3 mg/hour of lorazepam and 200 mcg/hour of fentanyl. In an attempt to improve her oxygenation, she is paralyzed and placed on inverse ratio ventilation. Which one of the following statements about therapeutic paralysis is true? A. Sedatives should be discontinued after paralysis is initiated. B. Once paralysis is initiated, it should not be stopped until after the patient is extubated. C. Depth of paralysis should be monitored with a peripheral nerve stimulator. D. The goal of monitoring paralysis is to continuously observe four of four twitches on train-of-four.

V. THERAPEUTIC PARALYSIS A. Indications for Neuromuscular Blockade 1. Poor oxygenation or patient-ventilator interaction that persists despite adequate sedation 2. Potentially dangerous movements in an instrumented patient that persists despite sedation 3. Assist in the control of elevated intracranial pressure 4. Assist in the control of muscle spasm (tetanus, neuroleptic malignant syndrome) B. Nondepolarizing Neuromuscular-blocking Agents for Prolonged Use (not just intubation, e.g., rapid-sequence intubation)
Trade name Duration of effect (hours) Prolonged in renal failure Prolonged in hepatic failure Loading dose Maintenance Adverse effects Tachycardia Hypotension Daily drug cost per 70 kg Pancuronium Pavulon 0.751.5 Yes Yes 0.08 mg/kg 0.020.04 mg/kg as needed Yes No $5$15 Vecuronium Norcuron 0.50.75 Yes Yes Atracurium Tracrium 0.250.5 No No Cisatracurium Nimbex 0.51 No No 0.1 mg/kg 210 mcg/kg/ minute No No $200$400

0.1 mg/kg 0.4 mg/kg 0.020.04 mg/kg/ 0.4 mg/kg/hour hour No No $100$200 No Dose dependent $100$200

C. Monitoring Therapeutic Paralysis 1. Adequate sedation and analgesia MUST be achieved before starting paralytic agent; sedation should be ordered around the clock as a continuous infusion (never as needed). 2. Dosage should be monitored with a peripheral nerve stimulator. a. A baseline train-of-four should be documented before initiating paralysis. b. Doses should be titrated to maintain one or two twitches on train-of-four and desired clinical effect. c. Depth of paralysis (i.e., train-of-four) should be assessed several times per day. 3. Paralysis should be allowed to dissipate at least once daily (drug holiday) to ensure adequate sedation and assess the need for continued paralysis. Once this assessment is made, patients may be reparalyzed, if necessary. 2008 American College of Clinical Pharmacy 1-301

Critical Care E. Common Factors That May Affect Neuromuscular Blockade


Drugs Potentiate Block Corticosteroids Aminoglycosides Clindamycin Tetracyclines Colistin Calcium channel blockers Type Ia antiarrhythmics Furosemide Lithium Hypermagnesemia Hypocalcemia Hypokalemia Antagonize Block Aminophylline Theophylline Carbamazepine Phenytoin (chronic)

Electrolyte disorders

Hypercalcemia Hyperkalemia

Patient Cases 9. The patient in case 8 was paralyzed as instructed and appeared to being doing well until about 1 hour after her third dose of pancuronium, when she began to move around violently in her bed. At this time, she was tachycardic (120 beats/minute) and appeared very agitated; her SaO2 fell to 80%, and the nurse reported that the patient had regained all four twitches on train-of-four. Which one of the following actions is best? A. Administer a pancuronium bolus. B. Administer a fentanyl bolus. C. Increase the lorazepam drip rate. D. Change pancuronium to vecuronium. 10. After that event, the patient did poorly throughout the rest of the night. A Swan-Ganz catheter was placed, conrming the diagnosis of sepsis (i.e., high cardiac output and low systemic vascular resistance). The patient was started on a dopamine infusion at 20 mcg/kg/minute to maintain an adequate BP. Other medications included clindamycin, cefepime, and gentamicin. By morning, her SCr has increased to 2.8 mg/dL, and the night shift nurse reports that the patient had zero of four twitches on train-of-four for the past 8 hours. Which one of the following may potentiate the effects of pancuronium? A. Clindamycin. B. Gentamicin. C. Renal failure. D. All of the above. 11. Which one of the following agents has been associated with prolonged paralysis when used with the nondepolarizing neuromuscular-blocking agents? A. Penicillin. B. Prednisone. C. Theophylline. D. Digoxin.

2008 American College of Clinical Pharmacy 1-302

Critical Care VI. SHOCK AND SEPSIS A. Differential Diagnosis of Shock Based on Hemodynamic Parameters
Hemodynamic Cardiac Index PCWP Subset (2.54.0 L/minute/m2) (812 mm Hg) Septic High Low SVR (8001400) Low Treatment

Fluids Vasopressors Antibiotics Hypovolemic Low Low High Normal saline Colloids Blood products Cardiogenic Low High High Inotropes Afterload reducers Diuretics PCWP = pulmonary capillary wedge pressure; SVR = systemic vascular resistance.

B. Surviving Sepsis Campaign: international guidelines for management of severe sepsis and septic shock: 2008 (Dellinger RP, Levy MM, Carlet JM, et al. Intensive Care Med 2007: DOI 10.1007/ s00134-007-0934-2). *Only drug- or uid-related recommendations are included in this chart.* Sepsis = systemic response to infection (conrmed or suspected infection PLUS more than two systemic inammatory response syndrome criteria) Severe sepsis = sepsis associated with organ dysfunction or tissue hypoperfusion or hypotension (systolic BP [SBP] less than 90 or mean arterial pressure less than 70 or drop of SBP more than 40 mm Hg)
Hypoperfusion abnormalities include (but are not limited to): Altered metal status Hyperglycemia in the absence of diabetes Cardiac index less than 3.5 L/minute/m2 Acute oliguria (more than 2 hours) Increased SCr (more than 0.5 mg/dL over baseline) Thrombocytopenia: platelets less than 100,000/cm3 Hyperlactatemia (more than 4 mmol/L) Edema or increased uid balance Decreased venous oxygen saturation Arterial hypoxemia PaO2/FiO2 less than 300 Coagulopathy (international normalized ratio more than 1.5 with no anticoagulant) Ileus (absent bowel sounds) Hyperbilirubinemia (more than 40 mg/dL) Decreased capillary rell or mottling

2008 American College of Clinical Pharmacy 1-303

Critical Care
Table 2. 2008 Surviving Sepsis Campaign International Guidelines for Management of Severe Sepsis and Septic Shock Recommendation Treatment Rationale/Clinical Pearls Early goal-directed therapy: (rst 6 hours of Fluids Improved survival in patients resuscitation) Red blood cell transfusions presenting with septic shock in CVP 812 mm Hg (1215 if ventilated) (hematocrit > 30%) emergency department MAP 65 mm Hg Dobutamine Urine output: 0.5 mL/kg/hour (maximum 20 mcg/kg/ Central venous (ScVO2) > 70% or mixed minute) SvO2 65% Abx Start within the rst hour of Change to narrow-spectrum Abx Empiric therapy of one or two drugs that recognition of severe sepsis, when possible to prevent resistance, cover likely pathogens; broad spectrum after appropriate cultures are reduce toxicity, and reduce cost. Penetrate presumed source of sepsis obtained Be aware that blood cultures will Reassess daily be negative in most cases of sepsis; Combination therapy for therefore, use clinical judgment and 35 days, with deescalation other cultures as a basis to change therapy therapy when sensitivities available D/C after 710 days, unless slow response, undrainable foci, or immunosuppression D/C Abx if noninfectious cause is determined No clinical outcome differences Fluid resuscitation Most patients require between treatments; crystalloids Crystalloid (normal saline, lactated ringers) aggressive resuscitation require more uid to achieve the Colloid (albumin, hetastarch) during the rst 24 hours of same end points and result in more management Fluid challenges: 1000 mL of edema crystalloids or 300500 mL of colloids over 30 minutes; repeat based on response Decrease rate of uid challenge if cardiac lling pressures (CVP or PAOP) increase without concurrent hemodynamic improvement Vasopressors (central line) NE (more potent) or DA Advantages of these agents over After appropriate uid challenge fails to (good for systolic dysfunction epinephrine (potential tachycardia, restore BP and organ perfusion but more tachycardia and dec splanchnic perfusion, and Use arterial catheter for BP arrhythmias and HPA and hyperlactemia) and phenylephrine Vasopressin may be considered in refractory immunosuppressive effects) (decreased stroke volume) shock (0.03 units/minute, no titration) Epinephrine should be the Vasopressin may decrease cardiac Do NOT use DA for renal protection alternative if NE or DA fail output Inotropic therapy Dobutamine First-line inotropic therapy For low cardiac output after adequate uid (220 mcg/kg/minute) Goal: achieve adequate levels of resuscitation oxygen delivery or avoid owCombine with vasopressor in patients with dependent tissue hypoxia low BP

2008 American College of Clinical Pharmacy 1-304

Critical Care
Table 2. 2008 Surviving Sepsis Campaign International Guidelines for Management of Severe Sepsis and Septic Shock Recommendation Steroids Intravenous corticosteroids For SHOCK refractory to vasopressors No contraindication to continuing maintenance steroid therapy or using stress dose steroids in patients with sepsis on maintenance steroid therapy or with endocrine disorders Treatment Hydrocortisone 200300 mg/day for 7 days, divided TID or QID Some experts would add udrocortisone 50 mcg/day PO if hydrocortisone is not available Rationale/Clinical Pearls Signicant shock reversal and decrease in mortality ONLY in patients who are nonresponsive to uids and pressors Do NOT use doses of corticosteroids > 300 mg hydrocortisone Decrease dosage or taper off steroids when vasopressors are no longer required Uncertain if tapering or abrupt discontinuation is best Improves survival in patients with sepsis-induced organ failure (PROWESS) Once patient has been identied as high risk, start treatment immediately Do not use in APACHE II < 20 or one organ failure

Recombinant human activated protein C For patients at high risk of death (APACHE II > 25) or sepsis induced (> 2) multiple organ failure and with no contraindications Endogenous anticoagulant with antiinammatory properties

24 mcg/kg/hour for 96 hours, based on actual body weight No change in PK parameters was observed in patients weighing up to 227 kg (Levy H, Small D, Heiselman DE, et al. Obesity does not alter the pharmacokinetics of drotrecogin alfa (activated) in severe sepsis. Ann Pharmacother 2005;39:2627)

Blood product administration Only when hemoglobin decreases to < 7 g/ dL to target a hemoglobin of 79 g/dL Do not use fresh frozen plasma for clotting abnormalities in the absence of bleeding or planned invasive procedures EPO is NOT recommended as a specic treatment of anemia but may be used in patients with renal failure and compromised RBC production Platelets should be given when < 5000/ mm3 regardless of apparent bleeding, 500030,000/mm3 and signicant risk of bleeding. Maintain platelets > 50,000/ mm3 for surgery or invasive procedures Sedation, analgesia, and neuromuscular blockade Use protocols and sedation scales (to a predened end point) to manage sedation of mechanically ventilated patients Intermittent bolus or continuous infusion sedation Neuromuscular blockers should be avoided if possible because of the risk of prolonged blockade after discontinuation in patients with sepsis. Train-of-four monitoring should be conducted to determine depth of block

A transfusion threshold of 7 g/dL was NOT associated with increased mortality RBC transfusion in patients with sepsis increases O2 delivery but not consumption This contrasts with the hematocrit target of 30% in the rst 6 hours of resuscitation of septic shock EPO use in critical care clinical trials shows some decrease in transfusions but no effect on clinical outcome

Daily interruptions/lightening of continuous sedation with awakening and retitration if necessary can decrease duration of MV and ICU stay (improved neurologic assessment and decreased cost) Sedation protocols have been shown to decrease the duration of MV, LOS, and tracheostomy rates Prolonged skeletal muscle weakness has been reported after neuromuscular blockade in critically ill patients Keep inspiratory plateau pressures < 30 cm H20

2008 American College of Clinical Pharmacy 1-305

Critical Care
Table 2. 2008 Surviving Sepsis Campaign International Guidelines for Management of Severe Sepsis and Septic Shock. Recommendation Treatment Rationale/Clinical Pearls Glucose control Continuous infusion of Improvement in survival of surgical Maintain blood glucose < 150 mg/dL with insulin and glucose patients when insulin was used to an insulin protocol Protocol should be developed maintain glucose between 80 and 110 Monitor glucose every 12 hours until for patient safety mg/dL stable and then on a regular basis (every 4 Minimize hypoglycemia Best results were obtained when hours) by continuous glucose glucose was 80110, but < 150 mg/dL A nutritional protocol with preference of supplementation (5% or 10% also improved outcome when compared the enteral route should be included in the dextrose infusion followed by with higher concentrations treatment of patients with severe sepsis continuous enteral feeds) Control of blood glucose appears to requiring glycemic control Use of POC testing should be more important than the amount of be interpreted with caution, insulin given because of the potential for Use arterial or central catheters for overestimation of glucose blood sampling because of frequency values Bicarbonate therapy No evidence to support its use for the Not recommended for the treatment of purpose of improving hemodynamics hypoperfusion/lactic acidemia with pH or reducing vasopressor requirements > 7.15 at lower pH or clinical outcome at any pH Benet of DVT prophylaxis has been DVT prophylaxis Low-dose unfractionated conrmed in trials of general ICU Should be considered for all patients with heparin or low-molecularpopulations, including patients with severe sepsis weight heparin If contraindications sepsis Consider combination therapy (thrombocytopenia, Patients at very high risk should (pharmacologic and mechanical) in very severe coagulopathy, receive a low-molecular-weight heparin high-risk patients (e.g., severe sepsis, active bleeding, or recent because they are superior in this history of DVT) intracerebral hemorrhage): population use mechanical devices H2-receptor antagonist SUP Patients with severe sepsis and septic Should be given to all patients with severe (preferred agents) shock present with conditions that sepsis Proton pump inhibitors have been shown to benet from SUP: Benet must be weighed against the coagulopathy, MV, hypotension potential for increased risk of ventilatorassociated pneumonia Abx = intravenous antibiotics; APACHE II = Acute Physiology and Chronic Health Evaluation II; BP = blood pressure; CVP = central venous pressure; DA = dopamine; D/C = discontinue; DVT = deep vein thrombosis; EPO = erythropoietin; HPA = hypothalamus-pituitary-adrenal; ICU = intensive care unit; LOS = length of stay; MAP = mean arterial pressure; MV = mechanical ventilation; NE = norepinephrine; PAOP = pulmonary artery occlusion pressure; PK = pharmacokinetics; PO = orally; POC = point-of-care; PROWESS = Recombinant Human Activated Protein C Worldwide Evaluation in Severe Sepsis [PROWESS] Trial; QID = 4 times/day; RBC = red blood cell; SUP = stress ulcer prophylaxis; SvO2 = mixed venous oxygen saturation; TID = 3 times/day.

2008 American College of Clinical Pharmacy 1-306

Critical Care C. Vasopressors and Inotropic Agents


Drug
Dopamine Dose

13 mcg/kg/ minute NOT recommended 310 mcg/kg/ minute 1020 mcg/kg/ minute

1 +/-

1 ++

2 Dopa Cautions/Clinical Effects +/- ++++ Renal, coronary, mesenteric, and cerebral arterial vasodilation
and natriuretic response

++

+++

+ 0

++ +

++++ +++

Norepinephrine

0.011 mcg/kg/ ++++ +++ minute for septic shock Dose should be tapered slowly 0.041 mcg/ kg/minute for refractory hypotension

Epinephrine

+++

+++

++

Phenylephrine

0.58 mcg/kg/ ++++ minute for septic shock Dose should be tapered slowly 0.03 units/ minute (physiologic replacement dose) 220 mcg/kg/ minute

A choice in sepsis can BP by contractility and SVR Can induce tachyarrhythmias Immediate precursor of norepinephrine Prolonged infusions can deplete endogenous norepinephrine stores resulting in a loss of vasopressor response Effects on renal blood ow may be lost at higher doses because of predominant 1 effects Renal perfusion SVR, BP 0 cardiac output Can induce tachyarrhythmias and myocardial ischemia Extravasation produces ischemic necrosis and sloughing (treatment: phentolamine 510 mg IV diluted) Positive inotropic and chronotropic effects can induce myocardial ischemia SVR, BP Low doses = -adrenergic; -adrenergic = with escalating doses Extravasation produces ischemic necrosis and sloughing (treatment: phentolamine 510 mg diluted) Renal perfusion Pure -adrenergic agonist with minimal cardiac activity Rapid SBP and DBP can cause a reex bradycardia Extravasation produces ischemic necrosis and sloughing (treatment: phentolamine 510 mg diluted) Direct stimulation of smooth muscle V1 receptors; peripheral vasoconstriction, no adrenergic activity Doses > 0.04 units/minute associated with coronary vasoconstriction and peripheral necrosis Positive inotrope cardiac output can lead to SVR Higher doses can cause tachyarrhythmias and changes in BP, which can lead to myocardial ischemia

Vasopressin

Dobutamine

+++

50 mcg/kg load 0 0 0 0 Noncatecholamine, phosphodiesterase inhibitor Positive inotrope for 10 minutes, Vasodilation, arrhythmias possible followed by Use lower doses in renal failure 0.3750.75 mcg/ kg/minute = -adrenergic effect; = -adrenergic effect; BP = blood pressure; DBP = diastolic blood pressure; Dopa = dopaminergic effect; SBP = systolic blood pressure; SVR = systemic vascular resistance; ++++ = maximal effect ranging to 0 = no effect; increase; decrease.
Milrinone (Primacor)

2008 American College of Clinical Pharmacy 1-307

Critical Care D. Recombinant Human Activated Protein C (Drotrecogin- [Xigris]) 1. Criteria for use (per the Recombinant Human Activated Protein C Worldwide Evaluation in Severe Sepsis [PROWESS] Trial): patients must meet ALL three criteria. a. Known or suspected infection b. At least THREE of the following systemic inammatory response syndrome criteria: i. Temperature 36C or less OR 38C or more ii. HR 90 beats/minute or more iii. Respiratory rate 20 or more OR PaCO2 32 mm Hg or less or need for intubation and mechanical ventilation iv. WBC of 4000 or less OR 12,000 or more OR 10% or more immature neutrophils c. At least ONE of the following sepsis-induced organ or system dysfunctions within the past 48 hours i. Mean arterial pressure less than 70 or SBP less than 90 or need for vasopressor support after adequate uid challenge ii. UOP less than 0.5 mL/kg/hour for 1 hour after adequate uid challenge iii. PaO2/FiO2 ratio less than 250 iv. Platelets less than 80,000 or 50% decrease from baseline within 72 hours v. Unexplained metabolic acidosis pH 7.3 or less or base excess 5 or more with lactate more than 1.5 times normal 2. Contraindications: active internal bleeding; recent (within 3 months) hemorrhagic stroke; recent (within 2 months) intracranial or intraspinal surgery or severe head trauma; trauma with an increased risk of life-threatening bleeding; presence of an epidural catheter; intracranial neoplasm or mass lesion; or evidence of cerebral herniation 3. Monitoring signs and symptoms of bleeding: the study showed a trend toward increased risk for serious bleeding events in the drotrecogin- group (p=0.06); patients were excluded from the study for platelet count less than 30,000, conditions that increased the risk of bleeding; the activated partial prothrombin time may be elevated during therapy, but should not be used to determine adjustments in treatment. 4. Cost of therapy: $7000$8000 (70kg patient 96 hour infusion)

VII. ADVANCED CARDIAC LIFE SUPPORT (ACLS)/CARDIOPULMONARY RESUSCITATION


Patient Case 12. A 65-year-old woman was admitted to the coronary care unit after suffering a myocardial infarction. On the fourth day of hospitalization, she is hypotensive (BP 80/50 mm Hg), tachycardic (HR 125 beats/minute), tachypneic (respiratory rate 30), hypoxemic (PaO2 55 mm Hg), febrile (102F), and confused. The patient is given two 500-mL boluses of normal saline, intubated, and started on piperacillin/tazobactam 4.5 g intravenously every 8 hours and ciprooxacin 400 mg intravenously every 12 hours for possible nosocomial pneumonia. After further uid boluses fail to improve her clinical status, a pulmonary artery catheter is placed, which reveals a pulmonary capillary wedge pressure of 14 mm Hg, cardiac index of 3.8 L/minute/m 2, and systemic vascular resistance of 515 dynes/second/cm-5. Her chest radiograph shows diffuse interstitial inltrates, and she is still requiring 100% FiO2. Which one of the following actions is best? A. Add clindamycin 600 mg intravenously every 8 hours for possible aspiration pneumonia. B. Add drotrecogin- 24 mcg/kg/hour immediately for severe sepsis. C. Dobutamine infusion titrated to achieve an SBP of 100 mm Hg. D. Norepinephrine infusion titrated to achieve an SBP of 100 mm Hg.

2008 American College of Clinical Pharmacy 1-308

Critical Care

A. Early Cardiopulmonary Resuscitation (Chain of Survival): 1. Check for responsiveness 2. Activate emergency response systems 3. Call for debrillator 4. ABCDs Airway: Open the airway. Breathing: Provide positive pressure ventilations. Circulation: Chest compressions Debrillation: Assess and shock ventricular brillation/pulseless ventricular tachycardia, up to 3 times (200 J, 200300 J, and 360 J) if necessary. B. Goals 1. Cerebral resuscitation 2. Return of spontaneous circulation a. Cessation of arrhythmia b. Adequate vascular tone 3. American Heart Association Treatment algorithms can be found on the following web sites (accessed March 1, 2008): http://circ.ahajournals.org/cgi/reprint/112/24_supp/IV-78 http://circ.ahajournals.org/cgi/reprint/112/24_supp/IV-58 http://circ.ahajournals.org/cgi/reprint/112/24_supp/IV-67

2008 American College of Clinical Pharmacy 1-309

Critical Care C. ACLS Medication Management


Drug Epinephrine (Epi) Indication Shock refractory ventricular brillation (VF)/pulseless ventricular tachycardia (VT) Pulseless electrical activity (PEA) Asystole Bradycardia w/serious signs or symptoms Equivalent to Epi as rst line for shock refractory VF/ pulseless VT PEA Asystole Hypovolemic shock Dose 1 mg intravenous push (IVP)/intra-osseous (IO) every 35 minutes High-dose Epi is not recommended Continuous infusion (for bradycardia) 210 mcg/minute 40 units IVP/IO one time only (two vials of 20 units/ mL) to replace either rst or second dose of Epiif no response after 1020 minutes, start Epi at 1 mg IVP every 35 minutes Endotracheal tube dose same as IV (but endotracheal route not recommended) 0.040.1 units/minute IV infusion Shock refractory VF/pulseless 300 mg IVP/IO diluted in 20 VT mL of D5W Can repeat at a dose of 150 Atrial brillation/utter mg IV push Maintenance continuous infusion 1 mg/minute for 6 hours, then decrease to 0.5 mg/minute thereafter 150 mg IV over 10 minutes for breakthrough VF Max cumulative dose of 2.2 g IV/24 hours Adverse Effects Tachyarrhythmias, myocardial infarction (MI), tissue necrosis from extravasation Compatibility/Pearls Not compatible with sodium bicarbonate (HCO3-) (inactivates catecholamines)

Vasopressin

Hypertension, bradycardia, MI, decreased cardiac output, possible necrosis if inltrates into the tissue

Compatible with normal saline (NS) and D5W Onset 12 minutes, halflife: 1020 minutes (can last up to 60 minutes)

Amiodarone

Hypotension (decrease infusion rate, give uid bolus), bradycardia (decrease or discontinue infusion, pacemaker)

Dilute with D5W Undiluted may cause more hypotension Maintenance infusion should be prepared in a nonpolyvinyl chloride (PVC) container Not compatible with HCO3Hepatically metabolized with multiple drug interactions Long half-life and large volume of distribution

Reduce maintenance infusion by 50% in hepatic dysfunction, congestive heart failure, elderly (> 70 years) Contraindicated in patients with hypersensitivity to amide local anesthetics Infusion rates > 30 mg/ Procainamide Intermittent/recurrent VF/VT Loading dose up to 17 mg/ Hypotension, QRS/ minute exacerbate adverse Atrial brillation/utter kg until rhythm suppressed, QT prolongations, widening of QRS or QT depressed myocardial effects Common cause of torsades intervals by 50% of baseline, contractility, proarrhythmic effects de pointes or hypotension Maximum rate of infusion 20 mg/minute Maintenance infusion 14 mg/minute Lidocaine Shock refractory VF/pulseless 11.5 mg/kg IVP/IO VT Repeat in 35 minutes Maximum dose 3 mg/kg Continuous infusion 14 mg/minute

Bradycardia, sinus arrest, seizures

2008 American College of Clinical Pharmacy 1-310

Critical Care
Drug Atropine Indication Symptomatic sinus bradycardia Atrioventricular (AV) block at the nodal level Asystole or PEA with relative or absolute bradycardia Narrow complex paroxysmal supraventricular tachycardia (PSVT) Dose Adverse Effects 0.5 mg IVP/IO as needed for Tachycardia, bradycardia or AV block mydriasis 1 mg IVP/IO every 35 minutes (3 mg max) for asystole or PEA Rapid 6 mg IV bolus for 13 Flushing, dyspnea, seconds chest pain, transient May repeat twice at a dose bradycardia of 12 mg IV bolus for 13 seconds after 12 minutes Follow each bolus with 20 mL of normal saline ush Compatibility/Pearls Paradoxical bradycardia at dose < 0.5 mg in adults

Adenosine

Calcium chloride Magnesium sulfate

Hyperkalemia Hypocalcemia Calcium channel blocker toxicity Drug of choice for torsades de pointes Potential benet in refractory VF/VT Hypomagnesemia

12 g IVP 10% solution over 10-minute intervals

Brief period of asystole may follow rapid injection Half-life < 10 seconds Methylxanthines (e.g., caffeine, theophylline) are competitive antagonists and can be used to reverse adenosine-induced hypotension and/or bradycardia Lower dose (3 mg) with carbamazepine, dipyridamole, or cardiac transplant Higher doses with large caffeine users and theophylline Hypotension, syncope, Not compatible with HCO3(forms precipitate) bradycardia, skin necrosis Hypotension, asystole, Do not exceed a rate of 150 ushing mg/minute because of risk of severe hypotension or asystole Hyperosmolarity, hypernatremia, intracellular and cerebral acidosis, mixed venous hypercarbia, tissue necrosis possible if inltration occurs Dosing guided by arterial blood gas (ABG) FLUSH IV lines before and after use due to compatibility issues

HCO3-

12 g in 10 mL of D5W IVP over 12 minutes Give IV push in VF May be followed by a continuous infusion of 0.51 g/hour Hyperkalemia 1 mEq/kg IVP Documented or preexisting Repeat doses of bicarbonate responsive 0.5 mEq/kg every 10 acidosis or tricyclic overdose minutes during continued or to alkalinize the urine cardiac arrest in aspirin or other drug overdoses Hypoxic lactic acidosis

D. Antiarrhythmics in ACLS 1. Consider underlying cardiac function before choosing agent (all are negative inotropes except for amiodarone). 2. It is better not to combine multiple antiarrhythmics secondary to No. 1. E. Intravenous Drug Administration 1. Central venous administration preferred 2. Peripheral administration should be followed by a 1020 mL ush of 0.9% NaCl (normal saline), and elevate the limb. 3. Continuous infusions should be administered by a central line. 4. Monitor for extravasation. 5. Do NOT stop cardiopulmonary resuscitation for drug administration. 2008 American College of Clinical Pharmacy 1-311

Critical Care

F. Intra-osseous Administration Preferred Over Endotracheal if Intravenous Administration Not Possible G. Endotracheal Drug Administration 1. Dose = 22.5 times the standard intravenous dose 2. Dilute the dose in 510 mL of sterile water. 3. Stop cardiopulmonary resuscitation. 4. Inject deep into endotracheal tube; give ve quick insufations. 5. Resume cardiopulmonary resuscitation. 6. Drugs that can be given by endotracheal tube Lidocaine Atropine Naloxone Epinephrine Vasopressin (same as intravenous dose, no adjustment) H. Hypothermia (3234C) 24 hours postarrest 1. Improve neurologic recovery and mortality 2. Ice packs to armpits, neck, torso, and groin 3. Cooling blankets 4. Iced uid infusions 5. Cooling pads 6. Adjunctive sedation, analgesia, neuromuscular blockade
Patient Cases 13. Which one of the following statements about epinephrine is true? A. The recommended dose of epinephrine is 1 mg intravenously every 10 minutes. B. Benecial effects of epinephrine in cardiac arrest are caused by its -agonist effects. C. Epinephrine is the drug of choice for bradycardia. D. High-dose epinephrine is more effective than the 1 mg dose. 14. A 70 kg patient is to receive a continuous infusion of dopamine for BP support. The nurse has a 250 mL bag of D5W containing 400 mg of dopamine. At what rate should the dopamine drip be given to provide the patient with a dose of 5 mcg/kg/minute? A. 13 mL/hour. B. 13 mL/minute. C. 22 mL/hour. D. 22 mL/minute.

VIII. STRESS ULCER PROPHYLAXIS (SUP) A. Pathogenesis of Stress-related Mucosal Damage (SRMD) 1. Physiologic stressmucosal ischemiainability to maintain homeostasis in the gastric mucosafailure of mucosal defense mechanismsSRMDbleeding 2. Disruption of mucosal defense mechanisms a. Decreased mucosal blood ow b. Decreased secretion of mucus and bicarbonate c. Decreased protective prostaglandin production d. Decreased cell regeneration e. Decreased gastrointestinal (GI) motility 2008 American College of Clinical Pharmacy 1-312

Critical Care B. Overt Stress-related Mucosal Bleeding 1. Hematemesis 2. Gross blood in nasogastric aspirates: coffee grounds 3. Hematochezia 4. Melena C. Clinically Signicant Stress-related Mucosal Bleeding 1. Overt bleeding complicated by any one of the following: Decreased SBP of 20 mm Hg or more Increased HR more than 20 beats/minute Decreased SBP more than 10 mm Hg orthostatic change Decreased hemoglobin by 2 g/dL or more without recovery after transfusion D. Risk Factors for Clinically Important Stress-related Mucosal Bleeding 1. Mechanical ventilation more than 48 hours (independent risk factor) 2. Coagulopathy more than 24 hours (platelet less than 50,000/mm3 or international normalized ratio more than 1.5 or partial prothrombin time more than 2 times control) (independent risk factor) or any two or more of the following: 3. Neurologic trauma (head injury/spinal cord injury) 4. Hypoperfusion (sepsis, shock) 5. Severe burns (more than 35% of body surface area) 6. Multiple organ failure three or more organs 7. Medical history of GI ulcers/bleeding within 1 year 8. High-dose steroids (more than 200 mg of hydrocortisone equivalents) 9. Multiple traumas 10. Liver failure with associated coagulopathy 11. Postoperative transplant 12. Acute renal insufciency 13. Major surgery E. Goals of Therapy for SUP
Gastric pH 3.5 4.5 5 7 8 Physiologic Effects Decreased incidence of stress-related mucosal bleeding Pepsin inactivation Acid neutralization Clotting and platelet aggregation Pepsin destruction

F. Therapeutic Options for SUP 1. Antacids


Dose-dependent Neutralization of Acid Electrolyte imbalances Diarrhea/constipation Disadvantages Require multiple daily doses and frequent administration Available only for PO or NG administration Numerous drug interactions Risk of nosocomial pneumonia caused by high gastric volumes NG = nasogastric; PO = orally. Action Adverse effects

2008 American College of Clinical Pharmacy 1-313

Critical Care 2. Sucralfate (Carafate)


Action Forms a complex by binding with positively charged proteins in exudates, forming a protective coating that protects the lining. It may also stimulate prostaglandin release, causing an increase in bicarbonate and mucus production. 1 g tablet PO/NG QID

Constipation Possible aluminum accumulation in renal dysfunction Disadvantages Inferior to H2RA for the prevention of clinically signicant bleeding More frequent dosing than H2RA and PPIs Drug interactions Only available for PO or NG administration H2RA = H2-receptor antagonist; NG = nasogastrically; PO = orally; PPI = proton pump inhibitor; QID = 4 times/day.

Available agents/ dosage Adverse effects

3. H2-Receptor Antagonists (H2RAs)


Action Available agents/ dosage (dosage based on clinical data, not FDA approved for SUP) Adverse effects Advantages Competitive blocker of H2-receptors on parietal cells Ranitidine 150 mg PO every 12 hours or 50 mg IV every 8 hours Famotidine 20 mg IV/PO every 12 hours Nizatidine 150 mg PO every 12 hours Cimetidine 300 mg PO/IV every 6 hours or continuous infusion 37.550 mg/ hour (only FDA-approved agent for SUP)

Mental status changes, thrombocytopenia Ease of administration Cost Disadvantages Drug interactions (cimetidine) Potential for reduced efcacy over time (tolerance) Adjustment in dose for renal dysfunction Risk for nosocomial pneumonia FDA = U.S. Food and Drug Administration; IV = intravenously; PO = orally; SUP = stress ulcer prophylaxis.

2008 American College of Clinical Pharmacy 1-314

Critical Care 4. Proton Pump Inhibitors (PPIs)


Action Prodrugs that are activated in the acidic environment of the parietal cell and then bind to and inhibit active proton pumps. Oral formulations are designed to dissolve at a pH > 5.6 to protect from degradation and premature activation in the stomach. Available agents/dose PO (dose based on Omeprazole (Prilosec) clinical data, not FDA Capsules 20 mg/day PO approved for SUP) Capsules 20 mg/day by tube o Open capsule and suspend granules in a syringe with 40 mL of apple juice and give by NGT/OGT, ush with 20 mL apple juice (DO NOT CRUSH GRANULES) o Simplied omeprazole suspension (SOS): by NGT/OGT dissolve granules in 10 mL of 8.4% HCO3- and ush with 10 mL of HCO3- or water Powder for oral suspension (Zegerid) 20 mg/day PO Esomeprazole (Nexium) Capsules 40 mg/day PO Capsules 40 mg by tube o Suspend granules in a syringe with 50 mL of water and give by NGT/OGT, ush with 10 mL of water so all granules are delivered (DO NOT CRUSH GRANULES) Lansoprazole (Prevacid) Capsules 30 mg/day PO Capsules 30 mg by tube o Open capsule and suspend granules in a syringe with 40 mL of apple juice and give by NGT/OGT, ush with 20 mL of apple juice (DO NOT CRUSH GRANULES) o Simplied lansoprazole suspension (SLS): dissolve granules in 10 mL of 8.4% HCO3- and ush with 10 mL of HCO3- or water Delayed-release orally disintegrating table (Prevacid SoluTab) 30 mg, dissolve in 10 mL of water and give PO/NGT/OGT; ush NGT/OGT with 5 mL of water (NGT/OGT > 8 French) Delayed-release suspension: DO NOT USE by NGT/OGT, xantham gum in formulation that will likely clog the NGT/OGT Pantoprazole (Protonix) Enteric-coated tablet 40 mg/day PO Enteric-coated tablet 40 mg by tube o Crush and dissolve tablet in 10 mL of 4.2% HCO3-, add an additional 10 mL for a total volume of 20 mL, and ush with 10 mL of HCO3- or water Rabeprazole (AcipHex): 20 mg/day enteric-coated tablet PO IV (ONLY for patients who can NOT tolerate PO/NG administration): Lansoprazole 30 mg/day IV In-line lter required, administer for 30 minutes, little incompatibility information Pantoprazole 40 mg/day IV No lter required (ush before and after administration); administer for 25 minutes Esomeprazole 2040 mg/day IV; administer over 3 minutes; little compatibility information Headache, diarrhea, constipation, abdominal pain, nausea No adjustment needed for renal or liver dysfunction Drug interactions: omeprazole and esomeprazole (2C19 inhibitors: diazepam, phenytoin, warfarin), lansoprazole (1A2 inducer: theophylline) Cost Administration issues Risk of nosocomial pneumonia Risk of Clostridium difcile infection (nosocomial or community acquired)

Adverse effects Advantages Disadvantages

FDA = U.S. Food and Drug Administration; HCO3- = sodium bicarbonate; IV = intravenous; NG = nasogastric; NGT = nasogastric tube; OGT = orogastric tube; PO = orally; SUP = stress ulcer prophylaxis.

2008 American College of Clinical Pharmacy 1-315

Critical Care G. Recommendations for Therapy 1. Identify patients at high risk for SRMD. 2. Evaluate route of administration. a. Nasogastric/oral accessH2RA b. No nasogastric/oral accessintravenous H2RA i. H2RA-associated thrombocytopeniaintravenous PPI 3. Discontinue therapy when risk factors are no longer present. If the risk factor is gone, the SUP should be gone.
Patient Cases 15. A 73-year-old woman is admitted to the ICU after an episode of cardiac arrest with successful resuscitation. She was intubated during the code. Now, she is being mechanically ventilated. Twenty-four hours after resuscitation, she develops acute renal failure. Her BP is currently 104/65 mm Hg, her HR is 88 beats/minute, her O2 saturations are 98% on 40% FiO2 and PEEP 5, and her Glasgow Coma Scale score is 11. She has a nasogastric tube in place, is being fed enterally, and has no gastric residuals. Her current medications include amiodarone 400 mg 2 times/day, simvastatin 20 mg every night, heparin 5000 units subcutaneously every 8 hours, and 0.9% NaCl intravenously at 75 mL/hour. The physicians would like to start SUP. Which one of the following is the best recommendation for this patient? A. Famotidine 20 mg intravenously every 12 hours. B. Simplied lansoprazole suspension 30 mg/day by NG tube. C. Pantoprazole 80 mg intravenous bolus and then 8 mg/hour as a continuous infusion. D. Cimetidine 50 mg/hour as a continuous infusion. 16. One week later, the patient from question 15 has improved substantially. She is extubated, and her acute renal failure has resolved. Her Glasgow Coma Scale score is 15, her BP is 112/70 mm Hg, and her HR is 75 beats/ minute; she is eating a general diet. Which one of the following statements is true regarding SUP for this patient? A. SUP should continue for 6 weeks. B. SUP should be discontinued because she no longer has risk factors for SRMD. C. A PPI is more effective than an H2RA for SUP. D. Either an H2RA or PPI is appropriate prophylaxis at this time.

2008 American College of Clinical Pharmacy 1-316

Critical Care REFERENCES


Acid/Base 1. Gehlbach BK, Schmidt GA. Bench-to-bedside review: treating acid-base abnormalities in the intensive care unitthe role of buffers. Crit Care 2004;8:25965. 2. Adrogue HJ, Madias NE. Management of lifethreatening acid-base disorders. Part I. N Engl J Med 1998;338:2634. 3. Adrogue HJ, Madias NE. Management of lifethreatening acid-base disorders. Part II. N Engl J Med 1998;338:10711. Respiratory Failure and Mechanical Ventilation 1. Bulger EM, Jurkovich GJ, Gentilello LM, Maier RV. Current clinical options for the treatment and management of acute respiratory distress syndrome. J Trauma 2000;48:56271. 2. Ware LB, Matthay MA. The acute respiratory distress syndrome. N Engl J Med 2000;342:133449. 3. Adhikari N, Burns KEA, Meade MO. Pharmacologic therapies for adults with acute lung injury and acute respiratory distress syndrome. The Cochrane Database of Systematic Reviews 2004, Issue 4. Article No.: CD004477. pub2. DOI: 10.1002/14651858.CD004477.pub2. Sedation 1. Jacobi J, Fraser GL, Coursin DB, et al. Clinical practice guidelines for the use of sedatives and analgesics in the critically ill adult. Crit Care Med 2002;30:11941. 2. Kress JP, Pohlman AS, OConnor MF, Hall JB. Daily interruption of sedative infusions in critically ill patients undergoing mechanical ventilation. N Engl J Med 2000;342:14717. 3. Pun B, Dunn J. The sedation of critically ill adults. Part 1. Assessment. Am J Nurs 2007;107:408. Delirium 1. Ely EW, Shintani A, Truman B, et al. Delirium as a predictor of mortality in mechanically ventilated patients in the intensive care unit. JAMA 2004;291:175362. 2. Milbrandt EB, Kersten A, Kong L, et al. Haloperidol use is associated with lower hospital mortality in mechanically ventilated patients. Crit Care Med 2005;33:2269. 3. ICU Delirium and Cognitive Impairment Study Group: brain dysfunction in critically ill patients. Vanderbilt Medical Center. Available at http:// www.icudelirium.org/delirium. Accessed February 27, 2006.

Therapeutic Paralysis 1. Murray MJ, Cowen J, DeBlock H, et al. Clinical practice guidelines for sustained neuromuscular blockade in the adult critically ill patient. Crit Care Med 2002;30:14256. Shock and Sepsis 1. Surviving Sepsis Campaign: international guidelines for management of severe sepsis and septic shock: 2008 (Dellinger RP, Levy MM, Carlet JM, et al. Intensive Care Med 2007: DOI 10.1007/ s00134-007-0934-2). 2. Dellinger RP, Carlet JM, Masur H, et al. Surviving Sepsis Campaign guidelines for management of severe sepsis and septic shock. Crit Care Med 2004;32:85873. 3. Dellinger RP, Masur H, Carlet JM, Gerlach H, eds. The Surviving Sepsis Campaign guidelines for the management of severe sepsis and septic shock: background, recommendations, and discussion from an evidence-based review. Crit Care Med 2004;32:S445597. 4. Hollenberg SM, Ahrens TS, Annane D, et al. Practice parameters for the hemodynamic support of sepsis in adult patients: 2004 update. Crit Care Med 2004;32:192848. 5. Kumar A, Mann HJ. Appraisal of four novel approaches to the prevention and treatment of sepsis. Am J Health-Syst Pharm 2004;61:76576. 6. Mutlu GM, Factor P. Role of vasopressin in the management of septic shock. Intensive Care Med 2004;30:127691. 7. Russell JA. Management of sepsis. N Engl J Med 2006;355:1699713. Advanced Cardiac Life Support/Cardiopulmonary Resuscitation 1. American Heart Association. Advanced Cardiovascular Life Support Provider Manual. Dallas, TX: American Heart Association, 2006. 2. Wenzel V, Krismer AC, Arntz HR, Sitter H, Stadbauer KH, Linder KH. A comparison of vasopressin and epinephrine for out-of-hospital

2008 American College of Clinical Pharmacy 1-317

Critical Care
cardiopulmonary resuscitation. N Engl J Med 2004;350:10513. American Heart Association. ACLS algorithms. Available at http://www.ace.cc/ new%20acls%20guidelines.htm OR http:// w w w. a m e r i c a n h e a r t .o r g /d o w n l o a d a b l e / heart/1053714937281ACLSPROV_ App3.pdf. Accessed February 27, 2006. Emergency Cardiovascular Care Committee, Subcommittees and Task Forces of the American Heart Association. 2005 American Heart Associate guidelines for cardiopulmonary resuscitation and emergency cardiovascular care. Circulation 2005;112(suppl 24):1203.

3.

4.

Stress Ulcer Prophylaxis 1. ASHP Commission on Therapeutics. ASHP therapeutic guidelines on stress ulcer prophylaxis. Am J Health-Syst Pharm 1999;56:34779. 2. Allen ME, Kopp BJ, Erstad BL. Stress ulcer prophylaxis in the postoperative period. Am J Health-Syst Pharm 2004;61:58896. 3. Cook D, Guyatt G, Marshall J, et al. A comparison of sucralfate and ranitidine for the prevention of upper gastrointestinal bleeding in patients requiring mechanical ventilation. N Engl J Med 1998;338:7917. 4. Cadle RM, Mansouri MD, Logan N, et al. Association of proton-pump inhibitors with outcomes in Clostridum difcile colitis. Am J Health-Syst-Pharm 2007;64:235963.

2008 American College of Clinical Pharmacy 1-318

Critical Care ANSWERS AND EXPLANATIONS TO PATIENT CASES


1. Answer: B This arterial blood gas is consistent with a metabolic acidosis. The pH is less than 7.40 (indicating it is an acidosis), and the HCO3- and PaCO2 are both decreased from normal. In a metabolic acidosis, the decrease in HCO3- is the primary disorder. When a metabolic acidosis is present, the anion gap should be calculated to provide additional insight regarding the potential cause of the disorder. The anion gap is calculated by subtracting the sum of measured anions (Cl- and HCO3-) from cations (Na+). This patients anion gap (8 mEq/L) is within the normal range of 612 mEq/L; thus, it is referred to as a normal anion gap metabolic acidosis or nonanion gap metabolic acidosis. C. difcileinduced diarrhea is the most likely cause of this patients acid-base disorder. 2. Answer: A This patient has an elevated anion gap metabolic acidosis (anion gap = 32 mEq/L). The causes of an anion gap metabolic acidosis may be remembered using the pneumonic MUDPILES (see III.C). This patients presentation is most consistent with diabetic ketoacidosis (complaints of vague abdominal pain, vomiting, polyuria, elevated glucose, and positive urine ketones). As illustrated by this case, ketoacidosis may be the initial presenting manifestation of type 1 diabetes mellitus. 3. Answer: D Given this patients neurologic status and his elevated PaCO2, he should be intubated and transferred to the ICU. In patients without chronic obstructive pulmonary disease, a PaCO2 above 50 mm Hg is usually an indication for mechanical ventilation regardless of oxygenation status (this patient was oxygenating well; PaO2 111 mm Hg, SaO2 100%). Albuterol or oxygen therapy alone is unlikely to correct this patients cause of respiratory failure (i.e., hypoventilation). Likewise, his acid-base disturbance is consistent with pure acute respiratory acidosis (elevated PaCO2 and normal HCO3) and is therefore unlikely to respond to HCO3 which is usually reserved for severe metabolic acidosis. 4. Answer: C This arterial blood gas is consistent with a respiratory acidosis. The pH is below 7.40 (indicating it is an acidosis), and the PaCO2 (an acid) is higher than normal (about 40 mm Hg). In chronic respiratory acidosis, the kidneys will conserve bicarbonate (a base) in an attempt to maintain a normal pH. This compensatory response is obvious in this patient, because her serum HCO3- is 28 mEq/L (about 4 mEq/L higher than normal). The elevated HCO3concentration in this patient conrms the diagnosis of respiratory acidosis (because one would expect the HCO3- to be less than 24 mEq/L if the acidemia were due to a metabolic cause). 5. Answer: D Now, the patient has a mixed metabolic and respiratory alkalosis. The pH is greater than 7.40 (indicating that it is an alkalosis), and now, the PaCO2 (an acid) is below normal, leading one to suspect that this is simply a respiratory alkalosis. However, this patients HCO3- (a base) is still elevated at 28 mEq/L (due to long-standing compensation for an elevated PaCO2). If this were a pure respiratory alkalosis, the serum HCO3- would be 24 mEq/L, because the primary and compensatory changes always occur in the same direction in simple acid-base disorders. Because the HCO3- is above normal and the PaCO2 is below normal, this is a mixed disorder. In this case, the pH indicates it is an alkalosis; the PaCO2 is below normal, which is consistent with a respiratory alkalosis, and the HCO3- is above normal, which is consistent with a metabolic alkalosis (therefore, it is a mixed metabolic and respiratory alkalosis). This occurred because of rapid hyperventilation (36 breaths/minute on assisted-control ventilation) in a patient with a chronically elevated HCO3- concentration, which is a normal compensatory response in chronic CO2 retention. Serum HCO3- concentrations require more time to correct than PaCO2 concentrations. Ideally, the ventilator mode/rate would be adjusted to maintain a PaCO2 that is normal for the patient (in this case, probably around 50 mm Hg). 6. Answer: C One of the primary disadvantages of injectable haloperidol for ICU sedation is the risk for QTc prolongation, torsades de pointes, and ventricular brillation. This risk is probably greatest in patients with a baseline QTc more than 480 milliseconds, those with hypokalemia, those with hypomagnesemia, or those with concomitant medications known to prolong QT intervals. The other answers are wrong for obvious reasons. 7. Answer: D The patient should be given midazolam 2.5 mg intravenously now and then be rebolused until the patient is sedated; then, the patient should be intubated. Midazolam is a short-acting sedative used frequently for rapid sedation. The drug should be dosed to desired effect. Propofol can induce apnea and is only recommended for use in intubated

2008 American College of Clinical Pharmacy 1-319

Critical Care
patients. All sedatives should be given as bolus doses to achieve rapid sedation goals before starting a continuous infusion; therefore, answer B is not correct. Vecuronium is a paralytic; it should never be used in a patient who is not intubated and sedated. 8. Answer: C Although it is not universally accepted, there is a consensus among experts that peripheral nerve stimulation should be used to guide therapeutic paralysis in the ICU. This is most often accomplished with the train-of-four sequence. It is imperative for clinicians to recognize that neuromuscular-blocking agents do not cross the blood-brain barrier and are not useful as either sedatives or analgesics. Adequate sedation and analgesia must be achieved before starting a paralytic agent and should continue throughout the paralysis. In addition, the paralytic should be allowed to dissipate at least once daily to assess the adequacy of sedation/analgesia. The goal response on trainof-four is typically one or two twitches. Four of four twitches would indicate inadequate paralysis (or possibly misplacement of electrodes directly over a muscle group). 9. Answer: B Although the patient is no longer paralyzed (moving around in bed, four of four twitches on train-of-four), it would be inappropriate to reparalyze an obviously agitated patient. That she is so agitated and tachycardic could be the result of being paralyzed without adequate sedation or analgesia. Before reinstituting pancuronium, the patient should be given a rapid-acting sedative, such as fentanyl (midazolam or diazepam may also be appropriate). In paralyzed patients, it is generally better to err on the side of oversedation rather than undersedation, so an increase in the sedative drip rates would also be appropriate in this patient. However, lorazepam has such a long half-life (and slow onset) that simply increasing the drip rate without rst giving a bolus of a rapid-acting agent would not result in timely sedation. Switching paralytics is unwarranted because her tachycardia can be explained by factors other than the pancuronium. 10.Answer: D Clindamycin and gentamicin have pharmacodynamic effects (i.e., they inhibit the release of acetylcholine at the nicotinic receptor), which may potentiate the action of nondepolarizing neuromuscular-blocking agents. About 60%80% of a pancuronium dose is excreted unchanged in the urine; therefore, renal insufciency may also result in signicantly prolonged effects. 11.Answer: B Prolonged treatment with either a neuromuscularblocking agent or a corticosteroid can result in acute myopathy. However, the combined treatment appears to be associated with a signicantly higher risk. Numerous reports have described profound muscle weakness (lasting days to weeks) in otherwise healthy asthmatic patients who received methylprednisolone, hydrocortisone, dexamethasone, or prednisone during therapeutic paralysis. Because most of these patients had no other risk factors for prolonged paralysis (e.g., organ dysfunction, aminoglycosides), it would be prudent to avoid this combination when possible. 12. Answer: D This patients hemodynamic prole is most consistent with sepsis (i.e., high cardiac index and low systemic vascular resistance). Her pulmonary capillary wedge pressure is also relatively low considering the degree of uid challenge she has received. Because she remains hypotensive despite receiving an adequate uid load, an -adrenergic agent such as dopamine or norepinephrine should be started. Norepinephrine is a more potent vasoconstrictor than phenylephrine and provides less -stimulation than dopamine. If she became more tachycardic on norepinephrine, phenylephrine could be tried. Dobutamine is an inotropic agent that increases the cardiac index and lowers pulmonary capillary wedge pressure. Dobutamine is usually avoided when the SBP is less than 100 mm Hg. The goals of treatment are to improve BP (typically, mean arterial pressure) and restore adequate organ perfusion. Piperacillin/ tazobactam and ciprooxacin will provide adequate gram-positive, gram-negative, and anaerobic coverage for nosocomial pneumonia, eliminating the need for clindamycin. The patient appears to meet the criteria for the use of drotrecogin-; however, her symptoms started less than 24 hours ago, and there has not been an adequate amount of time for evaluation of the current antibiotic treatment. There is no urgency to start drotrecogin-, because therapy appears to be benecial when initiated within 48 hours of meeting criteria. 13. Answer: B When using epinephrine in the patient with cardiac arrest, the primary goal of therapy is return of spontaneous circulation. Therefore, the benecial effects of epinephrine are caused by its -agonist effects. The recommended dose of epinephrine is a 1 mg intravenous push every 35 minutes during resuscitation. High-dose epinephrine and escalating doses of epinephrine are no longer recommended. Epinephrine is used as rst-line therapy for ventricular tachycardia/pulseless ventricular brillation, pulseless electrical activity, and asystole. It can be used for bradycardia with serious symptoms, but the rst-line agent for bradycardia is atropine.

2008 American College of Clinical Pharmacy 1-320

Critical Care
14. Answer: A Calculating an infusion rate is a very important role for the pharmacist in code situations. The infusion pump is set to run in milliliters per hour, so your answer should always be in these units. To determine the rate (milliliter per hour) to achieve a 5 mcg/kg/minute dose, use the following calculation: concentration of dopamine drip: 400 mg/250 mL = 1.6 mg/mL or 1600 mcg/mL 70 kg 5 mcg/kg/minute 60 minutes/1 hour 1 mL/1600 mcg = 13 mL/hour 15. Answer: B Although this patient had hypotensive episodes during her resuscitation period, she currently has a functioning GI system, as evidenced by her tolerance of tube feeds. Therefore, SUP should be given through her nasogastric tube if possible, making the best choice for this patient the simplied lansoprazole suspension. The remaining answers were not appropriate because they were intravenous. In addition, the famotidine and cimetidine dose choices were not appropriate for renal dysfunction, and the pantoprazole dose listed was for a GI bleed, not SUP. Cimetidine is the only FDA-approved agent for SUP; however, it is rarely used because of its high rate of drug interactions. The H2RAs are also associated with mental status changes in the elderly, which should be considered in this patient with neurologic decits. 16. Answer: B This patients risk factors for SUP (mechanical ventilation, acute renal failure, and hypoperfusion) are no longer present, so SUP should be discontinued. There is no evidence to prove that a PPI is more efcacious than an H2RA for SUP. Several studies have shown that PPIs can maintain a gastric pH of more than 4 for a longer time than H2RAs, but no data showing superior efcacy have been published. If cost is not an issue and there are no contraindications to a PPI or H2RA, agents in either class can be used for SUP when indicated.

2008 American College of Clinical Pharmacy 1-321

Critical Care ANSWERS AND EXPLANATIONS TO SELF-ASSESSMENT QUESTIONS


1. Answer: A This patients arterial blood gas and urine chloride are consistent with a saline-responsive metabolic alkalosis. In critically ill patients, the most common cause of metabolic alkalosis is volume contraction. In this case, the volume contraction is most likely caused by overly aggressive diuresis. In patients receiving diuretics, the urine chloride should be measured at least 1224 hours after the last dose. This patient should receive a normal saline infusion. Hydrochloric acid infusions are typically reserved for more severe alkalosis (pH more than 7.55) that is not responding to conventional therapy. Using HCO3- or increasing the patients tidal volume would be inappropriate, because these actions would likely worsen the alkalosis. 2. Answer: D This patient has severe acute delirium. This medical emergency must be treated with haloperidol and lorazepam because both are additive. Lorazepam provides anxiolysis, and haloperidol provides antidelirium effects. Propofol alone (Answer a) is incorrect, because it will simply mask the patients agitation but will not treat the underlying causes (anxiety and delirium). Answers b and c are incorrect because either agent alone is inferior to the combination of both agents in Answer d. 3. Answer: C Propofol is formulated in a 10% lipid emulsion, which will contribute to the total calories the patient is receiving. In addition, triglycerides should be monitored, especially in a patient with pancreatitis and in patients receiving high doses or prolonged infusions of propofol. Vecuronium is a paralytic and has no sedative properties. Tobramycin and magnesium may prolong paralysis and should be avoided in this patient, if possible. Morphine and propofol can have additive central nervous system effects when used in combination, so routine neurologic assessments need to be conducted, and doses should be titrated accordingly. 4. Answer: C A stable, comfortable, and responsive patient who is currently receiving mechanical ventilation and sedation is a candidate for daily sedation interruption (Answer C). This intervention has been shown to hasten the time to extubation by 2 days compared with conventional sedation in a randomized controlled trial (Kress JP, Pohlman AS, OConnor MF, Hall JB, N Engl J Med 2000;342:14717). No rationale exists to change opioid narcotics (Answer b). Answers a and d represent conventional sedation, which resulted in prolonged duration of mechanical ventilation and length of ICU stay. 5. Answer: C This patient meets the criteria for severe sepsis as dened by the American College of Chest Physicians/ Society of Critical Care Medicine Consensus Guidelines and the PROWESS Trial. This patient would be a good candidate for drotrecogin- therapy. The criteria that qualify her for drotrecogin- include a suspected infection; meeting systemic inammatory response syndrome criteria (temperature more than 38C, SBP less than 90 mm Hg, HR more than 90 beats/minute, and WBC more than 12,000), and having one or more organ system failures (e.g., respiratory, renal, cardiovascular). She is on the appropriate empiric antibiotics; however, this regimen should be reevaluated when culture and sensitivity results are available. Hemodynamic instability is still a problem despite uid resuscitation. The patients APACHE score is more than 25, which is associated with a high mortality rate, but a better response to drotrecogin- treatment. Drotrecogin- therapy has shown benet for up to 48 hours after symptom onset; therefore, treatment should be started immediately in this patient. 6. Answer: C The Surviving Sepsis Campaign guidelines recommend adequate uid resuscitation before the addition of vasopressor agents in patients with severe sepsis. This patients BP, HR, and BUN/Cr ratio indicate that she is severely dehydrated and needs to ll up her tank immediately. Therefore, intravenous uids should be the next therapy added to this patients regimen. 7. Answer: C Dopamine has both -adrenergic and -adrenergic properties, with the best -adrenergic effects achieved at doses of 510 mcg/kg/minute. This can cause an increased oxygen demand on the heart, which would not be benecial in a patient with a past myocardial infarction. Tachycardia is also a side effect of dopamine administration and appears to be causing this patients increase in HR, so increasing the dose will likely cause more tachycardia. The epinephrine dose listed is for cardiopulmonary resuscitation cases only and is therefore incorrect. Vasopressin continuous infusions should be added at doses less than 0.04 units/minute after other vasopressor agents have failed. Higher doses of vasopressin can cause myocardial ischemia and are not recommended. Phenylephrine is a potent

2008 American College of Clinical Pharmacy 1-322

Critical Care
-receptor agonist and is the best choice for the patient at this time. 8. Answer: D The patient has refractory pulseless ventricular tachycardia; therefore, amiodarone should be the next agent given. Atropine is used for bradycardia and pulseless electrical activity, not ventricular tachycardia. It has been only 1 minute since the last dose of epinephrine, so another dose (or a dose of vasopressin) is not recommended for another 2 minutes. The ACLS guidelines recommend a dose of 40 units of intravenous push vasopressin. 9. Answer: D Multiple t-tests are not the answer (Answers a or b), because as the number of groups exceeds two, the number of needed pairwise comparisons also grows. For this study, there are 21 possible pairwise comparisons that could be done; it would be likely to detect a signicant difference (which would happen less than 5% of the time) if we compared 21 pairs of means rather than 1 pair of means. In other words, for 21 pairwise comparisons, a p=0.05 for 1 pair cannot be considered signicant. One would have to adjust the p-value threshold for signicance downward accordingly to account for this, thereby making it much more difcult to achieve statistical signicance. Analysis of variance (Answer d) puts all the data into one number (F) and provides a single p-value for the null hypothesis. Analysis of covariance (Answer c) is used when factors are identied that may explain a statistical difference between groups. These may include a prior history of GI bleeding or nonsteroidal anti-inammatory drug use. 10. Answer: B Mechanical ventilation and coagulopathy are independent risk factors for SRMD. This patient is critically ill and may be intubated for an extended period; therefore, he is at risk for SRMD and requires SUP. The patient currently has an NJ feeding tube, which means that enteral nutrition and medication administered by the tube will go directly into the jejunum. Sucralfate has been shown to be inferior to H2antagonists for the prevention of clinically signicant bleeding from SRMD in a large randomized controlled trial (Cook DJ, et al, N Engl J Med 1998;338:7917) and is generally not recommended for SUP. PPIs, such as intravenous pantoprazole, are reserved as secondline alternatives for patients who are intolerant to H2-antagonists. Thus, famotidine is the best agent for SUP in this patient.

2008 American College of Clinical Pharmacy 1-323

Critical Care

NOTES

2008 American College of Clinical Pharmacy 1-324

Fluids, Electrolytes, and Nutrition

Fluids, Electrolytes, and Nutrition


Gordon S. Sacks, Pharm.D., BCNSP The University of WisconsinMadison. Madison, Wisconsin

2008 American College of Clinical Pharmacy 1-325

Fluids, Electrolytes, and Nutrition


Learning Objectives 1. 2. 3. 4. 5. 6. 7. 8. 9. 10. 11. 12. 13. 14. 15. 16. 17. 18. 19. 20. Understand the importance of the route of nutrient administration on host defense mechanisms in the metabolic response. Understand the potential mechanisms for differences in outcome related to enteral versus parenteral nutrient delivery. Discuss the physiologic processes involved in acid-base disorders. Identify primary and secondary acid-base disorders based on arterial blood gases (ABGs). Determine and use the anion gap for diagnostic purposes. Outline a stepwise approach for interpretation and treatment of acid-base disorders. Understand the importance of the route of nutrient administration on host defense mechanisms in the metabolic response. Understand the potential mechanisms for differences in outcome related to enteral versus parenteral nutrient delivery. Identify the multiple enteral access routes and administration devices that are available. List the complications associated with enteral access devices and administration of EN. Describe the uses of PN in specic diseases and conditions. Compare and contrast central PN and peripheral PN in terms of techniques, advantages, and disadvantages. Describe the adverse effects of excessive carbohydrate administration. Recall the available intravenous protein substrates and state the marketed use for each one. Describe the current recommendations for intravenous fat administration and discuss recent advances in intravenous fat products. Discuss the advantages and disadvantages associated with total nutrient admixtures (TNAs). Identify stability issues associated with calcium and phosphorus compatibility. Be familiar with the complications associated with PN therapy. Describe the homeostatic mechanisms responsible for sodium and water balance. Discuss the most common etiologies and list the signs and symptoms of hypo/hypernatremia, hypo/hypermagnesemia, and hypo/ hyperphosphatemia. Develop a treatment plan for the management of common electrolyte disorders in patients receiving nutritional support. Self-Assessment Questions: Answers to these questions may be found at the end of this chapter. 1. A 2-year-old, 13-kg boy with a history of Hirschsprungs disease is postoperative day 2 s/p a small bowel resection. His nasogastric tube is connected to low intermittent suction and is draining copious amounts of green uid. Urine output has decreased to 0.3 mL/kg/hour despite receiving maintenance intravenous uids of dextrose 5%/0.2% normal saline. Laboratory values and ABGs on room air are as follows: sodium, 144 mEq/L; potassium, 3.2 mEq/L; chloride, 94 mEq/L; pH 7.52; PO2, 90 mm Hg; PCO2, 48; and HCO3, 39. What acid-base disorder is present? A. Compensated metabolic alkalemia. B. Uncompensated metabolic alkalemia. C. Respiratory alkalemia. D. Respiratory alkalemia with compensated metabolic acidemia. A 9-year-old girl with a history of Crohns disease is admitted to the hospital with a perforated bowel and peritonitis. After undergoing surgery for a colectomy and peritoneal irrigation, she is transferred to the intensive care unit. Postoperatively, she becomes septic with a temperature of 103F. ABGs and laboratory values on arrival to the intensive care unit are as follows: pH, 7.12; PCO2, 19 mm Hg; HCO3, 5 mEq/L; sodium, 140 mEq/L; potassium, 4.8 mEq/L; and chloride, 105 mEq/L. What is acidbase disorder is present? A. Metabolic acidemia. B. Compensated respiratory acidemia. C. Combined respiratory and metabolic acidemia. D. Metabolic acidemia with respiratory alkalemia. A 5-year old, 25-kg boy sustained severe head injuries, a grade IV splenic laceration, a tibiabula fracture, and a pulmonary contusion after being hit by a car while riding his bicycle. He is stabilized and transferred to the intensive care unit with the following laboratory values and ABGs on admission: sodium, 135 mEq/L; potassium, 3.1 mEq/L; chloride, 103 mEq/L; pH, 7.51; PCO2, 25 mm Hg; and HCO3, 22 mEq/L. What acid-base disorder is present? A. Compensated respiratory alkalemia. B. Uncompensated respiratory alkalemia.

2.

3.

21.

2008 American College of Clinical Pharmacy 1-326

Fluids, Electrolytes, and Nutrition


C. Metabolic alkalemia with compensated respiratory acidemia. D. Combined metabolic and respiratory alkalemia. 4. Which one of the following is the best treatment strategy for correcting hyponatremia in a patient with syndrome of inappropriate section of antidiuretic hormone (SIADH)? A. Intravenous D5W. B. Intravenous D51/4NS. C. Vasopressin administration. D. Free water restriction. Which one of the following situations could be responsible for a persistent hypokalemia refractory to potassium supplementation? A. Hyponatremia. B. Hypophosphatemia. C. Hypomagnesemia. D. Hypoaldosteronism. Which one of the following has been associated with hypomagnesemia? A. Amphotericin B. B. Severe thermal injury. C. Alcoholism. D. GI uid loss. Too aggressive phosphate repletion therapy may result in: A. Hypophosphatemia rebound. B. Hypocalcemia. C. Metastatic calcication. D. B and C. Which one of the following best describes the concept of bacterial translocation? A. Infection caused by touch contamination. B. Bacterial uptake of iron from the circulation. C. Peristaltic movement of microorganisms along the intestinal tract. D. Passage of microorganisms from the intestinal tract across the mucosa into the systemic circulation. When compared with patients receiving PN, early enteral feeding after major trauma is associated with: A. Greater caloric intake. B. Decreased mortality. C. Decreased days of antibiotics. D. Fewer postoperative infections. 10. What is the most effective method of alimentation for decreasing gut mucosal atrophy? A. Central PN. B. PPN enriched with branch chain amino acids. C. EN. D. Pharmacological doses of zinc. 11. Aggressive nutrition support is most effective in reducing lean tissue breakdown in which one of the following disease states? A. Sepsis. B. Starvation. C. Multiple trauma. D. Head injury. 12. Which one of the following immunoglobulins found in intestinal secretions prevents luminal pathogens from attaching to and invading intestinal epithelial cells? A. IgA. B. IgE. C. IgG. D. IgM.

5.

6.

7.

8.

9.

2008 American College of Clinical Pharmacy 1-327

Fluids, Electrolytes, and Nutrition I. ABCs OF ACID-BASE BALANCE A. The Basics 1. Denitions a. Acid: a substance that can donate hydrogen ions (H+) H2CO3 (acid) H+ + HCO3-

b. Base: a substance that can accept H+ HCO3(base) + H+ H2CO3

c. Acidemia refers to an acid condition of the blood, pH less than 7.35, whereas acidosis is the process in the patient that causes acidemia. d. Likewise, alkalosis refers to the process in the patient that causes the alkalemia, pH more than 7.45. e. Primary disorder: a pathologic change in the respiratory component or the metabolic component, resulting in an acidosis or an alkalosis f. Correction: normalization of the component involved in a primary disorder g. Compensation: change in pH toward normal by the component not involved in the primary disorder. For every addition of acid the body receives, the body attempts to bring the pH back toward normal by an addition of a base. For example, a respiratory alkalosis can compensate for a primary metabolic acidosis. h. Buffers: agents that decrease the ability of acids and bases added to a solution to alter pH 2. Table 1. Normal Blood Gas Values
pH PO2 PCO2 HCO3O2 saturation Base excess Arterial Blood 7.40 (7.357.45) 80100 mm Hg 3545 mm Hg 2226 mEq/L 95% 2 to +2 Venous Blood 7.36 (7.337.43) 3540 mm Hg 4151 mm Hg 2428 mEq/L 70%75% 0 to +4

a. Blood gases are obtained for two major reasons: i. To determine the oxygenation of a patient ii. To determine the acid-base status of a patient b. Types of blood gases: i. Arterial (ABGs): mixture of blood from various parts of the body; characterizes how well lungs are oxygenating the blood ii. Venous: represents tissue oxygenation; identies possible heart and circulation failure c. Components of ABGs i. pH = identies the presence of acidemia or alkalemia ii. PO2 = pressure exerted by oxygen dissolved in the plasma iii. PCO2 = pressure of dissolved CO2 gas in the blood iv. O2 saturation = percentage of oxygen that hemoglobin is carrying related to the total amount the hemoglobin could carry v. Base excess = primarily reects the concentration of bicarbonate and is affected only by metabolic processes; positive values reect metabolic alkalosis, and negative values reect metabolic acidosis

2008 American College of Clinical Pharmacy 1-328

Fluids, Electrolytes, and Nutrition 3. Buffer System a. Optimal function of the human body depends on the maintenance of the H+ concentration within a relatively narrow range: pH range of 7.107.60. b. The body counters excessive shifts in pH by three mechanisms: i. Chemical buffering: bicarbonate (HCO3-), hemoglobin, proteins, phosphates ii. Respiratory compensation: PCO2 iii. Renal compensation: HCO3c. PCO2 (respiratory component) i. Inuenced ONLY BY THE LUNGS ii. Considered an ACIDIC substance iii. Breathing is regulated to match CO2 production (about 18,000 mmol/day) and to maintain a constant PCO2. (a) Increased PCO2 = hypoventilation or ventilation too low for CO2 production rate (b) Decreased PCO2 = hyperventilation or ventilation too high for CO2 production rate iv. Ventilation rate can change PCO2 and pH of the blood, and this can happen very quickly (i.e., with a few breaths) in a matter of minutes. Maximal compensation is achieved within 1224 hours. d. HCO3- (metabolic component) i. Inuenced ONLY BY METABOLIC PROCESSES, i.e., the kidneys ii. Considered a BASIC substance iii. Kidneys can affect blood pH by reabsorbing HCO3-, by allowing excess HCO3- to be excreted in the urine, and by generating new HCO3- by acid secretion. iv. Kidneys also excrete nonvolatile (titratable) acids that are metabolic end products. v. Thus, kidneys contribute to acid-base homeostasis by excreting nonvolatile acids and by controlling HCO3- concentration in the extracellular uid (ECF) compartment. *Note: CO2 is reported on serum electrolytes in milliequivalents per liter and represents total CO2, which is a combination of HCO3- and dissolved CO2/H2CO3 in the blood; because CO2 and H2CO3 are only a small fraction of total CO2 in a serum sample, total CO2 is considered equal to HCO3. Do not confuse PCO2 (expressed in mm Hg) in the ABG report with CO2 (expressed in millimeters of mercury) in the serum electrolytes report. vi. Metabolic compensation through the kidneys is slow, requiring several hours to change ECF HCO3- and 57 days to achieve maximal compensation. B. Stepwise Approach to Acid-Base Disorders 1. Use the ABG to determine the presence of acidemia or alkalemia. a. pH less than 7.35 = acidemia b. pH more than 7.45 = alkalemia 2. Determine whether the primary process is respiratory or metabolic. a. Abnormal PCO2 (less than 35 mm Hg or more than 45 mm Hg): respiratory b. Abnormal HCO3- (less than 22 mEq/L or more than 26 mEq/L): metabolic 3. Calculate the gaps: a. Law of electrical neutrality: number of cations in serum must equal the number of anions. i. Cl- plus HCO3- plus unmeasured anions = Na+ plus unmeasured cations ii. Unmeasured anions = phosphates, sulfates, organic anions, proteins iii. Unmeasured cations = Ca+2, K+, Mg+2

2008 American College of Clinical Pharmacy 1-329

Fluids, Electrolytes, and Nutrition b. Anion gap = Na+ minus (HCO3- plus Cl-) i. Normal anion gap = 10 plus or minus 4 mEq/L ii. Increased anion gap (more than 14 mEq/L) can indicate metabolic acidosis. iii. Increased anion gap (more than 25 mEq/L) always indicates metabolic acidosis. 4. Check for the degree of compensation. a. Compensation in metabolic acidemia i. Eyeball method: decrease in PCO2 = last two digits of the pH ii. Formula method: decrease in PCO2 = 1.2 decrease in HCO3b. Compensation in metabolic alkalemia i. Formula method: increase in PCO2 = 0.6 increase in HCO3c. Compensation for respiratory acidemia i. Acute: change in PCO2 = 0.1 change in HCO3ii. Chronic: change in PCO2 = 0.4 change in HCO3d. Compensation for respiratory alkalemia i. Acute: change in PCO2 = 0.2 change in HCO3ii. Chronic: change in PCO2 = 0.5 change in HCO35. Determine whether there is a 1:1 relationship between anions in the blood. a. Increased anion gap metabolic acidosis: every one-point increase in anion gap should be accompanied by a 1-mEq/L decrease in HCO3i. Decline in HCO3- of less than the increase in the anion gap suggests an underlying metabolic alkalemia (i.e., complex acid-base disorder) b. Normal anion gap metabolic acidosis: every 1-mEq/L increase in chloride should be accompanied by a 1-mEq/L decrease in HCO3i. Decline in HCO3- of less than the increase in Cl-: an additional primary metabolic alkalemia is present C. Etiologies and Management of Acid-Base Disorders 1. Metabolic Acidosis a. Gain of strong acid or loss of base from ECF, resulting in decreased HCO3-and decreased BE less than 2.5 mEq/L b. Respiratory compensation: onset is rapid, and maximal compensation averages about 1.2 mm Hg PCO2 for every 1-mEq/L decrease in HCO3c. Correction mechanisms: i. Increased renal acid secretion (decreased urine pH) ii. Metabolism of organic acids (a) Lactic acid is buffered by HCO3- and therefore decreasing HCO3(b) Lactic acid is metabolized to pyruvic acid and then acetyl coenzyme A and then to CO2 and H2O by the tricarboxylic acid cycle. (c) For each lactic acid metabolized, an HCO3- is recovered as the buffer equation shifts: Lact- plus H2CO3 HCO3- plus HLact CO2 plus H2O

This is why salts of metabolizable organic acids can be used therapeutically as bicarbonate precursors.

2008 American College of Clinical Pharmacy 1-330

Fluids, Electrolytes, and Nutrition d. Clinical causes of metabolic acidosis with normal anion gap i. GI losses of HCO3-: diarrhea, enteric or pancreatic stulae ii. Ileal or colonic conduit (urinary diversions): these segments secrete Na+ and HCO3while reabsorbing NH3, NH4+, H+, and Cl- when exposed to urine iii. Renal tubular acidosis iv. Drugs: acetazolamide, amphotericin B, cholestyramine v. Urinary anion gap may be helpful diagnostically. (a) Urine anion gap = Na+ plus K+ minus Cl(b) Normal urine anion gap = 20 to 0 mEq/L (c) NH4+ is the predominant unmeasured cation, and its excretion is usually accompanied by Cl-. Under normal circumstances, 2040 mEq/L of NH4+ is excreted each day, and the urine anion gap has a negative value (20 to 0 mEq/L). The normal response to an acid load is an increase in renal generation of ammonia, with an increase in urine NH4+ excretion. (d) In metabolic acidosis, NH4+ excretion should increase dramatically if acidication is intact, resulting in a large negative urine anion gap (20 to 50 mEq/L). (e) If a defect in renal acidication is present (e.g., renal tubular acidosis), NH4+ excretion is impaired, and the urine anion gap is positive. e. Clinical causes of metabolic acidosis with an increased anion gap. Think of the mnemonic A MUDPIE i. Aspirin ii. Methanol iii. Uremic renal failure (kidney disease) iv. Diabetic ketoacidosis and other forms of ketoacidosis v. Paraldehyde vi. Ischemic or idiopathic lactic acidosis vii. Ethylene glycol f. Management of patients with metabolic acidosis i. Identify and treat cause of acidosis Examples: (a) GI losses: treat diarrhea or stula (b) Ketoacidosis: initiate insulin drip if diabetic ketoacidosis (c) Lactic acidosis: treat underlying cause (e.g., shock) ii. Circumstances that require acute management of acidemia (a) Cardiovascular depression, hyperkalemia (b) Sustained pH less than 6.9 incompatible with life; pH less than 7.2 with hypotension is usually treated with alkalinizing agents g. Medications used to treat metabolic acidosis i. NaHCO3: problems associated with its use as a treatment of acidemia include: (a) CO2 generation (b) Overshoot alkalosis (c) Overcompensation (d) Na+ load ii. Dosing guidelines for NaHCO3 (a) If the base excess is decreased but the anion gap is normal, it usually represents a true HCO3- loss. (b) To estimate the HCO3- decit, multiply the base excess (expressed as milliequivalents per liter) the apparent volume of distribution (0.30.5 L/kg). (c) If there is an increased anion gap, it usually means that the low HCO3- is due to buffering rather than depletion, and the buffer may be recovered if the acid is

2008 American College of Clinical Pharmacy 1-331

Fluids, Electrolytes, and Nutrition metabolizable; thus, NaHCO3 therapy is not indicated because there is a greater risk of complications associated with alkali therapy. iii. If the patient is receiving total PN (TPN), use acetate salts (e.g., NaAc, KAc). iv. Choices for oral bicarbonate supplements: (a) NaHCO3 tablets: 1 g is about 12 mEq; some patients dislike taste; baking soda is an alternative to tablets (b) Shohls solution: sodium citrate plus citric acid, equivalent to 1 mEq of NaHCO3/ mL of solution; preferred to NaHCO3 tablets by some patients; citrate may increase gastrointestinal (GI) absorption of Al+3, which can be a problem for patients with renal failure (c) Potassium bicarbonate, acetate, and citrate: good choices for patients that require supplements of K as well as bicarbonate but must be avoided in patients with renal failure 2. Metabolic Alkalosis a. An increase in ECF HCO3-, with a BE greater than +2.5; caused by a gain of base or a loss of H+ from ECF b. Compensation mechanism is hypoventilation (increased PCO2) i. Respiratory compensation is rapid. ii. Maximal compensation is extremely variable among patients and is limited by the necessity of maintaining adequate ventilation for gas exchange; the average compensation is about a 0.5-mm Hg increase in PCO2 for each milliliter per liter rise in HCO3. The maximum PCO2 will rarely exceed 5055 mm Hg unless lung disease or another cause of ventilatory depression is present. c. Correction mechanism i. Excess bicarbonate can be excreted by the kidneys under normal conditions. ii. The ability of kidneys to excrete excess bicarbonate is compromised by decreased renal perfusion, inadequate availability of Cl- or K+, and mineralocorticoid activity. d. Clinical causes of metabolic alkalosis i. Usually a combination of an initiating acid loss/base gain together with a condition that compromises the kidneys ability to excrete the excess bicarbonate ii. If the kidneys are normal, it is usually possible to anticipate and prevent the development of metabolic alkalosis by providing intravenous uid maintenance that ensures good renal perfusion and adequate availability of K+ and Cl-. iii. ECF/Cl- depletion: loss of gastric juice, diuretic therapy, posthypercapnea iv. Mineralocorticoid excess: hyperaldosteronism, Cushings disease v. Hypokalemia: increases renal tubular bicarbonate generation and the maximal reabsorptive capacity for bicarbonate; note that potassium depletion commonly is present in metabolic alkalosis. e. Situations that require acute management of alkalemia: i. A pH of more than 7.55 is of concern, and a pH of more than 7.6 would require urgent treatment. ii. May cause tetany, muscle weakness, ileus, arrhythmias because of decreased ionized calcium, and intracellular shifts of K+, Mg+2, and PO4-3 f. Management of metabolic alkalosis. Think of the mnemonic CRAP. i. If adequate renal function, maximize the patients ability to excrete excess bicarbonate by correcting ECF volume depletion with NaCl and uid to improve Renal perfusion and minimize Aldosterone production, and supplement Potassium chloride. ii. If alkalemia is caused by gastric losses, the patient is usually ECF volume depleted,

2008 American College of Clinical Pharmacy 1-332

Fluids, Electrolytes, and Nutrition chloride depleted, and K+ depleted; H2-antagonists or proton pump inhibitors are useful to decrease gastric acid losses If alkalemia is diuretic induced, K+ and ECF volumes are often depleted; K+-sparing diuretic and NaCl may be helpful; also, temporarily stop diuresis and/or reduce dose If alkalemia is due to mineralocorticoid excess and patient is on high-dose steroid, change to steroid with lower mineralocorticoid activity (i.e., dexamethasone, methylprednisolone). Acetazolamide: 250- to 500-mg single dose will promote bicarbonate diuresis if adequate renal perfusion/renal function, adequate K+ and ClAcid therapy for severe alkalosis (i.e., HCl) (a) HCl is not commercially available and must be extemporaneously compounded by a pharmacist (b) Mix concentrated HCl with intravenous uids to make 0.1 N HCl in 5% dextrose as the nal solution and administer through the central vein only. If the patient is receiving TPN, use chloride salts (e.g., NaCl, KCl), and add H2antagonist to the TPN.

iii. iv. v. vi.

vii.

3. Respiratory Acidosis a. Increased PCO2 due to inadequate ventilation relative to CO2 production (hypoventilation) b. Compensation mechanisms: i. Small immediate increase in HCO3-from plasma buffers (slight decrease or no change in BE) ii. Increased renal reabsorption of HCO3- (increased BE) (a) Slow onset of 618 hours, maximal in 57 days (b) pH restored to close to normal range if hypoventilation continues for several days c. Clinical causes for respiratory acidosis: i. Decreased central nervous system (a) Central nervous system disease (e.g., injury, neoplasm, infection) (b) Drugs/Poison (e.g., opiates, alcohols, anesthetics) (c) Metabolic (e.g., anoxia) ii. Mechanical (a) Airway obstruction (e.g., severe asthma, foreign body, tumor) (b) Pneumothoraces iii. Neuromuscular (a) Spinal injury (b) Paralyzing drugs (c) Muscular dystrophy (d) Guillain-Barre, acute lateral sclerosis, multiple sclerosis iv. Loss of gas exchange area (a) Severe pneumonia (b) Severe pulmonary edema (c) Emphysema (d) Massive pulmonary embolus d. Situations that require acute management of acidosis i. pH values less than 7.2 may affect cardiovascular function. ii. Decreased O2 because of hypoventilation e. Management of respiratory acidosis i. Treat underlying cause; in chronic obstructive pulmonary disease, keep PO2 more

2008 American College of Clinical Pharmacy 1-333

Fluids, Electrolytes, and Nutrition than 60 mm Hg with cautious oxygen supplementation because it may exacerbate hypoventilation further in a patient who is chronically hypoxic and hypercapneic. ii. Drugs to improve ventilation (a) Naloxone: effective for opiate-induced respiratory depression (b) Cholinesterase inhibitors (e.g., neostigmine): effective for partial reversal of paralyzing drugs or myasthenia gravis (c) Flumazenil: effective for benzodiazepine-induced respiratory depression (d) Mechanical ventilation: necessary if underlying cause is not reversed quickly and patient is unable to maintain PO2 **NOTE: Bicarbonate therapy is not indicated and may depress ventilation further. 4. Respiratory Alkalosis a. Decreased PCO2 due to ventilation in excess of CO2 production (hyperventilation) b. Compensation mechanisms i. Small decrease in HCO3- immediately (with no change in BE) due to plasma buffers ii. Small increase in serum lactic acid iii. Increased renal excretion of HCO3(a) Slow onset of 618 hours with maximal effect in 57 days (b) Compensation very effective in chronic hyperventilation, but most cases are not chronic c. Clinical causes for respiratory alkalosis: i. Psychogenic hyperventilation: anxiety ii. Hypoxia: severe asthma, pneumonia, pulmonary embolus iii. Chronic heart failure/pulmonary edema iv. Fever v. Gram () sepsis vi. Increased intracranial pressure: head trauma, stroke/hemorrhage, tumor vii. Salicylate toxicity viii. Liver disease ix. Excessive mechanical ventilation d. Situations that require acute management of alkalosis: i. Symptoms of paresthesia, muscle spasms, seizures due to intracellular shifts of Ca+2, K+, PO4-3, Mg+2 ii. Hyperventilation can be caused by serious underlying conditions that are more urgent than hyperventilation itself. e. Management of respiratory alkalosis i. Treat underlying cause. ii. Psychogenic hyperventilation: training to control breathing or paper bag breathing (rebreathing exhaled CO2 prevents large decreases in blood PCO2) iii. Mechanically ventilated patients: may be necessary to suppress patient-triggered breaths, using opiates/sedatives; paralyzing drugs in some cases

II. ADVANTAGES OF EN A. Specialized Nutrition Support (JPEN 1995;19:12) Provision of specially formulated and/or delivered parenteral or enteral nutrients to maintain or restore optimal nutrition status

2008 American College of Clinical Pharmacy 1-334

Fluids, Electrolytes, and Nutrition B. Nutritionally at Risk (JPEN 1995;19:12) 1. Involuntary weight loss or gain of 10% or more of usual body weight (BW) within 6 months or 5% or more of usual BW in 1 month or 20% or more of ideal BW (IBW) 2. Inadequate nutrition intake, including not receiving food or nutrition products (impaired ability to ingest or absorb food adequately) for more than 7 days C. General Practice Guidelines for EN 1. Candidates for EN are patients at risk for malnutrition in whom oral feedings are inadequate to maintain their nutritional status. 2. Enteral access should be obtained in critically ill patients whenever possible, either at the time of surgery with direct enteral access or with nasoenteric feeding tubes. 3. Parenteral feedings should be administered ONLY when enteral access cannot be obtained or when feeding into the GI tract is contraindicated. D. EN Administration Routes and Techniques 1. Route a. Orogastric tubespreferred with nasal/facial trauma, sinusitis i. Advantages: lower incidence of sinusitis compared with nasoenteric tubes ii. Disadvantages: not tolerated for prolonged time in alert patients iii. Complications: esophageal perforation, pneumothorax, gastrointestinal (GI) bleeding, otitis media, pulmonary aspiration b. Nasally placed tubefor short-term feeding (less than 3 weeks) i. Nasogastricmost common tube used; may perform multiple functions in addition to feeding such as decompression of stomach, administration of medications, measurement of gastric pH (a) Advantages: available in a variety of sizes and tube materials, easily placed, permit bolus feeding and medication administration (b) Disadvantages: contraindicated in patients with nasal/facial trauma, intrinsic problems of the esophagus (e.g., masses, strictures), increased risk of complications such as GI bleeding, perforation, otitis media, sinusitis, nasal mucosal ulceration ii. Nasoduodenalif the tip is past the pyloric sphincter, the risk of EN-associated aspiration is reduced (a) Advantages: effective for patients with gastroparesis; usually, tubes are of smaller diameter and thus cause less patient discomfort (b) Disadvantages: small diameter may preclude administration of some medications, usually requires infusion pump for administration, precludes bolus feedings (c) Complications similar to that of nasogastric tube except decreased risk of aspiration iii. Nasojejunaldened as tip of tube in fourth portion of the duodenum or past the ligament of Treitz (a) Advantages: associated with the lowest risk of pulmonary aspiration, allows early postoperative feeding (b) Disadvantages: increased risk of tube occlusion, precludes early postoperative feeding in hemodynamically unstable patients c. Tube enterostomy i. Gastrostomy (a) Stamm gastrostomyfor patients requiring feeding for more than 4 weeks; surgically placed tube under general anesthesia, easily cared for and allows bolus as well as continuous feeds. Disadvantages are that these tubes are more

2008 American College of Clinical Pharmacy 1-335

Fluids, Electrolytes, and Nutrition invasive than the nasal route, with possible complications of dislodgment, wound infection, pneumoperitoneum, and stomal leakage. (b) Percutaneous endoscopic gastrostomyfor patients requiring feeding for more than 4 weeks; can be placed under conscious sedation, is easily cared for, and is replaceable. Disadvantages similar to Stamm gastrostomy ii. Jejunostomy (a) Needle catheter jejunostomysmall-caliber tubes that are placed during a laparotomy for only 68 weeks. These tubes have a low risk of enteric leakage if inadvertently dislodged but are prone to occlusion if not ushed often after medication administration (b) Percutaneous endoscopic jejunostomysimilar to needle catheter jejunostomy except that tubes are placed endoscopically into the small intestine. Associated with decreased risk of pulmonary aspiration but are difcult to replace compared with percutaneous endoscopic gastrostomy (c) Permanent jejunostomysimilar to needle catheter jejunostomy except that the tubes are of larger diameter and used for prolonged duration. Complications similar to Stamm gastrostomy and percutaneous endoscopic gastrostomy 2. Delivery methods a. Gravity controlrefers to delivery with tubing that is tted with a roller clamp to allow infusion for several minutes to hours based on patient tolerance i. Used for gastric feedings ii. Indicated only for neurologically intact patients because of risk of aspiration iii. Most common use in home care or long-term care environments b. Pump-assisted continuous infusion is indicated in the presence of volume sensitivity, small bowel feedings, and delayed gastric motility. i. Assists with decreased GI complications associated with rapid infusion such as nausea, cramping, and diarrhea ii. Usually delivered by a stationary or ambulatory pump without interruption c. Intermittent feeding is a delivery method in which predetermined volumes are administered for a specied period several times daily (e.g., 100300 mL for 3060 minutes every 46 hours). This method is considered more physiologic than the continuous infusion method but may be more appropriate for patients with uid balance or tolerance issues. d. Bolus feedings refer to the delivery of a predetermined volume of feeding at specied intervals by gravity or syringe for a short time. i. This delivery method is simple, inexpensive, and the most physiologic, and it provides the patient with freedom or breaks from feeding. ii. Does not require an infusion pump because but is administered by gravity or a syringe iii. Disadvantages include an increased risk of aspiration, volume intolerance, emesis, diarrhea, and delayed gastric emptying E. Selection of Route of Delivery 1. Enteral Nutrition (EN) Versus Parenteral Nutrition (PN) Support: Clinical Studies a. Alexander et al. Ann Surg 1980;192:50517. i. Eighteen pediatric thermal injury patients (around 60% total body surface area) (a) Control group: 16.5% of total kilocalories from protein (b) High-protein group: 23% of total kilocalories from protein ii. Results: control group displayed a signicantly lower opsonic index, lower levels of C3, lower levels of transferrin, more bacteremic days, and worse survival

2008 American College of Clinical Pharmacy 1-336

Fluids, Electrolytes, and Nutrition iii. Control group received 2 times as much of their nutrients by PN. b. Moore EE et al. J Trauma 1986;26:87481. i. Seventy-ve trauma patients abdominal trauma index (ATI) more than 15) (a) Control group: 5% dextrose in distilled water (D5W) for rst 5 days and then PN (b) EN: Vivonex TEN by needle catheter jejunostomy within 18 hours ii. Results: septic complications were 29% in control versus 9% in EN group iii. Criticism: comparing fed versus non-fed rather than EN versus PN c. Moore FA et al. J Trauma 1989;29:91623. i. Seventy-ve trauma patients (ATI more than 15 and less than 40) (a) TPN group: within 12 hours of injury by central line (b) EN group: within 12 hours of injury by needle catheter jejunostomy ii. Results: EN group exhibited a signicantly lower incidence of major infections (e.g., abdominal abscesses, pneumonia) compared with PN group (3% vs. 20%, p<0.05). d. Kudsk et al. Ann Surg 1992;215:50313. i. 98 trauma with ATI at least 15, even if reop within 72 hours, ATI more than 40, pelvic fracture with more than 6 units of blood loss, or blood loss of more than 25 units within the rst 24 hours (a) EN: within 24 hours by needle catheter jejunostomy (b) TPN: within 24 hours by central line ii. Resultsseptic morbidity (a) There were no differences in infection rates in patients with injury severity scores less than 20 or ATI 24 or less. (b) Signicantly fewer infections in EN patients with an injury severity score more than 20 (p<0.002) and an ATI more than 24 (p<0.005) versus PN patients (c) EN patients developed signicantly fewer pneumonias (11.8% vs. 31%, p<0.02), abscesses (1.9% vs. 13.3%, p<0.04), and episodes of line sepsis (1.9% vs. 13.3%, p<0.05) than PN patients. (d) Evaluating outcome by organs injured, signicantly fewer infection rates were noted in EN patients with pancreas or liver injuries than in PN patients. Potential mechanisms for host defense related to nutrition a. Mucosal thickness and permeability i. Mucosal atrophy with absence of EN ii. Bacterial translocation b. Gut-associated lymphoid tissue i. Peyers patches and mesenteric lymphoid cells secretory immunoglobulin A (sIgA) ii. sIgA with PN; sIgA with EN c. Metabolic effects i. Delayed EN resting energy expenditure, counterregulatory hormones, catecholamines Early ENhow early is early? a. It appears that initiation of EN in hemodynamically stable patients within 2448 hours of injury is desirable. b. Seventy-two hours may or may not be too late to see an appreciable effect. Early EN effects on intestinal permeability a. Increased intestinal permeability with EN started 24 hours after admission b. More severe form of multiple organ dysfunction syndrome if started on EN 24 hours after admission Summary of Advantages of EN over PN a. Ease and safety of administration

2.

3.

4.

5.

2008 American College of Clinical Pharmacy 1-337

Fluids, Electrolytes, and Nutrition b. More economical c. More physiologic i. Maintenance of GI integrity ii. Bacterial translocation (?) iii. Septic complications 6. (Relative) Contraindications to EN support a. Intestinal obstruction b. GI stulas c. Severe diarrhea d. Intractable vomiting e. Hemodynamic instability F. Composition and Characteristics of Enteral Formulations 1. Carbohydrate a. Common carbohydrate sources include maltodextrin, modied cornstarch, and corn syrup. b. Maltodextrin is the most complex carbohydrate source; it is easily digested and has a low osmotic contribution to the formulation. c. Modied cornstarch is generally used to reduce the osmolality of high osmolar enteral formulations. d. Carbohydrate content of enteral formulations may vary from 30% to 90% depending on the condition for which the product is intended. 2. Fat a. Common sources of fat include vegetable oils (corn, canola, soybean, sunower, and safower). b. Long-chain triglycerides contributed by vegetable oils are the source of essential fatty acids such as linoleic acid and -linolenic acid. c. Medium-chain triglycerides, 612 carbons in length, have advantages over long-chain triglycerides because bile salts or pancreatic lipase is not required for absorption. d. Fat content of enteral formulations may vary from 1% to 55% based on the formulations intended use. e. Adult formulations developed for pulmonary disease or glucose intolerance have high-fat contents, whereas products designed for malabsorption disorders are low in total fat and long-chain triglycerides. 3. Protein a. Commonly used intact protein or polymeric protein sources include cow milk protein, whey protein isolates, caseinates, soy protein isolates, milk protein concentrate, and beef. b. Protein sources that have undergone enzymatic hydrolysis may be referred to as oligomeric, semielemental, or peptide formulations. Examples of these protein sources include hydrolyzed casein, whey, lactalbumin, wheat, soy, and meat protein. c. Free amino acids are often used in enteral formulations intended for specic populations to avoid allergens. Monomeric or elemental formulations usually contain free amino acids. d. Specialty amino acids, such as branched-chain amino acids, glutamine, and arginine, may be added to enteral formulations. Branched-chain amino acids have been promoted for use in liver disorders, whereas glutamine and arginine have been denoted as having immunomodulating properties in critically ill patients. 4. Fiber a. Categories include insoluble and soluble bers. b. Insoluble ber is the most prevalent type of ber and includes soy polysaccharide. Uses include constipation prevention, increased stool volume, and decreased GI transit time.

2008 American College of Clinical Pharmacy 1-338

Fluids, Electrolytes, and Nutrition c. Soluble ber includes gum arabic, guar gum, and pectin. Uses include improvement of glucose tolerance, reduction of serum cholesterol, and maintenance of colonic mucosal surfaces. d. Fructooligosaccharides are naturally occurring nondigestible sugars that are used by bidobacteria in the GI tract. e. In the uid-restricted patient, ber-containing formulations may be associated with abdominal distention, gas, cramping, or even impaction. 5. Water content a. Enteral formulations that have a concentration of 1 kcal/mL are about 75%85% water. b. Enteral formulations that have a concentration of 2 kcal/mL are about 70% water. G. Categories of EN Products 1. Polymeric, nutritionally complete tube feeding a. Use: patients with normal digestive processes and normal organ function b. Caloric density: 1 kcal/mL c. Examples: Osmolite, Isocal 2. Polymeric, nutritionally complete concentrated tube feeding a. Use: patients with normal digestive processes who need uid restriction b. Caloric density: 2 kcal/mL c. Examples: Magnacal, TwoCal HN, Deliver 2.0 3. Polymeric, nutritionally complete oral supplement a. Use: to supplement a patients oral diet b. Caloric density: 11.5 kcal/mL c. Examples: Boost, Boost Plus, Ensure 4. Chemically dened, nutritionally complete tube feeding (elemental) a. Use: patients with malabsorption (short bowel, pancreatic insufciency) b. Caloric density: 1 kcal/mL c. Examples: Subdue, Peptamen, Vital HN, Vivonex TEN 5. Fiber-containing, nutritionally complete tube feeding a. Use: patients with either constipation or diarrhea/long-term tube feeding b. Caloric density: 11.5 kcal/mL c. Examples: Boost w/ber, Ultracal, Fibersource, Jevity, Promote w/ber, Protain XL 6. Peptide-based, nutritionally complete tube feeding a. Use: patients with malabsorption b. Caloric density: 11.2 kcal/mL c. Examples: Reabilan HN, Sandosource Peptide, Peptamen 7. Disease-specic formulas (several are nutritionally incomplete) a. Renal failure i. Use: acute renal failure without dialysis (low protein, low electrolytes) Caloric density: 2 kcal/mL Examples: Suplena, Renalcal Diet ii. Use: acute renal failure with dialysis (more protein, moderate electrolytes) Caloric density: 2 kcal/mL Examples: Magnacal Renal, Nepro, NovaSource Renal b. Hepatic failure (increased branched-chain amino acids, decreased aromatic amino acids) i. Use: liver failure with hepatic encephalopathy ii. Caloric density: 1.11.5 kcal/mL iii. Examples: NutriHep c. Pulmonary failure (increased fat, decreased carbohydrate) i. Use: patients with chronic carbon dioxide retention ii. Caloric density: 1.5 kcal/mL iii. Examples: Respalor, NovaSource Pulmonary, NutriVent, Pulmocare

2008 American College of Clinical Pharmacy 1-339

Fluids, Electrolytes, and Nutrition Oxepa (for patients with acute respiratory distress syndrome) d. Diabetes mellitus (increased fat, decreased carbohydrate, added ber) i. Use: glucose-intolerant patients ii. Caloric density: 11.06 kcal/mL iii. Examples: Choice DM, DiabetiSource, Glucerna, Glytrol, Resource Diabetic e. Immunocompromised states (enhanced with arginine, glutamine, omega-3 fatty acids, nucleotides, beta carotene) i. Use: stressed, trauma, burn patients ii. Caloric density: 1.3 kcal/mL iii. Examples: Impact, Impact w/glutamine, ImmunAid, Perative, Crucial f. Stress states (enhanced branched-chain amino acids, increased protein, or both) i. Use: stressed patients (burn, trauma, infection) ii. Caloric density: 11.2 kcal/mL iii Examples: TraumaCal, Protain XL, IsoSource VHN, Promote w/ber, Replete w/ber H. Complications of EN Support 1. Mechanical a. Displaced feeding tubes b. Clogged feeding tubesmost efcacious strategy to restore tube patency is to use a combination of pancreatic enzymes plus sodium bicarbonate 2. Pulmonary a. Aspiration of enteral feeding formula can result in signicant morbidity such as pneumonia and prolonged mechanical ventilation. b. Raising the head of the bed (around 30 degrees) is still the current practice for prevention of this problem. 3. Gastrointestinal a. Elevated gastric residuals b. Diarrhea iii. Hypoalbuminemiaminor cause iv. Hyperosmolar formulasminor cause v. Pharmacotherapymajor cause (primarily sorbitol-containing elixirs) 4. Metabolic a. Dehydration b. Electrolyte disordersvirtually all of them can occur I. Monitoring Guidelines for EN Support 1. Use a small-bore feeding tube for nasoenteric feedings and verify placement with an abdominal radiograph or physical examination. 2. Initially order every 6-hour AccuChecks with sliding scale insulin. 3. Chemstrip urine for glucose for AccuChecks more than 200 mg/dL 4. Have the head of patients bed elevated 30 degrees at all times during feedings. 5. Check gastric residuals every 6 hours; if more than 200 mL, replace feeding and hold for 4 hours and then recheck. If still more than 200 mL, hold feedings. If less than 200 mL, resume feedings (not necessary for jejunostomy feedings). 6. Order appropriate laboratory tests (check Chem-7, Chem-15, prealbumin at least weekly).

J. Developing an EN Regimen 1. Determine energy and protein requirements. a. Usually 2530 kcal/kg/day and 1.01.5 g/kg/day of protein b. Calculation: (70 kg)(25 kcal/kg/day) = 1750 kcal/day

2008 American College of Clinical Pharmacy 1-340

Fluids, Electrolytes, and Nutrition (70 kg)(1.2 g/kg/day) = 84 g/day c. Decide which formula to use and identify its concentration (i.e., 1.06 kcal/mL vs. 1.5 kcal/mL vs. 2 kcal/mL) d. Divide total kcal per day by formula concentration: 1750 kcal/day 1.06 kcal/mL = 1650 mL/day e. Divide total milliliters by 24 hours to get the number of milliliters per hour: 1650 mL/day 24 hours/day 70 mL/hour 2. Checking your calculations a. Multiply enteral formula rate formula concentration 24 hours to get total kcal/day: (70 mL/hour)(1.06 kcal/mL)(24 hours/day) = 1781 kcal/day or 25 kcal/kg/day b. Multiply enteral formula rate grams of protein per liter or grams of protein/1000 kcal 24 hours to get total grams per day: (70 mL/hour) (44.4 g/L) (24 hours/day) = 74.6 g/day or 1.1 g, (1000 mL/L)

III. PN INITIATION AND MANAGEMENT A. Indications 1. PN is dened as the delivery of nutrients by vein in patients whose GI tract is nonfunctioning or inaccessible. 2. Appropriate conditions for which PN may be indicated include: a. Severely undernourished patients unable to take oral nutrition for more than 1 week or unable to be fed through the GI tract b. Severe pancreatitis precluding use of the GI tract c. Severe inammatory bowel disease (Crohns disease and ulcerative colitis) exacerbated by oral food/EN extensive bowel surgery (i.e., short bowel syndrome) causing malabsorption/maldigestion d. Small or large bowel obstruction e. Pregnancy (in cases of severe nausea and vomiting) f. Head-injury patients with no enteral access or GI dysfunction B. Basic Components of PN Formulations 1. Energy Substrates a. Dextrose monohydrate is the major source of carbohydrate calories. i. Each gram of dextrose monohydrate provides 3.4 kcal. ii. Dextrose monohydrate is commercially available in concentrations ranging from 5% to 70%. iii. Final concentrations of dextrose greater than 10% are usually delivered only through central veins because of the risk of thrombophlebitis associated with peripheral vein delivery. b. Glycerol, an alternative carbohydrate source, is a sugar alcohol that is contained in some premixed PN formulations. i. Glycerol provides 4.3 kcal/g. ii. In combination with amino acids, glycerol administration produces a protein-sparing effect. c. Intravenous fat emulsions (IVFEs) provide essential fatty acids and may serve as an additional source of calories. i. Components of IVFEs available in the United States include soybean oil or a mixture

2008 American College of Clinical Pharmacy 1-341

Fluids, Electrolytes, and Nutrition of soybean and safower oils as the source of fatty acids; egg yolk phospholipids, which act as an emulsier; glycerin to adjust the osmolarity; and sodium hydroxide to maintain the nal pH (between 6 and 9). ii. See Special Considerations for alternative IVFEs available outside the United States. iii. IVFEs are commercially available in concentrations of 10% (1.1 kcal/mL), 20% (2 kcal/mL), and 30% (3 kcal/mL). iv. The 30% product is approved only for use as a component of a TNA and not for administration as a separate infusion. v. Whereas each gram of an IVFE provides 9 kcal, the glycerol component contributes additional calories such that an IVFE 10% is equivalent to 11 kcal and IVFEs 20% and 30% are equivalent to 10 kcal. Amino acids are provided as a source for protein synthesis, which is required for body functions such as tissue growth, tissue repair, and immune function. a. Each gram of crystalline amino acids yields 4 kcal on oxidation. b. Amino acids preparations may be categorized as standard or specialized. i. Standard or balanced amino acid preparations contain a physiologic mixture of essential and nonessential amino acids. ii. Specialized or modied amino acid products are formulated to contain mixtures of amino acids to meet certain disease-state or age-specic requirements. c. Amino acid preparations are commercially available in concentrations ranging from 3% to 20%, although 8.5% and 10% are the most common concentrations used in compounding PN formulations. Electrolytes are added to PN formulations to maintain physiologic serum concentrations. Commercially available electrolyte salts added to PN include: a. Sodiumchloride and acetate b. Potassiumchloride and acetate c. Phosphorussodium and potassium d. Calciumgluconate (preferred salt) and gluceptate e. Magnesiumsulfate (preferred salt) and chloride Parenteral multivitamins are added to the PN formulation based on the recommendations of the American Medical Association and the Food and Drug Administration (FDA). a. Commercially available multivitamin products consist of both water-soluble and fatsoluble vitamins. b. Parenteral multivitamin products for use in adults include retinol (vitamin A), ergocalciferol (vitamin D), DL--tocopheryl acetate (vitamin E), phylloquinone (vitamin K), ascorbic acid (vitamin C), thiamin (vitamin B1), riboavin, pyridoxine (vitamin B6), niacinamide, dexpanthenol, biotin, folic acid, and cyanocobalamin (vitamin B12). Sterile water is often added to PN formulations to adjust the nal volume and meet a patients daily uid requirements. Histamine H2-receptor antagonists are often included in PN formulations to prevent and treat upper GI, stress-related ulceration. Regular insulin may be added to PN formulations to control blood glucose concentrations for patients who are at risk or exhibit signs of hyperglycemia.

2.

3.

4.

5. 6. 7.

C. PN Initiation 1. Quick nutritional assessment to determine nutritional status and requirements a. IBW: i. Female: 45.5 kg/5 ft + 2.3 (1 inch over 5 ft) ii. Male: 50 kg/5 ft + 2.3 (1 inch over 5 ft)

2008 American College of Clinical Pharmacy 1-342

Fluids, Electrolytes, and Nutrition b. Percent IBW = actual BW/IBW 100 i. obesity: more than 130% ii. moderate malnutrition: 70%79% iii. severe malnutrition: less than 70% c. Actual BW (ABW) is most commonly used for calculation of uid and nutritional requirements. When the actual weight of a patient exceeds a body mass index (BMI) more than 30 kg/m2, an adjusted BW can be calculated and used to determine nutritional requirements. The following equation may be used to calculate adjusted BW: i. Adjusted BW: (ABW IBW)0.25 + IBW ii. Alternatively, a high-protein, low-calorie prescription may be implemented for a grade II obese patient using the IBW. In this case, a goal of 22 total kcal/kg of IBW/ day and 2 g of protein/kg IBW/day is targeted to avoid overfeeding while maintaining lean tissue mass d. Body mass index (BMI): weight (kg)/height (m2) may be used to assess patients for obesity. i. Normal: 2024.9 ii. Grade I obesity (overweight): 2529.9 iii. Grade II obesity (moderate): 3040 iv. Grade III obesity (severe or morbid): more than 40 2. Fluid Requirements a. General: i. For the average adult with normal renal function, daily uid requirements may range from 2500 to 3500 mL. ii. For the average adult with renal insufciency, daily uid requirements may be reduced to a range of 5001500 mL (depending on the severity of the disease) to prevent uid overload. b. Fluid reuirements may be calculated using either of the two methods below: i. 1500 mL/rst 20 kg and then 20 mL/for each kilogram of IBW above the rst 20 kg Ex. For a 70-kg adult = 1500 mL + (20 mL/kg 50 kg) = 2500 mL ii. 3035 mL/kg of ABW Ex. For a 70-kg adult = (3035 mL/kg)(70 kg) = 21002450 mL 3. Protein (Amino Acid) Requirementsamino acid products are generally manufactured in 10%, 15%, or 20% concentrations a. In an adult, protein requirements range from 0.8 to 2.0 g of protein (amino acids) per kilogram of actual BW. Ex. For a 70-kg adult: (1.5 g/kg)(70 kg) = 70105 g amino acids b. The amount of amino acids given will depend on the stress level and/or the extent and level of body injury, with 1.2 g of protein per kg given for mild stress and up to 2 g/kg for severe stress. c. The following items are general guidelines for protein requirements based on stress or changes in organ function:
Maintenance Stress or repletion Renal failure/predialysis Renal failure/hemodialysis Renal failure/peritoneal OR continuous veno-venous hemodialysis Hepatic failure Liver transplant Bone marrow transplant 1 g/kg/day actual BW 1.32 g/kg/day actual BW 0.81 g/kg/day dry BW 1.21.5 g/kg/day dry BW 1.52 g/kg/day dry BW 0.61.2 g/kg/day dry BW 11.5 g/kg/day dry BW 1.52 g/kg/day dry BW

2008 American College of Clinical Pharmacy 1-343

Fluids, Electrolytes, and Nutrition

d. The choice of stock solution used (10% vs. 15% or 20%) will depend on the patients uid status. A 15% amino acid stock solution is the more concentrated amino acid product that is used in cases of uid restriction. 4. Dextroseproduct used for compounding of PN is 70% dextrose in water a. Dextrose is the major source of nonprotein kilocalories, but intravenous fat is also used. b. General requirements i. Because the maximum oxidation rate for glucose is 7 mg/kg/minute, most clinicians use a target range of 35 mg/kg/minute for glucose administration. (a) Ex. For a 70-kg adult: 35 mg/kg/minute 70 kg 1 g/1000 mg 1440 minutes/ day =302504 g of dextrose each 24 hours. (b) On the rst day of PN, start with 2 mg/kg/minute of dextrose; do not add intravenous fat to the TNA formulation because lipid is unstable in a TNA formulation with low concentrations of dextrose (less than 10% nal concentration) and low amino acid concentrations (less than 4% nal concentration). (c) For a TNA to be stable, the nal dextrose concentration must be more than 10%, the nal amino acid concentration more than 4%, and the nal fat concentration more than 2%. c. Control of hyperglycemia: Only regular human insulin may be added to the PN. i. The initial dose of insulin to start with in a PN is generally 0.1 unit/g of dextrose. For example, for a PN with 250 g of dextrose per day, 25 units of regular human insulin would be added to the bag. ii. After the rst day, add the total amount of sliding scale insulin used during the previous 24 hours to the next bag. iii. Caution must be used when adding insulin to the PN bag for patients with impaired renal function because insulin is renally eliminated and may be cleared more slowly in this patient population. iv. If a patient is on an insulin drip, clinical judgment is used to determine when insulin is added to the PN. Often, it is safe to add insulin to the PN when the rate of the insulin drip is stable and the patient is becoming more clinically stable (e.g., off vasopressor agents, off steroids) To calculate the amount of insulin to add to the PN bag, add the total amount of insulin used in the previous 24 hours, take two-thirds of that amount and add it to the next PN bag. For example: insulin drip rate = 2 units/hour 2 units/hour 24 hours = 48 units 2/3 of 48 units = 32 units to add to the PN 5. Intravenous Fat Emulsion a. IVFE is given on a daily basis to nearly all patients. b. Intravenous fat should consist of 1%4% of the total calories delivered to prevent essential fatty acid deciency. c. The dextrose, amino acids, electrolytes, and fat emulsion are often incorporated into one container. This is referred to as All-in-One, 3-in-1 solutions, or TNAs, and these preparations are infused through a central vein. d. The package insert for IVFEs warns against the provision to patients with egg allergies because the emulsion is stabilized with egg phospholipids. e. Intravenous preparations containing fat emulsion should be given with an in-line intravenous lter, and a lter size of 1.2 m is recommended. f. IVFEs may be given as a separate peripheral intravenous infusion. When administered as a separate infusion in addition to dextrose/amino acids, the duration of infusion

2008 American College of Clinical Pharmacy 1-344

Fluids, Electrolytes, and Nutrition should not exceed 12 hours in order to prevent the growth of microorganisms within the manufacturers original container. g. The product should always be examined visually before administration, per the standards in the USP monograph for IVFE. It should not be used if there is any evidence of creaming, aggregation, coalescence, or any other form of phase separation. h. Propofol, a sedative used in intensive care unit patients, is manufactured in a lipid vehicle, which provides 1.1 kcal/mL. Thus, the amount of lipid in the PN formula should be adjusted to account for the caloric contribution from propofol. 6. Electrolytes: when adding electrolytes to PN, it is best to add only one salt of each electrolyte to reduce the potential for errors (e.g., do NOT add K Phos AND Na Phos, do NOT add KCl AND KAc). a. Sodiumparenteral recommended daily intake (RDI) is determined by clinical need i. Sodium is principally an extracellular cation with no established RDI. Its inclusion in the PN is based on clinical need. ii. Usually, try to make about normal saline; therefore, add 80 mEq/L. iii. Patients with end-stage liver disease, congestive heart failure, or iatrogenic uid overload may require severe sodium restriction. iv. In general, if serum sodium is more than 150 mEq/L, no more than 40 mEq/day of sodium should be in the PN. v. Review the patients medication list for sources of sodium, such as albumin, medications in normal saline, and sodium-containing antibiotics such as Timentin. Other medications can cause dehydration that is manifested as hypernatremia, such as with lactulose. vi. Certain disease states can cause uid overload which is manifested as hyponatremia, such as worsening edema with chronic heart failure, cirrhosis, or nephrotic syndrome. In this case, sodium content of the PN should still be restricted. vii. Conversely, patients with large nasogastric uid losses, high ileostomy or pancreatic stula outputs, or large small bowel losses often require substantial quantities of sodium per day. viii. Sodium content in PN should not exceed 154 mEq/L. (It should contain no more sodium than normal saline.) b. Potassiumparenteral RDI is determined by clinical need. i. Potassium is principally an intracellular cation with no established RDI; thus, its inclusion in PN is dictated by clinical need. ii. To estimate potassium requirements, see if the patient is already receiving potassium in the intravenous uid, and add this amount to the PN; if there is no potassium in the intravenous uid and the patient has normal renal function, then around 40 mEq/L is usually sufcient. iii. Potassium requirements can be greatly inuenced by acid-base status. (a) During metabolic acidosis (pH less than 7.2), an excess of hydrogen ions are present in the circulation, and potassium exchanges its intracellular position for hydrogen ions in an attempt to abate the transfer, thus causing hyperkalemia. (b) Conversely, hypokalemia results during metabolic alkalosis. iv. Renal insufciency (creatinine clearance less than 30 mL/minute) is also associated with impaired potassium clearance and hyperkalemia. v. Many medications are associated with alterations in serum potassium concentrations: (a) Hyperkalemia: K-sparing drugs, including angiotensin-converting enzyme inhibitors, spironolactone, triamterene, amiloride, heparin, trimethoprim (i.e., Bactrim) (b) Hypokalemia: K-wasting drugs, including amphotericin B, aminoglycosides,

2008 American College of Clinical Pharmacy 1-345

Fluids, Electrolytes, and Nutrition antipseudomonal penicillins (Ticarcillin), loop/thiazide diuretics, glucocorticoids, insulin, inhaled -agonists (e.g., albuterol) vi. In general, if serum potassium is more than 5.1 mEq/L, no more than 20 mEq/day of potassium should be in the PN. c. Calciumparenteral RDI is about 10 mEq or 200 mg/day i. Up to 98% of total body calcium is in bone and can be readily mobilized in times of need under the inuence of parathyroid hormone. ii. Certain patients, such as those with severe short bowel syndrome or those requiring massive blood transfusions, may require substantially greater quantities of calcium, and such increases in the PN admixture should be accomplished gradually. iii. Dosage increases of 5 mEq daily for acute care are reasonable, and simultaneous monitoring of serum phosphorus is recommended during such times. iv. Remember that calcium is highly protein bound (especially to albumin). The following formula may be used to adjust serum calcium concentrations for low albumin concentrations, although it is still inaccurate: corrected calcium = [(4-albumin) 0.8] + measured calcium The most sensitive measure of calcium is ionized calcium. v. Low amounts of calcium may be required for patients with hyperphosphatemia, metastatic cancer, or hyperparathyroidism. vi. Usually, 10 mEq/day is a sufcient amount to add to the PN. d. Magnesiumparenteral RDI is around 10 mEq or 120 mg/day i. Magnesium is closely linked to calcium metabolism where it is necessary for parathyroid hormone secretion. ii. Patients with short bowel syndrome, alcoholics, etc., often require larger doses to achieve magnesium homeostasis and can be advanced incrementally by 50% of the parenteral RDI as with calcium. iii. Medications associated with magnesium wasting include amphotericin B, aminoglycosides, cyclosporin A, cisplatin, loop and thiazide diuretics, and piperacillin. iv. The amount of magnesium plus calcium should NOT exceed 20 mEq/L for a 3-in-1 formulation; otherwise, the emulsion may become unstable. v. In general, 12 mEq/L is appropriate to add to the PN; if the patient has renal dysfunction, then add 4 mEq/L. e. Phosphorusparenteral RDI is about 30 mmol or 1000 mg/day i. The role of phosphorus in physiologic processes is diverse, and it inuences respiration, myocardial function, platelets, and red and white blood cell function. ii. If omitted from PN formulations in the presence of normal renal function, it can induce a potentially life-threatening hypophosphatemia within a week of PN therapy. iii. Disease states associated with low serum phosphorus concentrations include alcohol abuse, thermal injury, trauma, and refeeding syndrome. iv. Medications associated with hypophosphatemia include antacids, sucralfate, diuretics, theophylline, and insulin. v. Medications associated with increased serum phosphorus concentrations include intravenous lipid emulsions and Fleets enemas in the presence of renal dysfunction. vi. In general, 30 mmol/day should be added to the PN; if the patient has renal dysfunction, then add 35 mmol/L. f. Chloride and acetate i. The use of chloride and acetate salts in the PN admixture should be based on the acid-base status of the patient. ii. For example, in cases of metabolic alkalosis, only chloride salts of either sodium or

2008 American College of Clinical Pharmacy 1-346

Fluids, Electrolytes, and Nutrition potassium should be used. iii. Conversely, in cases of acidemia, the emphasis should be placed on acetate salts of either sodium or potassium. iv. Under no circumstances should salts such as calcium chloride or sodium bicarbonate be used in a PN formula, because these can result in the formation of insoluble precipitates, which could fatal. 7. Trace elementsmost institutions use combination products containing four or ve individual trace elements a. Trace elements are added to the PN solution once each day. b. Standard trace element solutions: ve trace elements, such as Se, Cr, Cu, Mn, and Zn. Commercially available combination products typically provide 12 mcg of chromium, 1.2 mg of copper, 0.3 mg of manganese, 60 mcg of selenium, and 3 mg of zinc per 3 mL. c. Patients who have sustained small bowel or large bowel uid losses should receive supplemental zinc (510 mg/day) added separately in addition to the amount in the trace element cocktail (35 mg/day). d. Patients who have hepatic cholestasis should have copper and manganese withheld from the PN solution because these trace elements are excreted in the bile. Neurologic damage from deposition of manganese in the basal ganglia has been reported in PN patients with chronic liver disease or cholestasis. e. Under certain conditions, such as for long-term home PN, additional selenium may be necessary. 8. VitaminsMVIAdult (Hospira, Chicago, IL) and Infuvite Adult (Baxter Healthcare, Deereld, IL) are the only two products available in the United States a. See Table 1 for the individual vitamin content of multivitamin for infusion. The multivitamin solutions are mixed together just before use and added to the PN. The limited stability of the combined multiple vitamins should be considered when assigning an expiration date to a PN solution that contains multiple vitamins. b. Every PN should contain multivitamins daily. c. Although prescribers may ask to add intravenous iron to a TNA formulation, iron is generally not stable. D. Infusion of the formulation 1. Line access a. PN should be given through a central line, unless the solutions are specically formulated to be given peripherally (peripheral PN, or PPN). b. Central lines include PICC, Hickman, and Port-a-Cath or any lines inserted in which the tip of the catheter is positioned in the superior vena cava. c. If a peripheral vein is used for the administration of PN, the solution given must be less hypertonic. PPN solutions have osmolarities of around 700900 mOsm/L. d. To avoid exceeding normal daily uid requirements, the nutrients are usually administered as highly concentrated hypertonic solutions. To get some perspective, approximate osmolarities of plasma (in milliosmoles per liter) and typical large volume parenterals are as follows:
Plasma 0.9% NaCl D5W PN (central) 300 300 250 1800

e. Vein damage that would be caused by giving these highly hypertonic solutions (more than 900 mOsm/L) is minimized by administering the PN solution through a large-diameter central vein where blood ow is rapid. This enables the PN solution to be diluted rapidly as it ows into the body. 2008 American College of Clinical Pharmacy 1-347

Fluids, Electrolytes, and Nutrition f. If a peripheral vein is used for the administration of PN, the solution given must be less hypertonic. PPN solutions have osmolarities of about 700900 mOsm/L. g. Calculating the osmolarity of a PPN formula i. (grams of dextrose per liter of PN) 5 = milliosmole per liter ii. (grams of protein per liter of PN) 10 = milliosmole per liter iii. Convert total milliosmoles supplied by electrolytes in PN to milliosmoles per liter (see table below). iv. Total milliosmoles per liter of PPN should be less than 900 (see below).
Electrolyte NaCl Sodium acetate KCl K acetate Na Phos Na Phos K Phos K Phos Ca Mg mEq/mL 2.5 4 2 2 4 3 mmol 4 3 mmol 0.465 4 mOsm/mL 5 8 4 4 7 7.4 0.68 4.06

h. In essence, the electrolyte and intravenous fat contents are similar to central PN, but the amino acid content is cut by about half, and the dextrose is greatly reduced. i. Uses of PPN - PPN may be used to support patients who are only able to ingest a portion of their caloric and protein requirements orally or enterally and when central vein PN is not feasible. ii. PPN is traditionally a short-term therapy (less than 10 days) because it does not provide total caloric/protein requirements and is not tolerated well by peripheral veins for extended periods. iii. Limitations of PPN include lower concentrations (osmolarity) of nutrients to avoid thrombophlebitis and uid overload. Thus, PPN is not recommended for patients with severe undernutrition, increased electrolyte needs (especially potassium), uid restriction, or need for prolonged intravenous nutrition support. i. Heparin (3000 units/day) and hydrocortisone (5 mg/L) are often added to PPN formulations to decrease the risk of thrombophlebitis. 2. Cycling the PN: a. Sometimes, cycling the PN overnight for 12 hours is desired to allow the patient freedom for physical activity during the day or avoid hepatic fat accumulation. b. The PN infusion should be gradually decreased for 34 days to prevent hyperglycemia while tapering the infusion rate up and rebound hypoglycemia when tapering the infusion rate downward. For example, the PN infusion should be decreased from 24 hours to 18 hours to 14 hours to 12 hours. c. The following formula should be used to calculate the cyclic rates for PN: i. Divide the total volume of the PN by the [desired cycle infusion time minus 1 hour] to identify the maximum infusion rate during the cycle. ii. Multiply the maximum infusion rate by the [desired cycle infusion time minus 2 hours]. iii. Subtract the total volume from [maximum infusion rate 2 hours] to obtain the volume left for tapering up and down the PN. iv. Divide the volume left by 2 and this will be the rate for the rst and last hour of the PN cycle. 2008 American College of Clinical Pharmacy 1-348

Fluids, Electrolytes, and Nutrition v. For example, suppose a 2-L PN is to be infused for 18 hours: (a) 2000 mL (18 hours 1) = 117 mL/hour (round up to 120 mL/hour) (b) 120 mL/hour (18 hours 2) = 1920 mL (c) 2000 mL 1920 mL = 80 mL (volume left for tapering up/down) (d) 80 mL 2 = 40 mL/hour vi. Thus, the cycle regimen would be 40 mL/hour 1 hour, followed by an increase to 120 mL/hour 16 hours and then a decrease to 40 mL/hour 1 hour. vii. Fingerstick glucose tests should be checked 3 hours after the PN is started (to monitor for hyperglycemia) and 30 minutes after the PN is stopped (to monitor for rebound hypoglycemia). 3. Discontinuation of the PN Formulation a. Abruptly stopping the PN formulation may result in rebound hypoglycemia. b. If a patient is not receiving continuous enteral feedings, the PN rate should be reduced by 50% every 2 hours until the rate is less than 25 mL/hour. c. Once the PN rate is less than 25 mL/hour, the PN may be stopped. d. It is usually recommended to check a ngerstick glucose test 30 minutes after the PN is stopped to make sure the patient is not hypoglycemic. E. Special Considerations with PN Formulations 1. Special Amino Acid Solutions a. Formulas and descriptions of various specialty amino acid solutions can be found in Drug Facts and Comparisons. There are special formulations for patients under stress, those with hepatic failure, and those with renal disease. b. The use of these specialty products is somewhat controversial. They are usually more expensive than standard formulas, and some practitioners think they offer little signicant advantage. Additional information on this subject can be found in the journal article, Value of specialty intravenous amino acid solutions, beginning on page 671 of the March 15, 1996, issue of American Journal of Health-System Pharmacy. c. Amino Acid Solutions for Patients with Hepatic Failure i. These solutions contain higher concentrations of branched-chain amino acids, such as isoleucine, leucine, and valine, and lower concentrations of aromatic amino acids (tryptophan and phenylalanine) and of methionine. An example product is HepatAmine 7. ii. This formula modication resulted from an analysis of the plasma content of patients with encephalopathy, a clinical complication found in patients with liver failure. It was noted that in the plasma of these patients, there was an elevation of the ratio of aromatic amino acids to branched-chain amino acids. It was thought that the increased concentrations of aromatic amino acids were contributing to the development of encephalopathy. iii. Current recommendations support the use of branched-chain amino acid formulations only in chronic encephalopathy that is unresponsive to standard amino acid products and pharmacotherapy (i.e., lactulose). 3. PN Stability and Compatibility a. A distinction should be made between the terms stability and compatibility in reference to PN formulations. b. Stability of PN formulations denotes the degradation of components, which causes alterations from their original characteristics. i. The Maillard reaction, which occurs between amino acids such as lysine and intravenous dextrose causing a brownish discoloration, is an example of an instability issue with PN formulations.

2008 American College of Clinical Pharmacy 1-349

Fluids, Electrolytes, and Nutrition ii. Stability issues may also pertain to changes in chemical integrity or pharmacological action, such as the photodegradation associated with some vitamins due to light exposure. c. Compatibility issues, on the other hand, refer to precipitate formation, either solid or liquid. i. Liquid precipitates in PN formulations occur when there is phase separation of the IVFEs into oil and water. ii. An example of a solid precipitation problem with PN formulations is the formation of calcium phosphate precipitates. 4. Factors Affecting the Precipitation of Calcium Phosphate in PN Solutions Calcium and phosphate, two nutritional requirements for PN solutions, are conditionally compatible. Precipitation depends on several factors, as described below. The chemical equation given here should aid in understanding these factors. HPO4-2 + Ca+2 H2PO4-1 + Ca+2 CaHPO4 very insoluble Ca(H2PO4)2 relatively soluble

a. The pH of the solution The pH dependence of the phosphate-calcium precipitation is illustrated in the equation above. Dibasic calcium phosphate (CaHPO4) is very insoluble, whereas monobasic calcium phosphate (Ca(H2PO4)2) is relatively soluble. At low pH, the soluble monobasic form (H2PO4) is predominant, but as the pH increases, more dibasic phosphate (HPO42) becomes available to bind with calcium and precipitate. Therefore, the lower the pH of the parenteral solution, the higher the amount of calcium and phosphate that can be solubilized. b. The concentration of the calcium Because it is the free calcium that can form insoluble precipitates, enhanced precipitate formation is expected as the concentration of calcium is increased. c. The salt form of the calcium Although calcium gluconate is much more soluble than calcium chloride, calcium chloride has a much higher percent dissociation. The higher the dissociation, the more free calcium available. The concentration of calcium available for precipitation when added as the gluconate salt is less than that available when an equimolar amount of calcium is added as the chloride salt. Calcium gluceptate has no real advantage over the gluconate salt form. d. The concentration of the phosphate As in the previous equations, it is the dibasic calcium phosphate salt that is insoluble. The concentration of dibasic phosphate in solution depends on both the total phosphate concentration and the pH of the solution. Potassium phosphate Injection has a high pH (6.26.8) relative to that of dextrose or amino acid solutions. Addition of potassium phosphate injection to a PN solution not only increases the concentration of the phosphate but also may increase the pH of the solution, which favors precipitation. e. The concentration of amino acids Amino acids form soluble complexes with calcium and phosphate, reducing the amount of the free calcium and phosphate available for precipitation. Amino acids also appear to provide an intrinsic buffering system to a PN solution. Amino acid formulations have pH values in the range of 4.56.5. Those containing higher concentrations of amino acids show less of an increase in pH when phosphate is added and, consequently, an increased

2008 American College of Clinical Pharmacy 1-350

Fluids, Electrolytes, and Nutrition tolerance for calcium addition. f. The composition of amino acid solutions Amino acid solutions formulated with electrolytes contain calcium and phosphate, and these must be considered in any projection of compatibility. Some amino acids contain cysteine hydrochloride, which may affect the solubility of calcium and phosphate. Cysteine hydrochloride lowers the pH of the solution, enabling the more soluble monobasic form of phosphate to exist. Therefore, adding cysteine hydrochloride can increase the solubility of calcium and phosphate in a PN solution. g. The concentration of dextrose in the solution Dextrose forms a soluble complex with calcium and phosphate. In addition, it can act as a weak buffer. The pH of dextrose solutions is relatively low (45) because of the free sugar acids (e.g., gluconic acid) present and formed from the oxidation of the aldehyde moiety on dextrose during sterilization and storage of dextrose solutions. Studies have shown that higher concentrations of dextrose reduce the free calcium and phosphate that can form insoluble precipitates. h. The temperature of the solution Temperature of solution also plays a key role. As temperature is increased, the calcium salts (chloride or gluconate) are dissociated more completely, and more calcium ions become available for precipitation. Therefore, an increase in temperature increases the amount or possibility of precipitation. Care must be exercised when transferring these solutions to warmer environments. i. The presence of other additives The addition of other drugs to a PN solution may alter the pH of the solution. Additives may also introduce the possibility of precipitation of other products or incompatibilities with other ions. j. The order of mixing The FDA recommends that phosphate be the rst additive added to an admixture and that calcium be the last additive. 4. Monitoring a. For an excellent review article on PN monitoring, see Parenteral Nutrition Monitoring in Hospitalized Patients (2). b. Monitoring parameters i. Temperaturedaily to detect infection or sepsis ii. Weightdaily to monitor uid imbalance and maintenance and improvement of clinical condition. In general, patients should gain only 12 pounds per week. Larger weight gains are usually retained uid or fat from too many calories; a sudden weight gain usually reects uid retention. iii. Nitrogen balance (NB)this monitors nitrogen use to determine if the patients metabolic status is anabolic (buildup) or catabolic (breakdown). It is dened as the difference between the nitrogen intake and nitrogen excretion. NB = NI NO NI is the nitrogen put into the body by PN and other nutritional sources. NO is the nitrogen excreted by all routes.

2008 American College of Clinical Pharmacy 1-351

Fluids, Electrolytes, and Nutrition You may see two different, but equivalent, forms of this basic equation in the literature: NB = NI(g/24h) (UUN(g/24h) + 4 g) or NB = (protein(g/24h) 6.25)(UUN(g/24h) + 4 g) NI is obtained by calculating the number of grams of protein infused in the form of amino acids and multiplying that by 16% (the approximate amount of nitrogen in amino acids). Note that you get the same number by dividing the number of grams of protein or amino acids given by 6.25. This value also can be determined from the specications for the particular amino acid solutions being used. This information can be found in the product package insert or from Drug Facts and Comparisons. UUN is urine urea nitrogen. Urea is the breakdown product of protein. 4 g is the estimated loss by other routes such as through the skin or in the feces. E X A M P L E: (a) J.S. is a patient receiving a PN solution that contains 100 g of amino acids (AA). The 24-hour urine urea nitrogen reported by the laboratory is 9.5 g/24 hours. What is his NI(g/24h)? (100 g AA 6.25) = 16 g N/24 hours (b) Calculate the Nitrogen balance (NB) for this patient: NB = 16 (9.5 + 4) = 2.5 g What is an acceptable value for NB? Although this depends on the clinical situation, a positive 46 g/day in unstressed patients is considered acceptable. If the NB is too low or is negative, the PN formula can be changed to increase the amino acid content. iv. Plasma proteinsconcentrations of serum plasma proteins can be used as a measure of nutritional status because an increase in these proteins reects an anabolic response. (a) Serum albumin is the most commonly determined plasma protein, but its usefulness in monitoring nutritional status is limited because of its long half-life, because the body pool of albumin is large and because its concentration in the serum is affected by so many other factors (2). (b) Two other plasma proteins, transferrin and prealbumin, have been found to be useful indicators (2). (c) Assessment of C-reactive protein will help quantify if the fall in short-term serum proteins (i.e., prealbumin) are associated with an acute-phase response or nutritional deciency. C-reactive protein is recognized as a positive acute-phase protein, dened as one whose plasma concentration increases by at least 25% during inammatory disorders. (d) If C-reactive protein is elevated and prealbumin has fallen, this is more indicative

2008 American College of Clinical Pharmacy 1-352

Fluids, Electrolytes, and Nutrition of the systemic response to inammation. However, a falling prealbumin with a concurrent low C-reactive protein concentration may represent an inadequate intake of energy or protein. (e) Use of these basic principles can assist the clinician in determining the appropriate time to alter a patients nutritional regimen. v. Laboratory Tests (a) Refer to the appendix for a typical frequency of laboratory test determinations. The tests done, the frequency, and the normal values vary with the hospital and the clinical condition of the patient. Note that there are hematologic tests, electrolyte and glucose concentrations, fat-cholesterol monitoring tests, and liver and renal function tests. For more detailed information on laboratory tests, see the article on monitoring PN therapy (2). (b) It is important to realize that acceptable concentrations for some laboratory tests may be different for patients on PN than for normal healthy individuals. (c) Monitoring of laboratory values with adjustments of PN formulas and therapy is becoming a focal point of pharmacist input and participation on the health care team. vi. Clinical Statushow is the patient doing. This is a very important monitoring parameter. A desired clinical outcome of therapy should be determined, and all efforts in PN therapy should be geared toward this end. Example: PN Order: G.D. is a 52-year-old, 176-lb man who is 61 tall. He is admitted to the trauma unit after an automobile accident. He is not expected to eat or take tube feedings for more than 7 days because of multiple injuries to his small bowel. His physician, Dr. Solier, has written the following PN order: The prescribed PN contains:
Amino acids Dextrose Fat Sodium chloride Potassium acetate Sodium phosphate Magnesium sulfate Calcium gluconate Ranitidine Multivitamins Multiple trace elements 120 g 346 g 55 g 150 mEq/day 80 mEq/day 30 mmol/day 24 mEq/day 10 mEq/day 150 mg/day 10 mL/day 10 mL/day

The ow rate for this PN is 100 mL/hour. 1. Calculate G.D.s IBW. Then, using a PN ow rate of 100 mL/hour, determine if all the nutrients, electrolytes, and uid volumes are within the normal range for G.D. You may ignore the electrolyte contributions in the intravenous fat. Sample calculations are given below. 2. Calculate the number of milliliters of each component for this PN solution. The crystalline amino acid solution is Travasol 10%; the dextrose solution is dextrose 70% in water. Sample calculations are given below.

2008 American College of Clinical Pharmacy 1-353

Fluids, Electrolytes, and Nutrition

Sample Calculations for 24-hour Requirements: For G.D.: A. BW 1. Weight in pounds (given): 176 lb 2. Actual weight in kilograms:176 lb 2.2 lb/kg = 80 kg 3. IBW in kilograms: IBW = 50 kg + 2.3 (13) = 79.9 kg G.D.s current weight is appropriate for his height. B. Fluid 1. Volume ordered per day: (100 mL/hour)(24 hours/day) = 2400 mL 2. Fluid requirements: 3035 mL/kg/day (3035 mL/kg/day)(80 kg) = 24002800 mL/day C. Protein (Amino Acids) 1. Grams of protein ordered per day: 120 g AA 80 kg actual BW = 1.5 g AA/kg actual BW 2. Kilocalories per day from protein: (120 g AA)(4 kcal/1 g AA) = 480 kcal/day 3. General requirement: 11.5 g AA/kg of actual BW D. Nonprotein Kilocalories (dextrose and fat) 1. Kilocalories per day from ordered dextrose: (346 g dextrose) (3.4 kcal/1 g dextrose) = 1176 kcal/day 2. Milligrams per kilogram per minute from ordered dextrose: (346 g dextrose/80 kg)(1000 mg/1 g dextrose)(1 day/1440 minutes) = 3 mg/kg/minute 3. General dextrose requirements: 35 mg/kg/minute 4. Kilocalories per day from ordered fat (do not add fat until second day): (55 g fat)(9 kcal/1 g fat) = 495 kcal/day 5. Total ordered kcal per day = 480 + 1176 + 495 = 2151 kcal/day 6. Total ordered kilocalories per kilogram of actual BW per day: (2151 kcal/80 kg actual BW)= 26.9 kcal/kg/day actual BW 7. General requirements for total kilocalories per day: 2535 kcal/kg actual BW 8. % total kilocalories as intravenous fat: (495 kcal/2151 kcal) = 23% total kcal as intravenous fat 9. General requirements for intravenous fat: 30% or less of total kilocalories/day E. Electrolytes 1. Sodiumin general, most patients require 12 mEq/kg/day in the PN 2. Potassiumin general, most patients require 12 mEq/kg/day in the PN 3. Calciumthe parenteral RDI of about 10 mEq/day is sufcient for most PN patients 4. Magnesium420 mEq/day are typically added to PN formulations 5. Phosphorusthe parenteral RDI of about 30 mmol is sufcient for most PN patients

2008 American College of Clinical Pharmacy 1-354

Fluids, Electrolytes, and Nutrition V. WATER HOMEOSTASIS A. Fluid distribution 1. In humans, total body water accounts for about 55% of total BW. 2. Body water is distributed throughout three compartments: a. Intracellular uid has a volume that approximates 35% of total body water. b. ECF has a volume that accounts for 20% of total body water. i. Interstitial uid (16%) ii. Intravascular uid (7%) B. Volume regulation and sodium metabolism 1. Serum sodium concentration reects the osmolality of all body uids. 2. *Serum osmolality = (2 Na+) + (glucose) + (blood urea nitrogen) 18 2.8 *The number of osmoles acting to hold uid within the ECF; therefore, serum osmolality becomes important in the distribution of total body water between intravascular, interstitial, and intracellular uid compartments 3. The two most important hormones that inuence water and sodium physiology in the body: a. Antidiuretic hormone (ADH) b. Aldosterone 4. ADH a. Synthesized in the hypothalamus and released from the posterior aspect of the pituitary gland b. An increase in osmolality stimulates the secretion of ADH. c. A decrease in blood volume stimulates ADH secretion. 5. Aldosterone a. Mineralocorticoid synthesized in the adrenal gland b. Causes Na+ to be reabsorbed and K+ and H+ to be secreted c. The major stimulus for release of aldosterone is angiotensin II, a substance activated by renin, which is released in response to a depletion of ECF volume. VI. WATER AND NA+ DISORDERS A. Hyponatremia (less than 135 mEq/L) 1. Etiologies a. Total Body Na+ more than Total Body Water i. Extrarenal ii. Renal b. Normal Total Body Na+ and Total Body Water i. Glucocorticoid insufciency ii. Hypothyroidism iii. Psychogenic polydipsia iv. SIADH c. Total Body Na+ less than Total Body Water i. Chronic heart failure ii. Cirrhosis iii. Nephrotic syndrome

2008 American College of Clinical Pharmacy 1-355

Fluids, Electrolytes, and Nutrition 2. Table 2. Signs and Symptoms of Hyponatremia


Total Body Na+ > Total Body Water Symptoms Signs Dizzy Lightheaded Total Body Na+ Total Body Water Total Body Na+ < Total Body Water Shortness of breath Swollen ankles Weight gain Distended neck veins Peripheral edema Ascites Urine Na+ Total Body Na+ < Total Body Water Furosemide Free water restriction Give intravenous medications in normal saline

Poor skin turgor Blood pressure Tachycardia Flat neck veins Urine Na+

Normal blood pressure

Laboratory values

Urine Na+

3. Table 3. Treatment of Hyponatremia


Total Body Na+ > Total Body Water Intravenous normal salinea Discontinue diuretics Treat diarrhea/vomiting
a

Total Body Na+ Total Body Water Free water restriction Normal saline Furosemide

Hypertonic saline.

4. *Guidelines for Use of Hypertonic Saline a. Rapid infusion: 3% saline at 12 mL/kg/hour 23 hours i. Indications: (a) Seizures and coma (b) Symptoms of acute water intoxication b. Slow infusion: 3% saline at 15 mL/hour i. Indications: c. Slow response to water restriction i. Inability to take oral salt ii. Cautions (a) Avoid correction by more than 12 mEq/L/day (b) Use with furosemide in patients at risk for chronic heart failure. A. Hypernatremia (more than 145 mEq/L) 1. Etiologies a. Total Body Na+ less than Total Body Water i. Replacement with hypertonic uids ii. Drugs b. Total Body Na+ and Total Body Water i. Nonrenal losses ii. Diabetes insipidus iii. Drugs c. Total Body Na+ more than Total Body Water i. Iatrogenic administration of Na+ ii. Drugs

2008 American College of Clinical Pharmacy 1-356

Fluids, Electrolytes, and Nutrition d. Table 4. Laboratory Values


Total Body Na+ < Total Body Water Urine Na+ > 20 mEq/L Total Body Na+ > Total Body Na+ Total Body Water Total Body Water UOP > 3 L Urine Na+ > 20 mEq/L Urine osmolality < 300 mOsm

UOP = urinary output. e. Physical symptoms i. Thirst ii. Restlessness iii. Irritability iv. Hyperactive reexes v. Muscle rigidity vi. Seizures

2. Table 5. Treatment of Hypernatremia


Total Body Na+ < Total Body Water IV NS IV D5W or NS Total Body Na+ Total Body Water IV D5W or NS Vasopressin Total Body Na+ > Total Body Water IV D5W Diuretics

D5W = 5% dextrose in distilled water; IV = intravenous; NS = normal saline.

VI. POTASSIUM HOMEOSTASIS A. Total body stores about 5055 mmol/kg in an adult; more than 95% intracellular; serum K 3.54.8 mmol/L; unit conversions: 1 mmol of K = 39.1 mg; 1 g of K= 25.6 mmol B. Median daily turnover in the United States: 85 mmol for men and 55 mmol for women; Institute of Medicine recommendation is 120 mmol/day for adults with good renal function and no interfering medications; 90% of excretion is renal, and the remainder is secreted into the gut C. Balance between intracellular and ECF affected by: 1. Insulin: increases glucose and potassium uptake by liver and skeletal muscle 2. Catecholamines (2-receptors increase vs. -receptors decrease uptake into cells); 2 effect dominates when both receptors are stimulated, e.g., with endogenous epinephrine 3. pHdecreased pH shifts K+ out of cells, increased pH shifts K+ into cells; however, this effect is highly variable clinically D. Adaptations to high K intake: renal tubular secretion (aldosterone-modulated and direct tubular effect of K+) and secretion into gut; adaptations in renal failure prevent problems of K retention until glomerular ltration rate is less than 10 mL/minute E. Causes of hyperkalemia 1. Decreased urinary excretion: renal failure, decreased renal blood ow, hypoaldosteronism, renal tubular defects 2. Shift from intracellular to extracellular compartment: acidosis, insulin deciency, cell lysis/ catabolism, severe exercise, hyperkalemic periodic paralysis; pseudohyperkalemiared blood cell lysis in blood sample 3. Increased K intake combined with compromised ability to excretediet, salt substitutes, oral supplements, iatrogenic intravenous potassium supplementation, potassium-rich herbal supplements

2008 American College of Clinical Pharmacy 1-357

Fluids, Electrolytes, and Nutrition 4. Drugs that may contribute to or cause hyperkalemia: a. Intracellular to extracellular shift: succinylcholine, digitalis poisoning, -blockers, -agonists b. Potassium salts: dietary salt substitutes, penicillin G or V (less than 2 mmol of K per million units), phosphate supplement (i.e., K Phos) c. Decreased excretion: K-sparing diuretics, angiotensin-converting enzyme inhibitors, trimethoprim, heparin, cyclosporine, nonsteroidal anti-inammatory drugs 5. Increasing frequency of drug-induced hyperkalemias in elderly people with congestive heart failure, due to reduced renal function and treatment with angiotensin-converting enzyme inhibitors plus spironolactone. F. Manifestations of hyperkalemia 1. Skeletal muscle weaknessunusual at serum K less than 8 mmol/L 2. Cardiac conduction disturbances: increased T waves (K more than 6), widening QRS (K more than 78), decreased P wave (K more than 8), progressing to sin wave pattern, V brillation, and asystole G. Treatment of hyperkalemia 1. Determine urgency: immediate aggressive Rx indicated for serum K more than 8, severe muscle weakness, or marked electrocardiographic (ECG) changes; for asymptomatic K 6.58.0, can start Rx with K removal (e.g., Kayexalate or dialysis); conservative management for mild asymptomatic hyperkalemia. 2. Urgent Rx a. Intravenous calcium ion to antagonize electrophysiologic effects of hyperkalemia; e.g., calcium gluconate 12 g (4.89.6 mEq) for 35 minutes; onset immediate, duration less than 30 minutes. Note that calcium is a physiologic antagonist and does not affect K+ concentrations in body uids. b. Intracellular shift of K into cells: i. Insulin and dextrose; e.g., 510 units of regular insulin plus 25 g of 50% dextrose intravenous bolus, followed by 10% dextrose at 50 mL/hour; onset less than 15 minutes, duration hours. ii. 2-agonist; for example, nebulized albuterol 20 mg, inhaled for 10 minutes after insulin and dextrose administration; note inhaled albuterol dose is several-fold higher than that used for asthma to achieve systemic effect; subcutaneous epinephrine also works, but avoid in patient on -blockersmay get increase in serum K due to dominant effect; onset less than 30 minutes, duration several hours iii. Sodium bicarbonate 45 mmol intravenously for several minutes; efcacy variable, unlikely to work unless patient is acidotic; onset 30 minutes to 4 hours 3. Removal of potassium from the body: note that serum [K] will decrease with relatively small amounts of K removed when in hyperkalemic range 4. Sodium polystyrene sulfonate (Kayexalate)cation exchange resin that exchanges Na for K in the gut (nonabsorbable); administer orally (more effective) or by retention enema (more rapid onset), 1530 g; onset 12 hours; each gram of resin exchanges 0.51 mmol of K; resin usually suspended in 33%70% sorbitol solution to prevent constipation/impaction problems 5. Dialysis: hemodialysis (most rapid removal) or peritoneal dialysis H. Causes of hypokalemia 1. Inadequate intakepoor dietary intake; inadequate intravenous supplements in NPO (nothing by mouth) patients; clay ingestion (geophagia or pica) 2. Extracellular/Intracellular shiftincreased pH; insulin; -adrenergic (exogenous agonists;

2008 American College of Clinical Pharmacy 1-358

Fluids, Electrolytes, and Nutrition endogenous sympathoadrenal dischargethyrotoxicosis, delirium tremens, coronary ischemia); familial periodic paralysis; hypothermia; drug toxicityxanthines, verapamil, chloroquine; pseudohypokalemia (intracellular shift after blood sample drawn) 3. GI or sweat losseslaxative or enema abuse may be cause of occult K+ losses 4. Urinary lossesdiuretic Rx; mineralocorticoid excess (renin/angiotensin/aldosterone system, adrenal tumor or hyperplasia, steroids, licorice); salt-wasting nephropathies; large dose penicillin; amphotericin B; hypomagnesemia; renal tubular acidosis; osmotic diuresis I. Clinical manifestations of hypokalemia 1. Muscle weakness/paralysis-associated with serum K+ less than 2.5, together with other factors (Ca++, pH, type, and rapidity of change); due to alterations in cell membrane potentials; lower extremities most sensitive; may also cause muscle cramps, tetany, paresthesias, tenderness; impaired muscle blood owischemia, rhabdomyolysis/myoglobinuria; smooth muscle involvementconstipation or ileus 2. Cardiovascular effectsECG changes (see above)T-wave attening, appearance of U wave; digitalis toxicity; arrhythmias (unusual in healthy hearts); chronic low-grade K+ depletion may contribute to hypertension 3. Renal effectsimpaired concentrating ability; increased renal NH3 production and bicarbonate reabsorption resulting in metabolic alkalosis 4. Glucose intoleranceimpaired insulin secretion

J. Treatment of hypokalemianote that large amounts of K may be necessary to raise serum [K] from the hypokalemic range 1. Urgency 3.0 less than K+ less than 3.5: usually asymptomatic; if patient on digitalis, should give oral supplement to correct; otherwise, dietary sources of K+ usually adequate to correct 2.5 less than K+ less than 3.0: treat with oral supplements 2.0 less than K+ less than 2.5: some clinical manifestations likely; treat promptly with oral supplements; use intravenous supplement if oral route is questionable K+ less than 2.0: severe hypokalemia with probable total body decit more than 600 mmol of K+; intravenous supplement should start immediately 2. Choice of potassium salts and preparations a. Dietary sourcesMuch of dietary K is present as phosphate rather than chloride salt and may not restore K deciency effectively if patient is also chloride decient b. KCl is preferred salt in most cases of hypokalemia due to concomitant chloride losses; options for oral Rx include solutions (bad taste/poor compliance), KCl wax matrix tablets (8, 10 mEq), KCl microencapsulated capsule (8, 10 mEq), KCl sustained-release microcrystalloids (10, 20 mEq) c. Other saltsK Phosphate (preferred if patient also needs P replacement); bicarbonate or bicarbonate precursors (citrate, acetate, lactate, gluconate)preferred if patient also has bicarbonate losses (e.g., lower GI tract losses, renal tubular acidosis). d. Cautions: potassium supplements may cause a precipitous rise in K+ in patients having compromised ability to excrete a potassium loadrenal dysfunction, potassium-sparing diuretics; angiotensin-converting enzyme inhibitors, angiotensin II antagonists, heparin, and trimethoprim may have some effect, also. Patients on -blockers have a slower penetration of K into their intracellular compartment. e. Parenteral potassium: general guidelines i. Concentration in intravenous uid should not exceed 40 mEq/L for peripheral vein, 120 mEq/L for central vein; a pump or controller should be used if concentration more than 80 mEq/L.

2008 American College of Clinical Pharmacy 1-359

Fluids, Electrolytes, and Nutrition ii Infusion rate should not exceed 40 mEq/hour (0.5 mEq/kg/hour for pediatrics); when infusing 20 mEq/hour or more (0.2 mEq/kg/hour or more in pediatrics), should monitor for ECG changes during infusion. Some experts reco mmend a maximum infusion rate of 20 mEq/hour.

VII. DISORDERS OF MAGNESIUM HOMEOSTASIS A. Biological functions and physiologic processes 1. Cofactor in oxidative-phosphorylation reactions in mitochondria 2. Catalyzes enzymatic processes and activates phosphatases concerning transfer, storage, and use of adenosine triphosphate (ATP) 3. Involved in second messenger generation of adenyl cyclase 4. DNA/protein/fat synthesis, glucose use 5. Maintenance of sodium-potassium ATPase pump and cell membrane electrolyte composition/ action potential 6. Structural component of bone 7. Parathyroid hormone secretion/synthesis (?) 8. Neuromuscular transmission 9. Cardiovascular excitability 10. Vasomotor tone 11. Muscle contraction 12. Pharmacokinetics (absorption, distribution, elimination) a. Total body distribution i. Second most common intracellular cation ii. 1 mEq = 12 mg = 0.5 mmol iii. Stores: 25 mEq/kg (17502000 mEq) in normal adult iv. 53% located in bone (a) 30% is exchangeable from surface-limited pool and source for endogenous supply (b) Crystal mineral lattice pool is not readily exchangeable. v. 27% located in skeletal muscle, 19% in soft tissues (liver, brain, heart, and kidney) vi. Less than 1% in extracellular (0.3% serum) and red blood cell count (0.5% erythrocyte) b. Serum i. 0.3% of total body stores, 25% bound to albumin and 8% bound to globulins (33% bound to proteins) ii. 55% of serum Mg is in ionized (free) form, and 12% is complexed to phosphate, citrate, and other compounds iii. Normal range is 1.52.0 mEq/L (1.62.3 mg/dL) c. Absorption i. RDI for adult is 0.4 mEq/kg/day (5 mg/kg/day) ii. 30%50% is absorbed (815 mEq/day) (a) Primarily in jejunum and ileum (b) Begins about 1 hour after ingestion and continues at uniform rate for 28 hours (c) Absorptive capacity may be as low as 25% on a high Mg diet and as high as 75% on a low Mg diet. d. Elimination i. Primarily eliminated through the kidney (a) 810 mEq/day in the urine

2008 American College of Clinical Pharmacy 1-360

Fluids, Electrolytes, and Nutrition (b) 35%45% of daily oral intake renally excreted in normal adult (c) circadian rhythm of Mg renal excretion occurs maximally at night (d) Only 1%2% of endogenous Mg is eliminated in the feces B. Hypomagnesemia (less than 1.5 mEq/L) 1. Laboratory diagnosis is often difcult. a. Difcult to predict cellular effects of low serum Mg based on serum concentration b. Urinary Mg less than 12 mEq (1224 mg)/24 hours is more reective of Mg deciency i. May develop within 7 days after decreased Mg intake ii. May develop decreased urinary Mg before decreased serum Mg in patients with normal renal function c. 24-hour urinary excretion after Mg load of 64 mEq of Mg sulfate i. Mg-decient patients excrete less than 50% of load. ii. NormoMg patients excrete more than 60% of load. 2. Mechanisms and etiologies a. Gastrointestinal i. Reduced intake: protein-calorie malnutrition, prolonged intravenous uid without Mg, TPN without Mg, alcoholism, anorexia ii. Reduced absorption: malabsorption syndromes (celiac disease, Whipples disease, radiation enteritis, tropical sprue), short bowel syndrome, intestinal bypass surgery for obesity, acute pancreatitis iii. Increased losses: diarrhea (ulcerative colitis, colonic neoplasms, Crohns disease), laxative abuse, bulimia or excessive vomiting, gastric suctioning, intestinal and biliary stulas b. Renal i. Drug induced: diuretics (especially loop), aminoglycosides, amphotericin B, cyclosporine A, cisplatin, alcohol, ticarcillin, piperacillin, carbenicillin, digoxin ii. Other: renal tubular acidosis, hypercalcemic states including malignancies, postobstructive diuresis, diuretic phase of acute tubular necrosis, hereditary renal magnesium wasting, hyperaldosteronism, and renal transplantation c. Intracellular redistribution i. Diabetic ketoacidosis, hyperthyroidism d. Miscellaneous: thermal injury patients, pregnancy, lactation, exchange transfusion 3. Clinical manifestations of hypomagnesemia a. Central nervous system: nystagmus, seizures, depression, agitation, psychosis b. Muscular: muscle fasciculations, tremors, hyperreexia, paresthesias, positive Chvosteks and Trousseaus signs, choreoathetosis, tetany c. Cardiovascular: premature ventricular beats, ventricular brillation and tachycardia, Torsades de pointes, predisposition to digoxin-mediated arrhythmias, PR prolongation and QT prolongation 4. Treatment a. Table 6. IV Dosage Guidelines for Hypomagnesemia
Serum Mg (mg/dL) 1.51.8 1.11.4 < 1.0 Dose (mEq/kg) 0.5 1 1.5 Infusion Rate (hours) 12 24 24

b. Oral dosage guidelines for hypomagnesemia

2008 American College of Clinical Pharmacy 1-361

Fluids, Electrolytes, and Nutrition i. Table 7. Oral Replacement Preparations


Mg Salt Sulfate Oxide Hydroxide Gluconate Elemental Mg Content per Salt Form (mg/g) 100 600 410 54 Diarrhea ++ ++ ++

C. Hypermagnesemia (more than 2.4 mEq/L) 1. Etiologies a. Decreased renal elimination b. Excessive intake c. Drugs 2. Table 8. Signs and Symptoms of Hypermagnesemia
Magnesium Serum Concentration (mEq/L) >4 47 710 > 10 > 15 Sign/Symptom Deep tendon reexes Hypotension, bradycardia, somnolence EKG changes, accid paralysis Voluntary muscle paralysis, apnea Respiratory paralysis, complete heart block

EKG = electrocardiographic.

3. Treatment of hypermagnesemia a. Discontinue supplemental magnesium b. Calcium administration c. Diuresis VIII. DISORDERS OF PHOSPHORUS HOMEOSTASIS A. Phosphorus 1. Biological functions and physiologic processes a. Important role in cell structure as a constituent of nucleotides (DNA), nucleoproteins, and membrane phospholipids b. Critical role in cell function because inorganic phosphorus (Pi) is required for: i. Normal glycolytic pathway functioning in red blood cells and other tissues ii. Production of phosphorylated intermediates including 2,3-diphosphoglycerate and ATP for maintenance of normal oxygen-hemoglobin dissociation and subsequent oxygen delivery to tissues c. Physiologic processes that have phosphorus-dependent metabolic pathways include, but are not limited to: i. Leukocyte phagocytic activity ii. Neurologic function iii. Skeletal, respiratory, and cardiac muscle function d. Phosphate is an important urinary buffer system that facilitates excretion of xed acids. 2. Pharmacokinetics (absorption, distribution, elimination) a. Absorption: normal adult dietary intake is about 1 g/day, of which 70%90% is absorbed. b. Adult body stores: i. Under normal conditions, there are about 500800 g, of which around 80% is stored in bones/teeth and around 9% is located in skeletal muscle/viscera. 2008 American College of Clinical Pharmacy 1-362

Fluids, Electrolytes, and Nutrition ii. Storage is in the organic form of phospholipids, phosphosugars, and phosphoproteins. iii. Most of the phosphorus located in skeletal muscle and viscera is intracellular. (a) Phosphorus is the major intracellular anion. (b) Serum phosphorus is not a reliable indicator of total body phosphate stores because phosphate is mainly an intracellular ion. iv. Only a small portion of the phosphorus pool is Pi and subsequently available for synthesis of intracellular energy compounds (i.e., ATP). c. Serum phosphorus is tightly regulated and maintained within a narrow range. i. Diurnal variation ii. Variation with meal ingestion (mean decrease of 0.25 mg/dL below fasting level common after carbohydrate and fat meals) iii. Serum concentrations drawn before breakfast are the most reliable. d. Hormonal controls i. 1,25-dihydroxyvitamin D3 (calcitriol) (a) Stimulates bone phosphate resorption (b) Increases GI phosphate absorption ii. Parathyroid hormone (a) Increases phosphate excretion and calcium in renal proximal tubule (b) Stimulates 1--hydroxylation of vitamin D to calcitriol e. Excretion of absorbed phosphate is around 90% in urine and around 10% in feces. B. Hypophosphatemia (less than 2.5 mg/dL) 1. Mechanisms and etiologies a. Nutritional i. Decreased intake through dietary deciency (rarely occurs) does not cause signicant hypophosphatemia by itself, but when a stress is imposed on the patient, acute severe hypophosphatemia can result. ii. Starvation is associated with catabolic release of intracellular phosphate, which is subsequently lost in the urine (maintaining normal serum phosphorus). iii. Ineffective GI absorption caused by medications (i.e., chronic antacid therapy or sucralfate) binding phosphorus in the GI tract and inhibiting absorption of ingested and secreted phosphorus iv. GI disorders (Crohns disease) causing malabsorption, maldigestion (pancreatic insufciency), or steatorrhea (postgastrectomy), all of which decrease calcium and vitamin D absorption, inducing secondary hyperparathyroidism and increased phosphaturia b. Increased loss i. Renal (a) Intrinsic (renal tubular abnormality): Fanconis syndrome and vitamin Dresistant rickets (b) Extrinsic (inhibition of phosphate reabsorption): hyperparathyroidism, acute expansion of ECF volume with saline or sodium bicarbonate, theophylline, glycosuria, hypokalemia, and hypomagnesemia ii. Nonrenal (a) GI losses: prolonged nasogastric suctioning, vomiting, and chronic diarrhea (b) Hemodialysis against a phosphate-poor dialysis solution (c) Renal losses do not account for the hypophosphatemia seen in patients with refeeding syndrome (urinary phosphorus concentrations decline to zero, and tubular reabsorption of phosphorus increases in this setting). c. Transcellular shifts

2008 American College of Clinical Pharmacy 1-363

Fluids, Electrolytes, and Nutrition i. Acute shift of phosphate from the extracellular to the intracellular compartment ii. The primary mechanism causing severe hypophosphatemia, especially in patients who are already phosphorus depleted iii. Intracellular Pi is used to form phosphorylated intermediates during glycolysis, glycogenolysis, oxidative phosphorylation, and the synthesis of protein and glycogen, which causes a depletion of intracellular Pi. iv. Extracellular Pi shifts inside the cell, leading to hypophosphatemia. v. Anabolism (synthesis of protoplasm) requires deposition of nitrogen in parallel with phosphorus (ratio of 0.07 g of phosphorus/1 g of nitrogen) as well as sodium and potassium. vi. Anabolism and glycolysis, precipitating cellular uptake of phosphorus, are stimulated by the administration of carbohydrates and amino acids. vii. Severe respiratory alkalosis is also thought to work by this mechanism of increased formation of phosphorylated organic compounds with subsequent intracellular shifting of Pi. viii. Metabolic alkalosis does not cause as profound a hypophosphatemia. ix. Medications such as insulin and 2-inhaled agonists can cause an intracellular shift. 2. Clinical manifestations of hypophosphatemia a. Central nervous system: acute areexic paralysis, paresthesias, dysarthria, weakness, numbness, seizures, confusion, coma, altered mental status, myalgias, Guillain-Barrelike syndrome, lethargy, rhabdomyolysis, and hyporeexia b. Pulmonary: respiratory muscle fatigue, decreased diaphragmatic contractility, ventilator dependence c. GI: anorexia, emesis, nausea d. Hematologic: decreased diphosphoglycerate, causing increased oxygen afnity for hemoglobin and increased red blood cell count rigidity, thrombocytopenia, and white blood cell count dysfunction e. Cardiovascular: arrhythmias, congestive heart failure, decreased cardiac output, and sudden death 3. Treatment a. Table 9. Intravenous Dosage Guidelines for Hypophosphatemia
Phosphorus (mg/dL) 2.33 1.62.2 1.5 Dose (mmol/kg) 0.32 0.64 1 Infusion Time (hours) 46 46 812

b. Oral dosing guidelines for hypophosphatemia i. Table 10. Oral Replacement Preparations
Product (mmol) Neutra Phos caplets Neutra Phos K caplets Neutra Phos powder Neutra Phos K powder Fleets Phosphosoda (per mL) Potassium (mmol) 7.13 14.25 7.13 14.25 0 Sodium (mmol) 7.13 0 7.13 0 4.8 Phosphorus (mmol) 8 8 8 8 4.1

ii. Must provide 2040 mmol per day because of poor oral absorption iii. Best option: 510 mL of Fleets Phosphosoda 2008 American College of Clinical Pharmacy 1-364

Fluids, Electrolytes, and Nutrition

C. Hyperphosphatemia (more than 4.5 mg/dL) 1. Etiologies a. Renal dysfunction: primary glomerular or tubulointerstitial disease, primary hypoparathyroidism b. Intracellular to extracellular shift: hemolysis, rhabdomyolysis, tumor lysis syndrome c. Drugs: antacids, enemas, or laxatives 1. Signs and symptoms of hyperphosphatemia a. Attributable to hypocalcemia: neuromuscular (paresthesias, tetany, seizures), cardiac (arrhythmia, hypotension) b. Serum Ca (mg/dL) P (mg/dL) product more than 60: increased risk of ectopic calcication in the heart, kidney, GI tract, and skin 2. Treatment of hyperphosphatemia a. Oral phosphate binders: Al(OH), calcium carbonate, calcium acetate b. Sucralfate c. Extracellular volume expansion (to achieve urinary output of more than 3 L/day) d. Hemodialysis

IX. DISORDERS OF CALCIUM HOMEOSTASIS A. Biological functions and physiologic processes 1. Bone metabolism 2. Blood coagulation 3. Platelet adhesion 4. Neuromuscular activity 5. Electrophysiology of the heart and smooth muscles B. Pharmacokinetics (absorption, distribution, elimination) 1. About 99% of Ca++ is in the bones, with less than 1% residing in the serum. 2. Ca++ in the blood is 45% bound to plasma proteins, primarily albumin. 3. Free, ionized calcium accounts for 45% of total plasma calcium; it is the free, unbound, ionized form that is physiologically active. 4. The remaining 10% of total plasma calcium is complexed to various anions, including bicarbonate, citrate, and phosphate. 5. Because Ca++ is highly bound to albumin, hypoalbuminemia can cause a decrease in total serum Ca++. a. For each 1-g/dL decrease in serum albumin concentration below 4 g/dL, total serum Ca++ decreases by about 0.8 mg/dL. b. Corrected serum Ca++ formula: i. Measured Ca++ (mg/dL) + [0.8 (4 measured albumin (g/dL))] 6. Ionized serum Ca++ is affected by acid-base status. a. Metabolic alkalosis increases Ca++ binding to plasma proteins, thus reducing the ionized serum Ca++. b. Metabolic acidosis decreases Ca++ binding to plasma proteins, thus increasing the ionized serum Ca++. C. Hypocalcemia (less than 8.6 mg/dL) 1. Mechanisms and etiologies a. Increased sequestration

2008 American College of Clinical Pharmacy 1-365

Fluids, Electrolytes, and Nutrition Hyperphosphatemia: Ca P product more than 60, soft tissue deposition of Ca-P complexes ii. Chelation: citrate (from transfusion or hemodialysis) iii. Soft tissue deposition: in acute pancreatitis and rhabdomyolysis iv Bone deposition: osteoblastic metastases b. Decreased parathyroid action i. Destruction of parathyroid glands: surgery, radiation, cancer ii. Inhibition of parathyroid release: hypomagnesemia c. Resistance to parathyroid action i. Vitamin D deciency: nutritional, malabsorption ii. Pseudohypoparathyroidism d. Medications i. Plicamycin (mithramycin), loop diuretics, pentamidine, phenytoin, foscarnet, intravenous amino acids 2. Clinical manifestations of hypocalcemia a. Central nervous system: lethargy, depression, psychoses seizures b. Neuromuscular: Chvosteks sign, Trousseaus sign, paresthesias, muscle cramps, tetany c. Cardiovascular: arrhythmias (heart block, ventricular brillation), prolonged QT interval 3. Treatment a. Intravenous dosing guidelines for symptomatic hypocalcemia i. 12 g of calcium gluconate for 3060 minutes and then 1015 mg/kg for 46 hours ii. Avoid calcium chloride because of thrombophlebitis and risk of acidosis b. Oral dosing guidelines for asymptomatic hypocalcemia i. Table 11. Comparison of Selected Calcium Products
Calcium Salt Ca chloride Ca gluconate Ca citrate Ca carbonate Ca acetate Elemental Calcium 13.6 mEq = 272 mg = 1 g 4.5 mEq = 90 mg = 1 g 90 mg/g salt 210 mg/g salt 400 mg/g salt 250 mg/g salt Route Intravenous Intravenous Oral Oral Oral Oral

i.

ii. Oral calcium supplements should be supplemented as 12 g/day elemental calcium. iii. Foods high in calcium: milk, ice cream, cheese, canned salmon, fresh oysters D. Hypercalcemia (more than 10.5 mg/dL) 1. Mechanisms and etiologies a. Increased calcium absorption due to increased vitamin D: tuberculosis, histoplasmosis, milk-alkali syndrome b. Malignancy: breast cancer, lung cancer, multiple myeloma, non-Hodgkins lymphoma c. Parathyroid excess: parathyroid hyperplasia, adenoma d. Enhanced bone calcium mobilization (nonparathyroid mediated): hyperthyroidism, adrenal insufciency, pheochromocytoma e. Miscellaneous: prolonged immobilization, vitamin A toxicity 2. Manifestations of hypercalcemia a. Central nervous system: depression, confusion, psychosis, coma b. Neuromuscular: muscle weakness, decreased deep tendon reexes c. Cardiovascular: arrhythmias, hypertension, shortened QT interval d. GI: nausea, vomiting, pancreatitis e. Renal: nephrolithiasis, nephrocalcinosis, type II distal renal tubular acidosis

2008 American College of Clinical Pharmacy 1-366

Fluids, Electrolytes, and Nutrition 3. Treatment of acute, symptomatic hypercalcemia a. Extracellular volume expansion: rst line of therapy i. Normal saline: initially, 12 L for 1 hour, followed by 300500 mL/hour to maintain diuresis ii. Hydration can increase urinary calcium excretion. iii. Can add loop diuretic (furosemide 2040 every 23 hours) to increase calciuric effect of volume expansion iv. Monitor K+ and Mg++ due to losses from forced diuresis. b. Biphosphonates (etidronate, pamidronate, alendronate, zoledronate): inhibitors of bone resorption through action on osteoblast and osteoclast precursors i. Often used for hypercalcemia of malignancy ii. Dosing: (a) Pamidronate: 6090 mg intravenously for 224 hours 1 dose (b) Zoledronate: 48 mg intravenously for 15 minutes 1 dose (c) Etidronate: 7.5 mg/kg/day for 2 hours 37 days c. Calcitonin i. Moderate efcacy for hypercalcemia of carcinoma, multiple myeloma, and primary hyperparathyroidism ii. Dose: 4 IU/kg subcutaneously or intramuscularly every 12 hours d. Glucocorticoids i. Useful in decreasing excessive vitamin D production or calcium absorption; not helpful in hyperparathyroid states ii. Dose: prednisone 1030 mg/day orally; takes several days to 2 weeks for onset e. Plicamycin i. Useful in hypercalcemia of malignancy ii. Blocks bone resorption by inhibiting RNA synthesis in bone cells iii. Dose: 25 mcg/kg intravenously for 23 hours iv. Risk of increased toxicity (renal, hepatic, hematopoietic) with repeated doses f. Hemodialysis g. Avoid intravenous phosphates because of risk of soft tissue calcication.

2008 American College of Clinical Pharmacy 1-367

Fluids, Electrolytes, and Nutrition


Table 12. Drugs Tested for Compatibility with 2-in-1 Parenteral Nutrition Solutions Acyclovir 7 mg/mL D5W Hydromorphone 0.5 mg/mL D5W Hydroxyzine 2 mg/mL D5W Amikacin 5 mg/mL D5W Aminophylline 2.5 mg/mL D5W Imipenem/Cilastatin 10 mg/mL 0.9% NaCl Amphotericin B 0.6 mg/mL D5W Insulin 1 unit/mL D5W Ampicillin 20 mg/mL 0.9% NaCl Ifosfamide 25 mg/mL D5W Ampicillin/Sulbactam 20/10 mg/mL 0.9% NaCl Leucovorin 2 mg/mL D5W Aztreonam 40 mg/mL D5W Levorphanol 0.5 mg/mL D5W Lorazepam 0.1 mg/mL D5W Bumetanide 0.04 mg/mL D5W Buprenorphine 0.04 mg/mL D5W Magnesium sulfate 100 mg (0.81 mEq)/mL D5W Mannitol 15% undiluted solution (150 mg/mL) Butorphanol 0.04 mg/mL D5W Calcium gluconate 40 mg (0.19 mEq)/mL D5W Meperidine 4 mg/mL D5W Mesna 10 mg/mL D5W Carboplatin 5 mg/mL D5W Cefazolin 20 mg/mL D5W Methylprednisolone 5 mg/mL D5W Methotrexate 15 mg/mL D5W Cefonicid 20 mg/mL D5W Metoclopramide 5 mg/mL D5W Cefoperazone 40 mg/mL D5W Cefotaxime 20 mg/mL D5W Metronidazole 5 mg/mL undiluted Cefotetan 20 mg/mL D5W Mezlocillin 40 mg/mL D5W Cefoxitin 20 mg/mL D5W Miconazole 3.5 mg/mL D5W Midazolam 2 mg/mL D5W Ceftazidime 40 mg/mL D5W Minocycline 0.2 mg/mL D5W Ceftizoxime 20 mg/mL D5W Mitoxantrone 0.5 mg/mL D5W Ceftriaxone 20 mg/mL D5W Cefuroxime 30 mg/mL D5W Morphine 1 mg/mL D5W Chlorpromazine 2 mg/mL D5W Nafcillin 20 mg/mL D5W Cimetidine 12 mg/mL D5W Nalbuphine 10 mg/mL undiluted Ciprooxacin 1 mg/mL D5W Netilmicin 5 mg/mL D5W Cisplatin 1 mg/mL undiluted Nitroglycerin 400 mcg/mL D5W Nitroprusside 400 mcg/mL D5W Clindamycin 10 mg/mL D5W Cyclophosphamide 10 mg/mL D5W Norepinephrine 16 mcg/mL D5W Cyclosporine 5 mg/mL D5W Octreotide 10 mcg/mL D5W Cytarabine 50 mg/mL undiluted Ooxacin 4 mg/mL D5W Dexamethasone 1 mg/mL D5W Ondansetron 1 mg/mL D5W Digoxin 0.25 mg/mL undiluted Paclitaxel 1.2 mg/mL D5W Diphenhydramine 2 mg/mL D5W and 50 mg/mL undiluted Pentobarbital 5 mg/mL D5W Phenobarbital 5 mg/mL D5W Dobutamine 4 mg/mL D5W Dopamine 3200 mcg/mL D5W Piperacillin 40 mg/mL D5W Doxorubicin 2 mg/mL undiluted Piperacillin/Tazobactam 40/5 mg/mL D5W Doxycycline 1 mg/mL D5W Potassium chloride 0.1 mEq/mL D5W Potassium phosphates 3 mmol/mL undiluted Droperidol 0.4 mg/mL D5W Enalaprilat 0.1 mg/mL D5W Prochlorperazine 0.5 mg/mL D5W Promethazine 2 mg/mL D5W Famotidine 2 mg/mL D5W Fentanyl 12.5 mcg/mL D5W and 50 mcg/mL undiluted Ranitidine 2 mg/mL D5W Sodium bicarbonate 1 mEq/mL undiluted Fluconazole 2 mg/mL undiluted Fluorouracil 16 mg/mL D5W Sodium phosphates 3 mmol/mL undiluted Furosemide 3 mg/mL D5W Tacrolimus 1 mg/mL D5W Ganciclovir 20 mg/mL D5W Ticarcillin 30 mg/mL D5W Gentamicin 5 mg/mL D5W Ticarcillin/Clavulanate 30/0.1 mg/mL D5W Granisetron 50 mcg/mL D5W Tobramycin 5 mg/mL D5W Haloperidol 0.2 mg/mL D5W Trimethoprim/Sulfamethoxazole 0.8/4 mg/mL D5W Heparin 100 units/mL undiluted Vancomycin 10 mg/mL D5W Hydrocortisone 1 mg/mL D5W Zidovudine 4 mg/mL D5W Drugs listed in bold and italicized are incompatible: Trissel LA, Gilbert DL, Martinez JF, et al. Compatibility of parenteral nutrient solutions with selected drugs during simulated Y-site administration. Am J Health-Syst Pharm 1997;54:1295300.

2008 American College of Clinical Pharmacy 1-368

Fluids, Electrolytes, and Nutrition


Table 13. Drugs Tested for Compatibility with 3-in-1 Parenteral Nutrition Solutions Acyclovir 7 mg/mL D5W Hydroxyzine 2 mg/mL D5W Imipenem/Cilastatin 10 mg/mL 0.9% NaCl Amikacin 5 mg/mL D5W Aminophylline 2.5 mg/mL D5W Insulin 1 unit/mL D5W Amphotericin B 0.6 mg/mL D5W Ifosfamide 25 mg/mL D5W Ampicillin 20 mg/mL 0.9% NaCl Leucovorin 2 mg/mL D5W Levorphanol 0.5 mg/mL D5W Ampicillin/Sulbactam 20/10 mg/mL 0.9% NaCl Lorazepam 0.1 mg/mL D5W Aztreonam 40 mg/mL D5W Magnesium Sulfate 100 mg (0.81 mEq)/mL D5W Bumetanide 0.04 mg/mL D5W Buprenorphine 0.04 mg/mL D5W Mannitol 15% undiluted solution (150 mg/mL) Meperidine 4 mg/mL D5W Butorphanol 0.04 mg/mL D5W Calcium gluconate 40 mg (0.19 mEq)/mL D5W Meropenem 20 mg/mL D5W Mesna 10 mg/mL D5W Carboplatin 5 mg/mL D5W Cefazolin 20 mg/mL D5W Methylprednisolone 5 mg/mL D5W Cefonicid 20 mg/mL D5W Methotrexate 15 mg/mL D5W Metoclopramide 5 mg/mL D5W Cefoperazone 40 mg/mL D5W Cefotaxime 20 mg/mL D5W Metronidazole 5 mg/mL undiluted Cefotetan 20 mg/mL D5W Mezlocillin 40 mg/mL D5W Cefoxitin 20 mg/mL D5W Miconazole 3.5 mg/mL D5W Midazolam 2 mg/mL D5W Ceftazidime 40 mg/mL D5W Minocycline 0.2 mg/mL D5W Ceftizoxime 20 mg/mL D5W Ceftriaxone 20 mg/mL D5W Mitoxantrone 0.5 mg/mL D5W Morphine 1 mg/mL D5W and 15 mg/mL undiluted Cefuroxime 30 mg/mL D5W Chlorpromazine 2 mg/mL D5W Nafcillin 20 mg/mL D5W Nalbuphine 10 mg/mL undiluted Cimetidine 12 mg/mL D5W Ciprooxacin 1 mg/mL D5W Netilmicin 5 mg/mL D5W Cisplatin 1 mg/mL undiluted Nitroglycerin 400 mcg/mL D5W Nitroprusside 400 mcg/mL D5W Clindamycin 10 mg/mL D5W Cyclophosphamide 10 mg/mL D5W Norepinephrine 16 mcg/mL D5W Cyclosporine 5 mg/mL D5W Octreotide 10 mcg/mL D5W Cytarabine 50 mg/mL undiluted Ooxacin 4 mg/mL D5W Ondansetron 1 mg/mL D5W Dexamethasone 1 mg/mL D5W Digoxin 0.25 mg/mL undiluted Paclitaxel 1.2 mg/mL D5W Diphenhydramine 2 mg/mL D5W and 50 mg/mL undiluted Pentobarbital 5 mg/mL D5W Phenobarbital 5 mg/mL D5W Dobutamine 4 mg/mL D5W Dopamine 3200 mcg/mL D5W Piperacillin 40 mg/mL D5W Doxorubicin 2 mg/mL undiluted Piperacillin/Tazobactam 40/5 mg/mL D5W Doxycycline 1 mg/mL D5W Potassium chloride 0.1 mEq/mL D5W Potassium phosphates 3 mmol/mL undiluted Droperidol 0.4 mg/mL D5W Enalaprilat 0.1 mg/mL D5W Prochlorperazine 0.5 mg/mL D5W Famotidine 2 mg/mL D5W Promethazine 2 mg/mL D5W Fentanyl 12.5 mcg/mL D5W and 50 mcg/mL undiluted Ranitidine 2 mg/mL D5W Fluconazole 2 mg/mL undiluted Sodium Bicarbonate 1 mEq/mL undiluted Fluorouracil 16 mg/mL D5W Sodium phosphates 3 mmol/mL undiluted Furosemide 3 mg/mL D5W Tacrolimus 1 mg/mL D5W Ganciclovir 20 mg/mL D5W Ticarcillin 30 mg/mL D5W Gentamicin 5 mg/mL D5W Ticarcillin/Clavulanate 30/0.1 mg/mL D5W Granisetron 50 mcg/mL D5W Tobramycin 5 mg/mL D5W Haloperidol 0.2 mg/mL D5W Trimethoprim/Sulfamethoxazole 0.8/4 mg/mL D5W Heparin 100 units/mL undiluted Vancomycin 10 mg/mL D5W Hydrocortisone 1 mg/mL D5W Zidovudine 4 mg/mL D5W Hydromorphone 0.5 mg/mL D5W Drugs listed in bold and italicized are incompatible: Trissel LA, Gilbert DL, Martinez JF, et al. Compatibility of medications with 3-in-1 parenteral nutrition admixtures. JPEN 1999;23:6774.

2008 American College of Clinical Pharmacy 1-369

Fluids, Electrolytes, and Nutrition Table 14. Monitoring Guidelines for Patients Receiving Parenteral Nutrition
Parameter Baseline Monitoring X Every 3 days; then, 3 or 4 times/week PRN Chem-7a Ca, Phos, Mg X 2 or 3 times for 1 week; then, once or twice weekly Fingerstick glucose X PRN; in an unstable patient every 46 hours Liver function tests X Weekly Prealbumin and CRP X Weekly CBC w/differential X Weekly Triglyceride X Weekly PT/PTT X Weekly 24-hour urine for UUN X PRN; for persistently low prealbumin and CRP Weight X Daily Vital signs, uid intake/output X Daily a Chem-7 includes serum Na, K, Cl, CO2, BUN, creatinine, and glucose. A critically ill patient most likely would require a Chem-7 daily. BUN = blood urea nitrogen; CBC = complete blood cell count; CRP = C-reactive protein; PRN = as needed; PT = prothrombin time; PTT = partial thromboplastin time; UUN = urine urea nitrogen.

2008 American College of Clinical Pharmacy 1-370

Fluids, Electrolytes, and Nutrition REFERENCES


Enteral Nutrition 1. A.S.P.E.N. Board of Directors. Guidelines for the use of parenteral and enteral nutrition in adult and pediatric patients. J Parenter Enter Nutr JPEN 2002;26(suppl):1SA138SA 2. Edes TE, Walk BE, Austin JL. Diarrhea in tubefed patients: feeding formula not necessarily the cause. Am J Med 1990;88:913. 3. Kirby DF, Delegge MH, Fleming CR. American Gastroenterological Association technical review on tube feeding for enteral nutrition. Gastroenterology 1995;108:1282301. 4. Klein S, Kinney J, Jeejeebhoy K, et al. Nutrition support in clinical practice: review of published data and recommendations for future research directions. J Parenter Enter Nutr JPEN 1997;21:13356. 5. Kompan L, Kremzar B, Gadzijev E, et al. Effects of early enteral nutrition on intestinal permeability and the development of multiple organ failure after multiple injury. Intensive Care Med 1999;25:15761. 6. Kudsk KA, Croce MA, Fabian TC, et al. Enteral vs parenteral feeding: effects on septic morbidity following blunt and penetrating trauma. Ann Surg 1992;215:50313. 7. McMahon MM, Farnell MB, Murray MJ. Nutritional support of critically ill patients. Mayo Clin Proc 1993;68:91120. 8. Minard G, Kudsk KA. Nutritional support and infection: does the route matter? World J Surg 1998;22:2139. 9. Vanek V. Ins and outs of enteral access. I. Short-term enteral access. Nutr Clin Pract 2002;17:27583. 10. Vanek V. Ins and outs of enteral access. II. Long-term enteral accessesophagostomy and gastrostomy. Nutr Clin Pract 2003;18:5074. 11. Vanek V. Ins and outs of enteral access. III. Longterm enteral accessjejunostomy. Nutr Clin Pract 2003;18:20120. PN Initiation and Management 1. Driscoll DF, Blackburn GL. Total parenteral nutrition: a review of its current status in hospitalized patients and the need for patientspecic feeding. Drugs 1990;40:34663. 2. Eggert LD, Rusho WJ, MacKay MW, et al. Calcium and phosphorus compatibility in parenteral nutrition solutions for neonates. Am J Hosp Pharm 1982;39:4953. 3. Fitzgerald KA, MacKay MW. Calcium and phosphate solubility in neonatal parenteral nutrition solutions containing Aminosyn PF. Am J Hosp Pharm 1987;44:1396400. 4. Henry RS, Jurgens RW, Sturgeon R, et al. Compatibility of calcium chloride and calcium gluconate with sodium phosphate in a mixed PN solution. Am J Hosp Pharm 1980;37:6734. 5. Klein S, Kinney J, Jeejeebhoy K, et al. Nutrition support in clinical practice: review of published data and recommendations for future research directions. J Parenter Enter Nutr JPEN 1997;21:13356. 6. Lenz GT, Mikrut BA. Calcium and phosphate solubility in neonatal parenteral nutrient solutions containing Aminosyn-PF or TrophAmine. Am J Hosp Pharm 1988;45:236771. 7. Manzo CB, Dickerson RN. Parenteral nutrition monitoring in hospitalized patients. Hosp Pharm 1993;28:5618. 8. McMahon MM, Farnell MB, Murray MJ. Nutritional support of critically ill patients. Mayo Clin Proc 1993;68:91120. 9. Mikrut BA. Calcium and phosphate solubility in neonatal parenteral nutrient solutions containing Aminosyn PF or TrophAmine. Am J Hosp Pharm 1987:44:27024. 10. Schuetz DH, King JC. Compatibility and stability of electrolytes, vitamins and antibiotics in combination with 8% amino acids solution. Am J Hosp Pharm 1978;35:3344. 11. Task Force for the Revision of Safe Practices for Parenteral Nutrition. Safe practices for parenteral nutrition. JPEN J Parenter Enter Nutr 2004;28:S39S70. 12. Trissel LA, ed. Calcium and phosphate compatibility in parenteral nutrition, 1st ed. Houston: TriPharma Communications, 2001. Fluid & Electrolyte Requirements General 1. Arieff AL, DeFronzo RA. Fluid, Electrolyte and Acid-Base Disorders, vols. 1 and 2. New York: Churchill Livingstone, 1985. 2. Kraft MD, Btaiche IF, Sacks GSS, et al. Treatment of electrolyte disorders in adult patients in the intensive care unit. Am J Health-Syst Pharm 2005;62:166382.

2008 American College of Clinical Pharmacy 1-371

Fluids, Electrolytes, and Nutrition


3. 4. 5. Nanji AA. Drug-induced electrolyte disorders. Drug Intell Clin Pharm 1983;17:17585. Narins RG, Emmett M. Simple and mixed acidbase disorders: a practical approach. Medicine 1980;59:161187. Rose DB. Clinical Physiology of Acid-Base and Electrolyte Disorders, 1st ed. New York: McGraw-Hill, 1977. 5. 6. 7. Oster JR, Singer I, Fishman LM. Heparin-induced aldosterone suppression and hyperkalemia. Am J Med 1995;98:57586. Wong SL, Maltz HC. Albuterol for the treatment of hyperkalemia. Ann Pharmacother 1999;33:103. Parazella MA, Mahnensmith RL. Trimethoprimsulfamethoxazole: hyperkalemia is an important complication regardless of dose. Clin Nephrol 1996;46:18792. Reardon LC, Macpherson DS. Hyperkalemia in outpatients using angiotensin-converting enzyme inhibitors. How much should we worry? Arch Intern Med 1998;158:2632. Gennari J. Hypokalemia. N Engl J Med 1998;339:451. Schepkins H, Vanholder R, Billiouw JM, et al. Life-threatening hyperkalemia during combined therapy with angiotensin-converting enzyme inhibitors and spironolactone: an analysis of 25 cases. Am J Med 2001;110:43841. Cohn JN, Kowey PR, Whelton PK, et al. New guidelines for potassium replacement in clinical practice. A contemporary review by the National Council on Potassium in Clinical Practice. Arch Intern Med 2000;160:242936. Palmer BF. Managing hyperkalemia caused by inhibitors of the renin-angiotensin-aldosterone system. N Engl J Med 2004;351:58592. Institute of Medicine, Dietary reference intakes: water potassium, sodium, chloride and sulfate. Available at http://www.iom.edu/report. asp?id=18495. Accessed 2/25/2008

Sodium 1. Androgue HJ, Madias NE. Medical progress: management of life-threatening acid-base disorders: rst of two parts. N Engl J Med 1998;338:2634. 2. Androgue HJ, Madias NE. Medical progress: management of life-threatening acid-base disorders: second of two parts. N Engl J Med 1998;338:107111. 3. Cluitmans FH, Meinders AE. Management of severe hyponatremia: rapid or slow corrections? Am J Med 1990;88:1616. 4. Kinzie BJ. Management of the syndrome of inappropriate secretion of antidiuretic hormone. Clin Pharm 1987;6:62533. 5. Oh MS, Carroll HJ. Disorders of sodium metabolism: hypernatremia and hyponatremia. Crit Care Med 1992;20:94103. 6. Sterns RH. Severe hyponatremia: the case for conservative management. Crit Care Med 1992;20:5349. 7. Sterns RH. The management of hyponatremic emergencies. Crit Care Clin 1991;7:12742. 8. Sunyecz L, Mirtallo JM. Sodium imbalance in a patient receiving total parenteral nutrition. Clin Pharm 1993;12:13849. Potassium 1. Greenberg A. Hyperkalemia: treatment options. Semin Nephrol 1998;18:46. 2. Saggar-Malik AK, Cappuccio FP. Potassium supplements and potassium-sparing diuretics: a review and guide to appropriate use. Drugs 1993;46:9861008. 3. Sawaya BP, Briggs JP, Schnermann J. Amphotericin B nephrotoxicity: the adverse consequences of altered membrane properties. J Am Soc Nephrol 1995;6:15464. 4. Alappan R, Perazella MA, Buller GK. Hyperkalemia in hospitalized patients with trimethoprim-sulfamethoxazole. Ann Intern Med 1996;124:31620.

8.

9. 10.

11.

12. 13.

Magnesium 1. Al-Ghamdi SMG, Cameron EC, Sutton RAL. Magnesium deciency: pathophysiologic and clinical overview. Am J Kidney Dis 1994;24:73752. 2. Dickerson RN. Treating hypomagnesemia. Hosp Pharm 1985;20:7613. 3. Gums JG. Clinical signicance of magnesium: a review. Drug Intell Clin Pharm 1987;21:2406. 4. Lee C, Zaloga GP. Magnesium metabolism. Semin Respir Med 1985;7:7580. 5. Salem M, Munoz R, Chernow B. Hypomagnesemia in critical illness: a common and clinically important problem. Crit Care Clin 1991;7:22552. 6. Whang R, Whang DD, Ryan MP. Refractory potassium repletion. Arch Intern Med 1992;152:405.

2008 American College of Clinical Pharmacy 1-372

Fluids, Electrolytes, and Nutrition


7. Sacks GS, Brown RO, Dickerson RN, et al. Mononuclear blood cell magnesium content and serum magnesium concentration in critically ill hypomagnesemic patients after replacement therapy. Nutrition 1997;13:3038.

Phosphorus 1. Clark C, Sacks GS, Dickerson RN, et al. Treatment of hypophosphatemia in patients receiving specialized nutrition support using a graduated intravenous dosing scheme: results from a prospective clinical trial. Crit Care Med 1995;23:150411. 2. Brown KA, Dickerson RN, Morgan LM, et al. A new graduated dosing regimen for phosphorus replacement in patients receiving nutrition support. JPEN 2006;30:20914. 3. Peppers MP, Geheb M, Desai T. Hypophosphatemia and hyperphosphatemia. Crit Care Clin 1991;7:20114. 4. Vannatta JB, Whang R, Papper S. Efcacy of intravenous phosphorus therapy in the severely hypophosphatemic patient. Arch Intern Med 1981;141:8857. 5. Vannatta JB, Andress DL, Whang R, et al. Highdose intravenous phosphorus therapy for severe complicated hypophosphatemia. South Med J 1983;76:14246.

2008 American College of Clinical Pharmacy 1-373

Fluids, Electrolytes, and Nutrition


ANSWERS TO SELF-ASSESSMENT QUESTIONS

1. Answer: A Step 1: Elevated pH indicates the presence of alkalemia. Step 2: The primary process is metabolic because the HCO3 is elevated and the PCO2 is not decreased. Step 3: Calculate the gaps. anion gap (AG) = 144 (39 + 94) = 11 mEq/L; a normal AG = 10 4 mEq/L; thus, the AG is normal. Step 4: Check for the degree of compensation. The increase in PCO2 is 8.4 (0.6 14); thus, 40 + 8.4 = 48.4. The calculated increase in PCO2 of 48.4 mm Hg is equal to the measured PCO2 of 48 mm Hg; thus, the metabolic alkalemia is fully compensated. Step 5: Determine the 1:1 relationship. This calculation is not necessary. 2. Answer: C Step 1: Acidemia is present because the pH is less than 7.35. Step 2: The primary process is metabolic because the bicarbonate is decreased and the PCO2 is not increased. Step 3: The anion gap is dramatically elevated: 140 (105 + 5) = 30. (normal AG = 10 4) Step 4: Compensation for metabolic acidosis is calculated by the following formula: PCO2 = 1.3 decrease in HCO3. The decrease in bicarbonate is by 20 (25 5 = 20), so the PCO2 should decrease to 14 mm Hg (PCO2 = 1.3 20 = 26; 40 26 = 14). The compensation for the metabolic acidemia is not complete because the PCO2 of 19 mm Hg is slightly high for normal compensation. Thus, the PCO2 is consistent with a mild superimposed respiratory acidosis. A second disorder, respiratory acidosis, is present. Step 5: 1:1 RelationshipThe bicarbonate decrease of 20 (25 5 = 20) is close enough to the AG of 18 (30 12 = 18) to exclude an underlying metabolic alkalosis. 3. Answer: A Step 1: Alkalemia is present because the pH is more than 7.45. Step 2: The primary process is respiratory because the PCO2 is less than 40 mm Hg and the HCO3 is not increased. Step 3: The AG is normal [135 (103 + 22) = 10]. Step 4: Compensation for respiratory alkalemia is calculated by the following formula: the HCO3 decreases by 2 mEq/L for every 10-mm Hg decrease in PCO2 (for acute respiratory alkalosis).

Because the decrease in PCO2 is 15 (40 25 = 15 mm Hg), the calculated decrease is 3. In this case, the calculated decrease is identical to the actual decrease (25 22 = 3), so only acute respiratory alkalemia is present with normal compensation. If the situation is chronic (more than 3 days), the degree of compensation varies: for every PCO2 decrease of 10 mm Hg, the HCO3 decreases by 4 mEq/L. If the calculation of the degree of metabolic compensation is not close to the actual value for the HCO3 ( 2 mEq/L), an additional primary metabolic disorder is present. Step 5: 1:1 RelationshipBecause the AG is normal, the bicarbonate decrease of 3 (25 22 = 3) is the same as the chloride increase of 3 (103 100 = 3). Thus, no underlying metabolic alkalosis exists. 4. Answer: D Free water restriction is the best treatment as patients with SIADH have impaired free-water excretion, with normal excretion of sodium. Continued intake of water or hypotonic uids will worsen the hyponatremia. 5. Answer: C Hypomagnesemia can result in refractory hypokalemia due to accelerated renal potassium loss or impairment of the sodium-potassium ATPase pump. 6. Answer: E Causes of hypomagnesemia include excessive GI uid losses rich in magnesium, excessive renal losses from medications (i.e., amphotericin B) or underlying conditions (i.e., thermal injury, alcoholism), and inadequate intake (alcoholism). 7. Answer: D The aggressive treatment of hypophosphatemia can lead to transiently high serum phosphorus concentrations that may result in precipitation of calcium phosphate salts and soft tissue (metastatic) calcication with symptomatic hypocalcemia. 8. Answer: D Bacterial translocation represents migration from the intestinal lumen to gut lymphatic or the portal venous. Studies suggest that early enteral feeding may protect the mucosal barrier. 9. Answer: D Prospective randomized clinical trials in trauma patients have shown that enteral nutrition compared with PN can decrease postoperative infections

2008 American College of Clinical Pharmacy 1-374

Fluids, Electrolytes, and Nutrition


including pneumonia, intraabdominal abscesses, and catheter-related line infections 10. Answer: C Enteral nutrition prevents the structural and functional alterations associated with bowel rest and the lack of luminal nutrients 11. Answer: B Due to the metabolic response to injury that occurs in sepsis, trauma and head injury, there is an increase release in pro-inammatory cytokines (glucagon, cortisol, catecholamines) which are catabolic and promote lean tissue breakdown. The delivery of aggressive nutrition support does not overcome this hormonal release, thus lean tissue breakdown persists even with the delivery of nutrition support. However, this hormonal environment does not exist in starvation, thus the provision of nutrition support does reduce lean tissue breakdown during starvation. 12. Answer: A The presence of luminal nutrients in the intestinal stimulates the secretion of immunoglobulin A (IgA), an immunoglobulin that can prevent bacterial adherence and translocation from the intestine across the mucosal and into the circulation.

2008 American College of Clinical Pharmacy 1-375

You might also like